Series 7

अब Quizwiz के साथ अपने होमवर्क और परीक्षाओं को एस करें!

Your FINRA member firm takes 100 GO bonds from a municipal bond dealer out-firm for one hour. This means that

your firm controls the bonds for one hour. A municipal securities dealer may quote a bond price that is firm for a certain time. This is called an out-firm with recall quote. Generally, these quotes are firm for an hour (or half hour) with a five-minute recall period. During this time, the municipal dealer cannot offer those bonds to anyone else—they are under the control of your firm.

If a customer fails to meet a Regulation T margin call of $2,500, securities may be sold out of the account with a value of

$5,000. Securities valued at twice the Regulation T cash call must be sold out if a customer fails to meet a Regulation T margin call ($2,500 × 2 = $5,000).

The underwriting manager of a Western underwriting syndicate has committed to sell $250,000 worth of bonds out of a total offering of $1 million. There are 10 underwriters of this new issue. The firm sells its entire share, but $100,000 worth of the total bonds remains unsold. What is the manager's remaining liability?

$0 In a divided (Western) syndicate, each syndicate member is responsible only for its own underwriting obligation. Because the underwriting manager sold its share, it is not liable for the unsold bonds. Rather, the manager will confirm the bonds to each member that did not sell its participation.

A limited partner (LP) invests $100,000 in a movie production limited partnership with a nonrecourse note for $300,000. The partnership liquidates, and the LP receives $100,000. His loss, for tax purposes, is

$0 LPs are liable for their investments and any shares of recourse debt. They are not liable for nonrecourse debt. Because the LP received the full amount of his original investment at the liquidation of the partnership, he has no loss to declare.

Automatic exercise will occur for equity options at expiration that are in the money by at least

$0.01. Automatic exercise will occur for equity options at expiration that are in the money by at least 0.01, unless specific instructions are given by the customer not to do so.

XYZ Corporation, whose common stock is currently selling for $40 per share, is having a rights offering. The terms of the offering require 10 rights plus $35 to subscribe to one share of stock. Compute the theoretical value of a right before the ex-rights date.

$0.45 Because the stock is trading with rights (before the ex-rights date), the formula is (M ‒ S ) divided by (N + 1). Plugging the numbers in, we have ($40 ‒ $35) divided by (10 + 1) = $5.00 ÷ 11 = $0.45.

A customer's restricted margin account shows the following: LMV $30,000 DB $16,000 SMA $0 If the customer sells $2,000 of securities, how much could be withdrawn from the account?

$1,000 In this restricted account, half of the sales proceeds will be used to reduce the DB balance to $15,000 and half of the sales proceeds are released to the special memorandum account (SMA). Therefore, when $2,000 of stock is sold, $1,000 is credited to SMA. This is the amount that can be withdrawn from the account.

A 38-year-old investor places $25,000 into a single premium deferred variable annuity. Twenty-two years later, with the account valued at $72,000, the investor surrenders the policy. If the investor is in the 25% marginal income tax bracket, the total tax liability is

$11,750. Only the deferred growth is taxable. In this case, it is the difference between the surrender value of $72,000 and the cost basis of $25,000. That $47,000 is taxed at the marginal rate of 25%. Because the investor is older than 59½ (38 + 22 = 60), there is no additional 10% penalty tax. Effectively, this is a 25% tax on $47,000.

Your customer opened a Roth IRA on February 2 and deposited $5,000. In August of the same the year, the value of the account had risen to $6,000, but by December, the value declined to $5,100 and the customer closed the securities positions and withdrew the entire amount. If the customer was age 55 at the time of the withdrawal, what would be the early withdrawal penalty?

$10 The customer would have an early withdrawal penalty of $10. A 10% early withdrawal penalty applies to the taxable amount withdrawn for those aged less than 59½.In a Roth IRA, the investment is made with after-tax dollars. Therefore, only the increase above the investment is subject to an early withdrawal penalty. In this case, the earnings of $100 would be taxable and have a 10% penalty equaling $10.

What is the profit to a syndicate member if a syndicate is offering an 8.5% bond at 100, the syndicate manager is giving a 0.75 concession and a 1-point total takedown, and the syndicate member sells 1,000 bonds?

$10,000 When a member of the syndicate sells a bond, the member is entitled to the total takedown. In this case, one point ($10) per bond (1,000 bonds sold × $10 per bond = $10,000 profit). Remember that the concession would only go to those who are not members of the syndicate but are part of the selling group instead.

A customer of a member firm has just invested $100,000 into an equipment leasing DPP. Under FINRA rules, the maximum compensation allowable to the firm is

$10,000. FINRA Rule 2310 limits compensation on the sale of a DPP to 10% of the offering price. That is the largest component of the offering expenses. Those are limited to 15% of the offering. DPPs are not covered by the 5% policy. Therefore, of the $100,000 invested, the business (the equipment leasing program) must receive at least $85,000 ($100,000 minus the maximum offering expenses of $15,000). If this question was about a DPP roll-up, then there is 2% maximum to the member if recommending the client vote in favor of the roll-up.

In early September, a customer buys 100 shares of QRS stock for $83 per share and simultaneously writes 1 QRS Mar 90 call for $4 per share. If the QRS Mar 90 call was exercised and the QRS stock delivered, what would be the customer's per-share profit?

$11. If the stock rises above $90, the writer will be exercised and make $700 on the stock (buy at $83, deliver at $90) and keep the $400 received in premiums. Alternatively, the breakeven point is $79 ($83 − $4), and the stock was sold (delivered) at $90 for an 11-point gain.

A shareholder invested in a mutual fund and has signed a letter of intent to invest $25,000. Her original investment was $13,000, and her current account value is $17,000. For her to complete the letter, she must deposit

$12,000. Under a letter of intent, the full contribution is required for the letter to be completed. Appreciation is not considered.

One of your customers owns five JLO 5s of 2042. The debentures have a conversion price of $15. When the market price of the convertible is 80, the parity price of the stock is

$12.00. A debenture with a conversion price of $15 is convertible into 66.66 shares ($1,000 ÷ $15). It is always the par value that is used, not the market price. To determine the parity price of the stock, divide the current market price of $800 by 66.66 and the answer rounds off to $12. Some students find it easier to recognize that the bond is 20% below its par value. To be equal (and that is what parity means), the stock must be 20% below the $15 conversion price (or 80% of it). Reducing $15 by 20% is a $3 reduction to $12 or taking 80% of $15 equals $12.

A 38-year-old investor places $25,000 into a single premium deferred variable annuity. Twenty years later, with the account valued at $72,000, the investor surrenders the policy. If the investor is in the 25% marginal income tax bracket, the total tax liability is

$16,450. Only the deferred growth is taxable. In this case, it is the difference between the surrender value of $72,000 and the cost basis of $25,000. That $47,000 is taxed at the marginal rate of 25%. Furthermore, because the investor is younger than 59½ (38 + 20 = 58), there is the additional 10% penalty tax. Effectively, this is a 35% tax on $47,000.

Your customer has made a margin purchase of 100 shares of DMF at 50. Two days later, before the customer has met his call, the current market value of DMF is 60. How much must your customer now deposit? (Regulation T is 50%.)

$2,500 The investor must come up with the initial call of $2,500. The amount of margin required for a new purchase is based on the current market value of the security at the time of purchase.

An investor has a short margin account. The current market value of the stock is $1.50 per share, and the investor owns 1,000 shares. Compliance with SRO minimum maintenance requirements is met with account equity of

$2,500. For stock trading under $5 per share, a customer must maintain 100% of SMV or $2.50 per share, whichever is greater. $2,500 (1,000 shares @$2.50) is greater than 100% of $1,500.

An investor with no other positions buys 1 CDE May 65 put at 3.50. If the investor buys the stock at 63.50 and exercises the put, what is the investor's profit or loss?

$200 loss The investor has the right to sell the stock for 65 when it is currently worth 63.50 for a gain of 1.50. The investor paid a premium of 3.50 minus the gain of 1.50 for a loss of 2 (2 × 100 = $200).

A municipal bond is purchased at a discount in the secondary market at 90. The face amount is $10,000, and the bond has 10 years to maturity. If the bond is sold for 97 after five years, what is the taxable gain?

$200. When a municipal bond is bought at a discount in the secondary market, the discount is accreted and taxable as ordinary income. Accretion increases cost basis. Therefore, five years later, the bond's cost basis is 95. At that point, the customer has a two-point capital gain. Had the bond been bought as an original issue discount, the annual accretion is considered interest income and is not taxable.

On April 15, 2016, your client purchased a variable life insurance policy with a death benefit of $450,000. The November 2019 statement showed a cash value of $28,000. If the client wanted to borrow as much as possible, the insurance company would have to allow a loan of at least

$21,000. Once a variable life policy is in force for a minimum of three years (this one is a bit longer than that), there is a requirement to make the loan provision available. At the three-year mark, that minimum becomes 75% of the computed cash value. Seventy-five percent of cash value of $28,000 is $21,000.

Your client writes 2 ABC Nov 220 calls at 5 and buys 200 shares of ABC common stock at $220 in his margin account. What is the breakeven point for the client's position?

$215 The breakeven point for covered call writing is the cost of stock purchased less the premium (220 − 5). Breakeven is the same if it is one covered call, or 1,000.

If a customer buys 500 shares of ABC at 48 and writes 5 ABC 50 calls at 2, what is the maximum loss?

$23,000. The investor pays $48 per share for the stock and receives $2 for selling the calls. The maximum loss is $48 per share minus the option premium collected, or ($48 minus $2) times 500 shares equals $23,000.

A 7% convertible debenture is selling at 101. It is convertible into the common stock of the same corporation at $25. The common stock is currently trading at $23. If the stock were trading at parity with the debenture, the price of the stock would be

$25.25 To determine the parity price of the common, first find the number of shares the debenture is convertible into (conversion ratio) by dividing par value by the conversion price ($1,000 / $25 = 40 shares). Next, divide the current price of the bond by the conversion ratio. The result is the parity price of the common stock. (1,010 / 40 = $25.25).

An investor opens the following position: Write 1 CDE Oct 30 call at 3.30 Buy 1 CDE Oct 40 call at 0.10 The maximum gain is

$320 The maximum gain on a credit spread is the net credit received (3.30 − 0.10 = 3.20 × 100 shares = $320).

With no other positions, a customer sells short 100 TIP at 40 and sells 1 TIP Oct 40 put at 5. At what stock price will the customer break even?

$45 On the downside, the short position fully covers the short put, and the profit is the $500 premium. On the upside, above $40, the short put expires, and the short stock position loses money. The first five points of loss (40 to 45) on the short stock position are offset by the premiums received. Above $45, losses begin and are potentially unlimited. The other way to look at this is with the T-chart. When the stock is sold short, $4,000 is credited to the account. When the put is sold, $500 is credited to the account. That means $4,500 has been credited. How much needs to go on the debit side (the other side of the T to make things even?) $4,500, so when the stock rises to $45 per share, the investor breaks even. Anything above that and losses begin.

An investor with no other positions buys 1 DWQ May 75 call at 6.50. If the investor exercises the call when the stock is trading at 77 and immediately sells the stock in the market, what is the investor's profit or loss?

$450 loss. The investor exercised the right to buy the stock for 75 and can sell the stock in the market for 77, for a gain of 2. The investor paid a premium of 6.50 minus the gain of 2, which gives the investor a loss of 4.50 (4.50 × 100 = $450).

A $50,000 20-year 7% municipal bond with semi-annual M/S coupon payments is issued on March 1, 2020. The full price for a trade of this bond, with a 7% yield to maturity to settle on June 30, 2026, is closest to

$51,156.94. The full price of a bond includes the accrued interest. First, we calculate the number of days of accrued interest. Because this is a municipal bond, each month has 30 days. Accrued interest is always paid up to, but not including, the settlement date. That means 30 days each for March, April, and May (90 days). Because we do not pay accrued interest for June 30, (on settlement date, the new owner is entitled to the entire six months of interest), there are 29 more days, giving us a total of 119 days. You can set it up like this: 6/30 minus 3/01 = 3 months, 29 days = 119 days. The next step is computing the amount of accrued interest in dollars and cents. With a coupon of 7% and semi-annual payments, each payment is 3.5% of the $50,000 par value. That is $1,750 each six months. We are going to solve for 119/180 days' worth of accrued interest. With the test center calculator, multiply $1,750 times 119 and divide the product by 180. The result is accrued interest of $1,156.94. Alternatively, you could find the interest for each day by dividing the semiannual interest of $1,750 by 180 days = $9.72222 per day. Multiply that times 119 days and the product is $1,156.94. Finally, when a bond with a 7% coupon has a yield to maturity of 7%, that tells us the bond is selling at par. All we need to do now is add the accrued interest to the $50,000 par value to arrive at $51,156.94.

A customer invests $20,000 in a direct participation program and signs a recourse note for $50,000. During the first year of operation, the customer receives a cash distribution of $15,000 from the partnership. At year's end, the customer receives a K-1 statement reporting his share of partnership losses of $75,000. How much of the loss may the customer deduct from passive income?

$55,000.00 A limited partner can only deduct partnership losses to the extent of his basis. To determine basis, add the original investment ($20,000) to any recourse debt assumed by the investor ($50,000). Recourse debt adds to basis as the partner is liable for this amount. Cash distributions received reduce basis ($15,000). At year's end, the investor's basis and the amount he can deduct from passive income is $55,000.

One of your customers takes a short position in 300 shares of LOP common stock. The sale price is $70 per share. One month later, with the stock selling at $73 per share, the investor purchases 3 LOP Sep 75 calls at a premium of 3.25. What is the investor's breakeven point?

$66.75 per share This customer is looking for the price of the stock to decline. The proceeds of the sale were $70 per share and the cost of the "insurance" (the call option) was 3.25. That means that the customer will not start making money until the stock falls below proceeds minus cost or 66.75. In a question like this, the current market price of the stock and the strike price of the option are irrelevant. Breakeven on short stock and a long call position is the proceeds minus the premium. As is always the case when computing breakeven, the number of shares and number of option contracts is meaninglessbreakeven is the same price for one or one thousand.

Your client's position is: Long 1 CYR 120 call at 4 Long 1 CYR 120 put at 3 Long 100 CYR purchased at 120 If the current market price of CYR is 120, what is the client's maximum possible loss?

$700 If the CYR stock drops to $0, the customer loses $12,000 on the long stock position but retains the right to put the stock to someone at $12,000 to prevent loss beyond the premium of $300. The call would expire out of the money, for a total loss of $700 (loss of both premiums paid).

If an investor sells 1 TCB Jan 50 put at 7 on July 15, 2019, and the put expires unexercised, what are the tax consequences?

$700 short-term capital gain reportable for tax year 2020 For tax purposes, any premiums earned are recognized at the expiration date. In this case, the January contract sold in July 2019 will expire in January 2020. Options writers always have short-term gains or losses.

A customer buys 200 XYZ at 39 and writes 2 XYZ Feb 40 calls at 3. When the stock rises to 44, the customer is exercised for a gain of

$800 The customer bought 200 shares at 39 and was forced to sell them at 40 for a $200 gain. In addition, the customer received $600 in premium income, so the overall gain is $800. Alternatively, the breakeven point for covered call writing is cost of shares purchased less premium received (39 − 3 = 36). As the customer is bullish, gain occurs above 36. However, for this customer, the stock can go no higher than 40 because she will be exercised (40 − 36 = 4 points × 200 shares = $800).

If a customer buys 1 ABC Jan 60 put at 6 and writes 1 ABC Jan 75 put at 13, the maximum loss is

$800. This is a credit spread. (More premium was received than was paid.) The maximum gain to a seller is the premium received (net credit of 7). In a spread, the maximum gain plus the maximum loss equals the difference in strike prices (75 − 60 = 15). Therefore, 15 minus the maximum gain of 7 equals the maximum loss of 8 multiplied by $100, or $800.

An investor buys 100 shares of QRS stock at $60 and writes 1 QRS 60 call at 4 and 1 QRS 60 put at 5. If QRS stock is trading at $74 on the expiration date, the investor realizes a profit of

$900. An investor can write (short) a straddle whether or not she owns the stock. The benefit in doing so when you own the stock is that the call being written is covered by the long stock, so you don't have the unlimited potential for loss. The question's solution relies on using the T-chart. "An investor buys 100 shares of QRS stock at 60 and writes 1 QRS 60 call at 4 and 1 QRS 60 put at 5. If QRS stock is trading at $74 on the expiration date, the investor realizes a profit of" Step 1: $6,000 goes out for the purchase of the stock. Step 2: $400 comes in from the sale of the call and $500 comes in from the sale of the put. Step 3: We always assume that out-of-the money short option positions are closed out (bought back) at their intrinsic value on expiration date, and in-the-money options are exercised. That means, with the market value of $74, the 60 call will be exercised with the investor selling the stock for $6,000 (in on the T-chart). No one is going to put us the stock at 60 when it is selling at 74, so the put expires, resulting in zero going out to close it. Step 4: The realized profit means only from gains or losses realized. Because the call option is exercised, we know the stock was sold for the 60 strike price Step 5: Put these all together. We started with $6,000 out for the stock purchase and $900 in for the sale of the two options. The call is exercised, so $6,000 comes in and the put expires, so $0 goes out. The result? $6,000 out and $6,900 in, for a profit of $900.

A 7% convertible debenture is selling at 101, and it is convertible into the common stock of the same corporation at $25. The common stock is currently trading at $23. What is the parity price of the debenture?

$920 To determine the parity price of the bond, first find the number of shares the debenture is convertible into (conversion ratio) by dividing par value by the conversion price ($1,000 / $25 = 40 shares). Next, multiply the current price of the common by the conversion ratio. The result is the parity price of the bond (40 shares × $23 = $920).

New offering: 800,000 units at $6 per unit. Each unit has two shares of common stock and one warrant. Each warrant is to purchase half a share of common stock. Based on this information, how many shares of stock will be sold, and how many warrants will be sold?

1.6 million shares and 800,000 warrants Warrants may be distributed to stockholders in an underwriting as part of a unit. The warrant is a form of bonus to entice investors to purchase the unit. As each unit contains two shares, 1.6 million shares are being distributed. As each unit also includes one warrant, 800,000 warrants are being distributed.

FINRA Rule 2310 defines a direct participation program as "a program which provides for flow-through tax consequences regardless of the structure of the legal entity or vehicle for distribution including, but not limited to, oil and gas programs, real estate programs, agricultural programs, cattle programs, condominium securities, Subchapter S corporate offerings and all other programs of a similar nature, regardless of the industry represented by the program, or any combination thereof." The rule places limits on the amount of broker-dealer sales compensation considered fair and reasonable. That limit is

10% of the gross proceeds.

For an investor in the 37% federal income tax bracket, the tax-equivalent yield of a general obligation municipal bond with a coupon rate of 4.17% is

6.62%. The computation for tax-equivalent yield is done by dividing the coupon rate by (100% - tax bracket). In this question, that is 4.17% ÷ (100% - 37%) = 4.17% ÷ 63%. That equals 6.62%.

ABC Corporation offers statutory voting. At the upcoming annual meeting, it is announced that six people are running for the four available open seats on the board. An investor with 100 shares would be able to cast

100 votes for each of four individuals. With statutory voting, the shareholder has one vote for each share and can use those votes for each open seat on the board. It is cumulative voting where the shareholder can lump votes to place on a single seat. Any of the other choices could be correct for cumulative voting.

An affiliate holding restricted stock wishes to sell shares under Rule 144. He has held the shares, fully paid, for six months, and the issuer has 2.4 million outstanding shares. Form 144 is filed on Monday, April 10, and the average weekly trading volume for the past four weeks is 24,500 shares per week. The maximum number of shares the customer can sell with this filing is

24,500 Under Rule 144, after holding the fully paid restricted shares for six months, the affiliate can begin selling. For affiliates, volume restrictions always apply. They can sell the greater of 1% of the total shares outstanding or the weekly average of the past four weeks' trading volume (the four weeks preceding the Form 144 filing). In this case, 1% of the total shares outstanding is 24,000 (1% × 2.4 million). The weekly average of the past four weeks' trading volume is 24,500. Therefore, the most the affiliate can sell during the 90 days following the Form 144 filing is 24,500 shares.

ALFA Enterprises pays a quarterly dividend of $0.15 and has earnings per share of $2.40. What is the dividend payout ratio?

25.00% Earnings per share are typically calculated for a year, so the annual dividend of $0.60 ($0.15 × 4) is divided by $2.40 to calculate what percentage of earnings is paid as a dividend—or rather, the dividend payout ratio (0.60 / 2.40 = 25%).

Achieving a Better Life Experience (ABLE) accounts are tax-advantaged savings accounts for individuals with disabilities and their families. The ABLE Act limits eligibility to individuals with significant disabilities where the age of onset of the disability occurred before turning age

26 One need not be under the age of 26 to be eligible to establish an ABLE account. One could be over the age of 26, but as long as the onset of the disability occurred before age 26, the person is eligible to establish an ABLE account.

The minimum maintenance requirement on short stock selling above $5 is

30% of the market value or $5 per share, whichever is greater. The minimum maintenance in a short account is 30% of the market value or $5 per share (whichever is greater) for stocks trading above $5. For stocks trading below $5, the minimum maintenance is $2.50 per share or 100% of market value (whichever is greater).

Member firms must keep records on customer complaints for

4 years. Member firms must keep the records on customer complaints for at least 4 years.

Nickelplate Manufacturing Corporation (NMC) is capitalized with 1 million shares of a 6% $50 par callable preferred stock and 10 million shares of $1 par common stock. With the preferred stock currently selling at $75 per share and the common stock at $60 per share, the current yield of the preferred stock is closest to

4%. Current yield on any security, stock or bond, is the annual income (dividend on stock, interest on bond) divided by the current market price per share (or per bond). The math in this question is the dividend of $3 (a 6% $50 par preferred stock is paying an annual dividend of 6% of $50, or $3 per share) divided by the current market price of the preferred stock ($75). The quotient is .04 or 4%. What about the common stock? All of that information is just to distract you. We cannot compute the current yield of the common stock because we do not have any information about its dividend.

A customer purchases a 6% municipal bond in the secondary market on a 7% basis. The effective after-tax yield is

6 to 7% Because the interest on a municipal bond is tax-free on the federal level, the effective after-tax yield is generally the same as the coupon. That is, a bond paying 6% interest, with none of it taxable, will have an after-tax yield of 6%. This question deals with an exception. This bond is purchased in the secondary market on a 7% basis or yield to maturity (YTM). Whenever the YTM is higher than the coupon, the bond is priced at a discount from par. That means the investor is going to receive the difference between the discounted purchase price and the par value when the bond matures. When the bond is purchased in the secondary markets, that difference is amortized over the remaining life of the bond and is taxed each year as ordinary (taxable) income. Therefore with the annual amortization of the discount (that annual profit per se) being taxable, the effective after-tax return will be higher than the 6% coupon, but not equal to the 7% yield to maturity.

Your client is considering two bonds: an ABC Corporation mortgage bond with a coupon yield of 9% and a municipal bond issued by the state that she resides in. If your client is in the 32% tax bracket, what is the tax-free equivalent yield for the corporate mortgage bond so that she will be able to compare it to the tax-free municipal bond she is considering?

6.12% When the yield on the corporate taxable bond is given and the question asks for the equivalent after-tax yield of a tax-free municipal bond, we calculate that as follows: corporate rate × (1 − investor's tax bracket). In this case, 0.09 × (1 − 0.32) = 0.09 × 0.68 = 0.0612, or 6.12%. Without all of those zeros, it is 9% × 68% = 6.12%. Alternatively, just subtract the 32% tax from 9%. That would be 9% − (9% × 32%) = 9% − 2.88% tax = 6.12%. Remember, because the investor is paying taxes at a rate of 32%, she is keeping only 68% of the income received on a taxable security compared to 100% on a tax-free municipal bond. When the coupon rate of the tax-free security is given and the question is asking about the tax-equivalent yield of that security (it will always be higher than the coupon), we need to divide. Divide the municipal coupon rate by (100% minus the tax bracket). For example, if this question has said that the coupon on the municipal bond was 6.12% and asked for the TEY, you would divide 6.12% by 68%. Do that on your calculator and the result is 9%. The key to remembering which formula works is that when the coupon on the taxable security is given, the answer will always be lower than that coupon. That means subtracting. When the coupon on the tax-free security is given, the answer will always be higher than that coupon and that means dividing.

A stock pays a $0.50 quarterly dividend. The company had earnings per share last year of $10. The company's dividend retention ratio is

80% The dividend retention ratio is the reciprocal of the dividend payout ratio. If the company pays a $2 dividend on earnings of $10, it pays out 20% of the earnings available to common shareholders in the form of the dividend. That means it retains 80% of the available monies.

A stock pays a $0.50 quarterly dividend. The company had earnings per share last year of $10. The company's dividend retention ratio is

80%. The dividend retention ratio is the reciprocal of the dividend payout ratio. If the company pays a $2 dividend on earnings of $10, it pays out 20% of the earnings available to common shareholders in the form of the dividend. That means it retains 80% of the available monies.

A municipal bond originally issued at 90 with a 10-year maturity will have a compound accreted value (CAV) after five years equal to

95. CAV is the cost basis of the bond, in this case, after five years accretion. There are 10 points to accrete (the difference between the issue price of 90 and par) over 10 years. One point each year will be added, so after five years, the adjusted cost basis will be 90 + 5, or 95.

Which of the following statements regarding yield shown on a bond confirmation for a bond that has been called is true?

A bond confirmation will show YTC if the bond has been called under an in-whole call provision. A bond confirmation for a bond called under an in-whole call provision will show yield to call (YTC), as the bond being called away is certain. However, in the event of an in-part call, there is uncertainty as to whether that particular bond will be called. Therefore, the lower of the YTC or yield to maturity would be shown on the confirmation.

One of your customers is concerned about future inflation. Which of the following mutual fund recommendations is likely to be the most suitable? A) A municipal bond fund B) A common stock growth fund C) A balanced fund D) A U.S. Treasury bond fund

A common stock growth fund On this exam, the investment most likely to provide inflation protection is common stock. A balanced fund, one that contains almost equal proportions of common stock and fixed income, is generally thought of as a conservative investment providing both safety through the fixed income and potential growth through the equity. That safety comes at the expense of slower growth. Municipal bonds are only suitable (on the exam) when the question deals with an investor in a high tax bracket. Likewise, Treasury bonds are for those looking for lower income, but with highest safety. Be careful, though. The mutual fund is not guaranteed by the Treasury; only the bonds are.

Congress passed the Trust Indenture Act of 1939 to protect bondholders. It requires that issuers of eligible bonds appoint a trustee to ensure that promises (covenants) between the issuer and the trustee who acts solely for the benefit of the bondholders are carried out. The Trust Indenture Act of 1939 applies to which of the following issues offered on an interstate basis?

A corporate bond in the amount of $200 million with a maturity of 20 years. There are four basic elements qualifying a bond issue for coverage under the Trust Indenture Act of 1939. Those are as follows: It is a corporate issue. The issue size is more than $50 million within 12 months. The maturity is 9 months or more. It is offered interstate. A $200 million corporate bond issue with a 20-year maturity would be covered under the act, and the other choices would not.

If at expiration for XYZ options, XYZ stock closes at 40.15, which of the following open option positions will automatically be exercised by the Options Clearing Corporation (OCC)? A) A customer long 1 XYZ 40 put B) A member firm long 1 XYZ 40 put C) A member firm long 1 XYZ 45 call D) A customer long 1 XYZ 40 call

A customer long 1 XYZ 40 call At expiration, the OCC will automatically exercise open option positions if those positions are in the money by 0.01 or more. In this case, the customer's long 40 call position is in the money by 0.15. The member firm 45 call and the 40 puts are out of the money.

What type of mutual fund has the stated objective to invest in securities of U.S.-domiciled entities, as well as those domiciled in foreign countries?

A global fund A global fund invests in both U.S. and international stocks and/or bonds. An international fund only invests in non-U.S. securities. Think of it this way. If you are traveling around the globe, you will spend part of your time in the United States. If you are traveling internationally, your travel will not include the U.S. There are no such formal terms as worldwide and all-Earth funds.

Which of the following capital structures would be considered the most highly leveraged?

A large value of bonds and a small value of common stock Leverage is using other people's money to enhance equity value. In this case, borrowing at a fixed rate of payment enhances cash flow, giving the company extra money to invest in its operations. Just as individuals, a company has to be careful not to borrow more than it can afford.

Investing into mutual fund Class A shares will result in a customer paying a sales charge that is imposed at the time of purchase. That is why these shares are called front-end load shares. With Class A shares, the investor might be entitled to breakpoints resulting in the customer paying a reduced sales charge based on a breakpoint schedule found in the prospectus. Purchases by which of the following can be combined to reach breakpoints? A) An investment club B) A parent and dependent adult child C) A parent and adult child D) A married couple

A married couple The accounts of married couples are eligible to be combined to reach breakpoint levels. Also eligible are parents with minor children. Parents with adult children are not eligible, even if the child is a dependent, and investment clubs are specifically mentioned in the law as not eligible.

Which of the following securities is not exempt from the registration provisions of the Securities Act of 1933? A) A new stock being offered in three states B) An equity security issued in only one state, solely to residents of that state C) A high-quality corporate promissory note maturing in 180 days D) A U.S. government bond

A new stock being offered in three states Government securities, money market instruments, and intrastate offerings are exempt from the registration provisions of the Securities Act of 1933. A stock being offered in three states would have to register with the SEC and with those states.

Which of the following individuals may not open a joint account?

A parent and a minor A minor may not be a party in a joint account because a minor cannot legally exercise control over the account. A custodial account may be set up for the minor.

Which of the following business structures will generally have the fewest number of owners? A) An LLC B) A general partnership C) A sole proprietorship D) An S corporation

A sole proprietorship A sole proprietorship has only one owner. That is what the "sole" means. Although S corporations and LLCs can be formed with a single shareholder or member, that would be the exception rather than the rule. A partnership needs at least two persons.

Which of the following will not affect special memorandum account (SMA)? A) A deposit of cash into the account by the customer B) A stock dividend on stock held long in the account C) A long sale at a profit D) A cash dividend on stock held long in the account

A stock dividend on stock held long in the account The value of the stock dividend received will be offset by the decline in current market value (CMV) of the long position on which the dividend is paid. The account's long CMV will not change; it will just be represented by more shares. There will be no change in the overall CMV, debit balance, or equity, and therefore no change in the SMA. On the other hand, a cash dividend means that new money is coming into the account, which will reduce the debit balance and be credited to SMA so the customer may withdraw the dividend, if desired.

Which of the following is not part of the Federal Farm Credit System? A) The Federal Home Loan Banks B) The Federal Intermediate Credit Bank C) The Bank for Cooperatives D) The Federal Land Banks

A) The Federal Home Loan Banks The Federal Farm Credit Bank system is for farms, not homes.

Which of the following statements best describes the effect of reinvesting mutual fund distributions? A) The reinvestment of capital gains and dividends results in a higher cost basis. B) The reinvestment of capital gains and dividends has no effect on cost basis. C) Dividends from investment income that are reinvested have tax-deferred status. D) Capital gains that are reinvested have tax-deferred status.

A) The reinvestment of capital gains and dividends results in a higher cost basis. Because reinvested distributions are taxed in the year received, the investor's cost basis is increased by the amount of the distribution. This is to prevent those distributions from being taxed twice. They are taxed once as the dividend or capital gain and then, if not added to the cost basis, would be taxed a second time when the shares are sold. Reinvestment of these distributions does not avoid or defer current taxation. This is not the same as receiving a stock dividend where the taxes are deferred until those shares are sold.

A customer with no other mutual fund investments wishes to invest $47,000 in the XYZ Technology Fund. If the Class A shares are eligible for a breakpoint sales charge discount at the $50,000 investment level, the least appropriate action for an agent is to A) place the order as instructed. B) inform the customer that she can reduce her sales charge through a letter of intent. C) inform the customer that she can reduce her sales charge by investing an additional $3,000. D) inform the customer that she can reduce her sales charge by combining purchases in other funds offered by XYZ group.

A) place the order as instructed. If a customer intends to invest an amount just below a breakpoint threshold, she should be informed of the breakpoint discount, as well as the various methods by which she can receive it.

All of the following are suitable objectives for a covered call writer except A) profiting from an increase in the price of stock. B) providing downside protection for a long stock position. C) speculating that a stock will not rise in price. D) increasing return on a long stock position.

A) profiting from an increase in the price of stock. Covered call writers are not able to benefit from an increase in the price of the underlying stock. For example, you buy stock at $40 and write a 40 call. Now, the stock is 80. Isn't that great? Your long stock position has doubled. Not so fast. With the stock at 80, it is certain that the 40 call will be exercised. So no matter how high the stock goes, the covered writer can't benefit because the call will be exercised and the stock will be sold at the $40 strike price. Why sell covered calls? This strategy provides downside protection to the extent of the premium received, and it increases the rate of return on a long stock position (because of the premium collected).

All of the following statements regarding negotiable jumbo certificates of deposit are true except A) they are fully insured in any denomination by the FDIC. B) they are readily marketable. C) they usually have maturities of less than one year. D) they are usually issued in denominations of $100,000 to $1,000,000.

A) they are fully insured in any denomination by the FDIC. The FDIC insures only up to $250,000.

Which of the following debt instruments is unsecured?

AAA/AAA-rated debentures Corporate debentures are unsecured bonds backed by the credit of the issuing corporation; they are not secured by underlying collateral. Mortgage bonds are secured with real estate serving as collateral. Collateral trust bonds are secured by securities that a corporation owns in other companies or bonds. Equipment trust certificates are secured by transportation equipment owned by the corporation.

You are reviewing a company's financial statements to assist a customer. What kind of information is most likely to be found in the footnotes to those financial statements?

Accounting methods used Footnotes to a company's financial statements will contain important financial information such as accounting methods used, as well as important management philosophy that may impact the company's overall financial health and performance.

Which of the following is not a source of revenue for a municipal revenue bond issue? A) Tolls B) Ad valorem taxes C) Fees D) Assessments

Ad valorem taxes Fund generators, such as tolls, assessments, and fees, subsidize revenue bonds. Ad valorem taxes support general obligation bonds.

A preliminary prospectus would be used during what time frame?

After the registration statement has been filed, but before it has become effective The preliminary prospectus (red herring) is made available during the cooling-off period—that is, after the registration statement has been filed but before the effective date. It is used to gauge investor interest.

Which of the following would be most likely to issue an equipment trust certificate?

An airline company When you see "equipment trust certificate," think transportation companies such as airlines and railroads.

A technical analyst would find which of the following to be a bullish indicator?

An increase in the short interest As the short interest increases, it is a bullish signal to technicians. Those short sales are going to have to be covered one day and that buying pressure is bullish on the stock. A head and shoulders top is bearish - it signifies that the market has reached a top. Odd-lot purchasers (the little guy) are always buying and selling at the wrong time. When they are buying more than selling, it is a sign that the market price of the stock is soon going to fall. When the advances outnumber the decliners, it is bullish. When it is reversed, as in the answer choice, it is bearish.

Wanting to be clear about an upcoming mutual fund dividend distribution, a shareholder does some research on his own. He comes away with what he thinks are all correct understandings, but one of them is not. Which understanding is not correct? A) An investor can receive dividends in the form of cash or choose to reinvest them in more fund shares. B) An investor is not liable for taxes if he automatically reinvests the distributions. C) The dividends are paid from the fund's net investment income. D) Dividend distributions are not guaranteed to occur annually, quarterly, or at any time.

An investor is not liable for taxes if he automatically reinvests the distributions. Mutual funds pay dividends from net investment income, and shareholders are liable for taxes on all distributions, whether reinvested or taken in cash. Dividend distributions from mutual funds like individual corporate dividend distributions are not guaranteed.

Which of the following statements about the underwriting manager of a syndicate is true? A) The other syndicate members elect the manager. B) An underwriting syndicate may have more than one manager. C) Syndicate managers are exempt from FINRA membership. D) There is a unitary syndicate manager.

An underwriting syndicate may have more than one manager. An underwriting syndicate may have more than one manager. This is typically the case on very large issues. Regardless of the number, all participants in the underwriting must be FINRA members.

A registered representative (RR) has just explained to a customer that to purchase a particular security, the customer would pay the asking price plus a commission, not a sales charge. Which of the following is the RR speaking of? A) Mutual funds B) Any closed-end fund C) All open-end funds D) All management company offerings

Any closed-end fund Closed-end funds are purchased on an exchange or over the counter where buyers pay the asking price plus a commission. The RR could not be speaking of all open-end funds because mutual funds—one classification of open-end funds—are purchased at the public offering price, which includes a sales charge. Management company offerings include both open-end and closed-end funds.

Which of the following is not generally associated with an existing real estate direct participation program?

Appreciation potential. Appreciation potential is generally not associated with existing real estate programs because most appreciation occurs in the earliest years for real estate assets.

Your manager notifies you that a new municipal revenue bond issue you have been working on has been oversubscribed. How is the order acceptance priority for this issue determined?

As outlined in the agreement among underwriters The priority of filling municipal orders is established by the managing underwriter in the release terms letter sent to the syndicate once the bid is won. This letter is an amendment to the agreement among underwriters. The priority is also disclosed in the official statement (full and fair disclosure document for municipal new issues).

Call-buying strategies include all of the following except A) acquisition of a stock position. B) covering a long stock position. C) protection of a profit on a short sale of stock. D) an increase in leverage with limited risk.

B) covering a long stock position. Long stock and long calls have the same market attitude: bullish. Therefore, covering a long stock position cannot be accomplished by buying calls. The remaining choices are all strategies that could warrant call buying: increased leverage, acquisition of stock, and hedging a short stock position.

One of the terms used in the discussion of SEC Rule 144 is affiliate. Which of the following would be defined as an affiliate? A) The sister of the CEO living in a neighboring town B) The brother-in-law of the spouse of the issuer's CEO, who lives in the same home as the CEO C) The CEO's contract bridge partner D) The CEO's mother-in-law, whose only source of income is her Social Security and a small pension

B) The brother-in-law of the spouse of the issuer's CEO, who lives in the same home as the CEO For purposes of Rule 144, an affiliate is a person that directly, or indirectly through one or more intermediaries, controls, or is controlled by, or is under common control with, the issuer. In addition, any relative or spouse of such person, or any relative of such spouse, any one of whom has the same home as such person is considered an affiliate of the issuer. For test purposes, unless stated otherwise, spouses always live in the same home.

If ABC Corporation reports a loss for the year, it is obligated to pay interest on all of the following except A) convertible bonds. B) adjustment bonds. C) variable-rate bonds. D) nonconvertible bonds.

B) adjustment bonds. Even if a corporation reports a loss, the corporation is obligated to pay interest on all of its outstanding debt except for income (adjustment) bonds. Income—or adjustment bonds—require interest to be paid only if declared by the board of directors.

Who attests to the legality of a bond issue and issues a legal opinion on a proposed new municipal bond issue?

Bond counsel The issuer hires a firm or an individual to act on its behalf as bond counsel.

A customer sells short 1,000 ZOO at $30 per share. If the ZOO stock declines to $25 per share, and the customer is worried the stock may reverse its trend, what should the customer do?

Buy 10 ZOO calls To protect the profit on the short stock position, the customer must be able to buy stock at the existing low price if the market moves up. By purchasing calls (say, at a $25 strike price), the customer can capture existing profit by exercising and buying stock at $25, regardless of how high the market moves.

Which of the following oral orders can be accepted from a customer without additional documentation? A) Buy 100 shares of ABC when the price is right B) Buy 200 shares of computer stock C) Buy $20,000 of quality bank stocks D) Increase my position in ABC

Buy 100 shares of ABC when the price is right Prices and time of execution do not require discretionary authority.

If a customer believes the market price of a stock will sharply rise or fall in the near future, which of the following is the best strategy?

Buy a straddle If the stock goes either up or down sharply, the investor will profit from owning a straddle.

An investor wants to profit from a speculative, near-future market advance, but he is uncertain which stocks will be affected. To limit his risk to a specific amount, which of the following actions would best meet his objectives?

Buy calls on a broad-based stock index Broad-based index options allow investors to profit on their beliefs about movement in the overall market. Index calls profit the investor if the index closes above the strike price plus the premium paid. The investor has limited risk because only the option's premium can be lost if the market moves downward. Narrow-based index options involve a single market sector only (e.g., technology stocks).

When discussing mutual funds with a customer, which of the following statements is not prohibited?

Buy shares of different funds in the same fund family, and you may qualify for a breakpoint on the total purchase. Most funds provide a combination privilege, allowing investors to aggregate purchases made in different funds in the same family to qualify for a breakpoint. The income yield of a mutual fund includes dividends only. A group of friends is not eligible for a breakpoint. (Investment clubs are not eligible.) Selling dividends is a prohibited practice because of the immediate tax liability incurred with the dividend and the share price adjustment that results after the dividend distribution.

A Japanese manufacturer sells recorders to a U.S. retailing firm. The manufacturer is to receive USD$1 million in 90 days. How can he best protect himself against a decline in the dollar?

Buy yen calls Because he is receiving U.S. dollars, his risk is that the U.S. dollar will go down in value against the Japanese yen. If the dollar goes down against the yen, the yen will rise. Therefore, to protect his risk against a rising yen, he should buy yen calls. The yen calls will increase in value if the yen rises.

One of your clients has $30,000 in the Balfour Balanced Fund in her personal account. She is preparing to invest an additional $12,000. Balfour has a breakpoint at $50,000. The client mentions that she has other holdings in Balfour and wonders if they will count toward the breakpoint. You respond that all of the following accounts would qualify except A) her IRA. B) the UTMA account where she is custodian for her child. C) her mother's account. D) her holdings in her 401(k) plan at work.

C) her mother's account. In most cases (all on the exam), a mutual fund will allow investors to get a breakpoint discount by combining their fund purchases with those of their spouse and minor children. They are also able to credit mutual fund transactions in retirement accounts, educational savings accounts, or even in accounts at other brokerage firms.

All the following would be considered current assets except A) inventory. B) cash. C) marketable securities. D) a warehouse.

Current assets are those that are either cash or expected to generate cash within the next year. Warehouses are fixed assets used for many years.

Mutual funds have several different methods for assessing charges to become shareholders. One of those is the front-end load and is charged when purchasing

Class A shares. Purchasers of Class A shares pay a sales charge on each investment. Because the sales charge comes off the amount invested, it is called a front-end load. Class B and C shares have back-end loads, although Cs usually drop off after one year. There are shares for almost the entire alphabet, but we are not hearing anything about them−only the A, B, and C.

The interest on which of the following instruments is subject to taxation at the federal, state, and local levels? A) Public Housing Authority bonds B) Collateralized mortgage obligations (CMOs) C) Treasury notes D) Revenue bonds

Collateralized mortgage obligations (CMOs) The interest on corporate debt securities is subject to taxation at the federal, state, and local levels, and CMOs are issued by corporations. Interest on Treasury instruments is taxable at the federal level only. Interest on municipal instruments is exempt from taxation at the federal level, and possibly the state level, if the holder is a resident of the state of issue. Public Housing Authority bonds and revenue bonds are types of municipal issues, which are tax free at the federal level.

Which of the following is not true in jurisdictions that recognize the marital property designation known as community property? A) There may be tax implications regarding the dissolution of community property at the time of a divorce, marriage annulment, or death. B) Community property applies to property that was owned individually before the marriage and is now joint property once the marriage has occurred. C) Community property laws do not apply to gifts. D) Community property laws do not apply to inheritances.

Community property applies to property that was owned individually before the marriage and is now joint property once the marriage has occurred. Community property applies to property obtained during a marriage but does not apply to property owned individually by one spouse before the marriage. In addition, it does not apply to inheritances or gifts. There can be federal tax implications for property designated as community property, and laws in states that recognize community property ownership differ from jurisdiction to jurisdiction.

Which of the following statements regarding both traditional and Roth IRAs is true? A) Contributions are tax deductible. B) Contribution limits are the same. C) Distributions must begin in the year after the owner reaches age 72. D) Withdrawals at retirement are tax free.

Contribution limits are the same. The common factor for both traditional and Roth IRAs is that contribution limits are identical. A significant difference between the two is that Roth IRAs do not have RMDs.

Corporations issue equity securities. One category of equity is preferred stock. A number of different adjectives can apply to preferred stock issues. Which of the following preferred stock issues would likely offer the greatest protection against interest rate risk? A) Participating B) Callable C) Cumulative D) Convertible

Convertible Owners of convertible preferred stock have the ability to convert the preferred into common stock. This offers growth potential not otherwise available to a preferred stockholder. This feature causes the stock's price to track that of the common. That results in convertible preferred stock having less interest rate risk than the others do.

Your customer, age 29, makes $42,000 annually and has $10,000 to invest. Although he has never invested before, he wants to invest in something exciting. Which of the following should you suggest?

Customer should provide more information before you can make a suitable recommendation It is necessary to get more information about this customer and his definitions of an exciting investment opportunity before making any recommendations. A suitability and risk-tolerance analysis should be performed before a recommendation is made.

All of the following debt instruments pay interest semiannually except: A) Ginnie Mae pass-through certificates. B) municipal general obligation bonds. C) municipal revenue bonds. D) industrial development bonds.

Ginnie Mae pass-through certificates. Ginnie Maes pay interest on a monthly basis, not semiannually.

Two registered representatives are discussing a collateralized debt obligation (CDO) backed by cash flow from credit card payments. Which of their statements during the discussion is not true? A) A customer would have to choose a tranche that has the right risk characteristics for him, in terms of suitability. B) CDOs are securitized products where pooling or repackaging of individual loans has occurred. C) CDOs are not considered suitable for all customers. D) CDOs always represent pools of assets that individually are very liquid, and that is why the CDOs themselves are very liquid.

D) CDOs always represent pools of assets that individually are very liquid, and that is why the CDOs themselves are very liquid. In most cases, the assets comprising the CDO portfolio are small and individually not very liquid. Generally, individual investors would not have an opportunity to purchase these assets separately. Repackaging the assets, however, facilitates them being sold to individual investors in the secondary markets.

ABC Corporation's earnings have increased by 10%, and its shares outstanding have increased 5%. This has what impact on ABC's EPS?

EPS has increased. If earnings (net income) have increased faster than the shares outstanding, EPS will increase. For example, if ABC's earnings were $10 million and it had 1 million shares, the earnings per share were $10. Now, with a 10% increase to earnings, those earnings are $1.1 million. If the shares have increased by 5%, there are now 1,050,000 shares. That makes the new EPS $11,000,000 ÷ 1,050,000 shares which equals approximately $10.48 per share. This proves the correct choice that EPS has increased.

All of the following information is included in a municipal bond resolution except: A) any call provisions that allow the issuer to redeem the bonds before their scheduled maturity. B) restrictive covenants that are binding on the issuer. C) an authorization to sell the securities. D) compensation paid to the underwriters.

D) compensation paid to the underwriters. The bond resolution is the document in which the issuer authorizes the issuance of municipal securities. Among other things, the resolution describes the characteristics of the proposed issue and the issuer's duties to the bondholders. Compensation paid to the underwriters would be found in the official statement.

The over-the-counter (OTC) market could be characterized as what type of market?

Dealer The OTC market is a dealer market.

A margin account is restricted by $5,000. Which of the following actions may the customer take to bring the account to the Regulation T requirement?

Deposit $10,000 of fully paid marginable stock. As we teach in the course, KISS (Keep It Series 7 Simple). Saying the account is restricted is just to make it seem more difficult. The question asks how to get the account off restriction? The customer needs $5,000 in cash to do so. However, there is no answer of $5,000. But there is another way to get money—deposit fully paid marginable securities. How much? The deposit should be 200% or twice the amount of cash needed. That would be $5,000 × 2, or $10,000.

Which of the following is true regarding a 5-for-4 stock split? A) The par value will be unchanged. B) Retained earnings will be increased. C) The net worth of the company will be reduced. D) Each shareholder's proportionate equity will be unchanged.

Each shareholder's proportionate equity will be unchanged. Because each shareholder will receive additional stock, the proportional equity will remain the same.

Which of the following statements regarding savings incentive match plans for employees (SIMPLEs) is not true? A) Employee contributions are pretax. B) Employers cannot make matching contributions for employees. C) SIMPLEs are retirement plans for small businesses with100 or fewer employees. D) Catch-up contributions for those age 50 and older are permitted.

Employers cannot make matching contributions for employees. SIMPLEs are retirement plans for businesses with no more than 100 employees that have no other retirement plan in place. The employee makes pretax contributions into a SIMPLE up to an annual contribution limit, which can include catch-up contributions for those age 50 and older. The employer is permitted to make matching contributions for employees.

Which of the following options transactions settles T+2? A) Exercise of long index option B) Closing sale of a long equity put option C) Opening purchase of a long equity put option D) Exercise of a long equity put option

Exercise of a long equity put option When a long equity put option is exercised, from a settlement standpoint, it is the same as anyone selling stock. That is T+2 settlement date. The trading of options, not the exercise, is T+1. When it comes to index options, because they settle in cash, exercise settlement is T+1 rather than T+2 like equity options.

Which of the following types of oil and gas limited partnership programs is the most risky?

Exploratory For oil and gas partnerships, exploratory programs are considered the riskiest because they involve drilling new wells in areas where oil has not yet been discovered. These programs would be followed by developmental, with income programs being the least risky. Existing property is a type of real estate program.

All of the following statements regarding the Federal National Mortgage Association (FNMA) are true except A) FNMA is owned by the U.S. government. B) FNMA pass-through certificates are not guaranteed by the U.S. government. C) interest on FNMA certificates is taxable at all levels. D) FNMA is a publicly held corporation.

FNMA is owned by the U.S. government. FNMA is a publicly held corporation. The interest income on all mortgage-backed securities is fully taxable. Though a government agency, FNMA pass-through certificates are not guaranteed by the U.S. government. The only U.S. agency whose securities are considered direct obligations of the U.S. government is the Government National Mortgage Association.

One of your customers would like to purchase a government agency security for the UTMA account of her daughter. The daughter worked in construction over the summer and would like to use $1,275 of her savings for the purchase. Securities issued by which of these agencies could be purchased for this account? A) Student Loan Marketing Association B) Federal Farm Credit System C) Federal National Mortgage Association D) Federal Home Loan Mortgage Corporation

Federal National Mortgage Association Of this group, the only agency that would be able to sell $1,275 of securities is Fannie Mae. Their securities are available with a minimum denomination of $1,000 and then increments of $1. FHLMC also has the $1,000 initial minimum, but with $1,000 increments. The same numbers apply to the FCS, and Sallie Mae's minimum is $10,000. Another agency that would have met the investor's need is GNMA.

Which of the following organizations determines which over-the-counter securities are eligible for purchase on margin?

Federal Reserve Board. The Federal Reserve Board determines whether any security is marginable.

Which of the following usually does not pay interest semiannually? A) Public utility bonds B) Treasury bonds C) Government National Mortgage Association (GNMA) D) Treasury notes

GNMA pass-through certificates pay principal and interest monthly. The others usually pay interest semiannually.

All of the following statements regarding Government National Mortgage Association (GNMA) pass-through securities are true except A) the minimum initial investment is $1,000. B) investors receive a monthly check representing both interest and a return of principal. C) investors own an undivided interest in a pool of mortgages. D) GNMAs are considered to be the riskiest of the agency issues.

GNMAs are considered to be the riskiest of the agency issues. GNMA securities, which are backed by the full faith and credit of the U.S. government, are considered to be the safest of the agency issues. As is the case with most agencies, the minimum denomination is $1,000.

An investor looking for income with the highest degree of safety would probably choose to purchase A) FNMAs. B) FHLMCs. C) GNMAs. D) SLMAs.

GNMAs. All of the choices are U.S. government agencies, but only those issued by the Government National Mortgage Association (GNMA) have the direct backing of the government.

There is a type of municipal revenue bond known as a special tax bond. Which of the following might be used as backing for a special tax bond?

Gasoline tax Common taxes backing special taxes include fuel (gasoline) taxes, sales tax, business license taxes, and tobacco taxes. Ad valorem are property taxes, and those are used to back GO bonds. Income taxes are generally used to back state-issued GO bonds.

Which of the following agencies approves private lending institutions to issue bonds that are backed by the full faith and credit of the U.S. government?

Government National Mortgage Association (GNMA) GNMA bonds are issued by private lending institutions approved by GNMA and are backed by the full faith and credit of the U.S. government. They are considered to be default risk free.

Which of the following real estate limited partnerships allows tax credits to the investor?

Historic rehabilitation Raw land partnerships seek appreciation. Existing property and new-construction partnerships seek passive income and tax deductions from business operations. Historic rehabilitation partnerships allow not just deductions, but actual tax credits.

A high-net-worth investor with substantial annual income likes real estate as a potential investment. The investor notes that any investment potentially offering tax credits would be most interesting to consider first. Which of the following would be suitable investments to discuss?

Historic rehabilitation and government-assisted housing direct participation programs (DPPs) REITs, either equity or mortgage, should be eliminated, as they offer no tax credits. Given the customer's high net worth and income, a discussion of DPPs is suitable. Of those DPPs shown here, only historic rehabilitation and government-assisted housing offer tax credits, and either should be suitable for discussion.

Traders can sell short I. when a stock ticks up. II. when a stock ticks down. III. unrestrictedly in both exchange and over-the-counter (OTC) markets.

I, II, and III In both exchange and OTC markets, traders can sell short at any time in the trade sequence.

If LMN, Inc., has filed for bankruptcy, in what order would interested parties be paid? I. Holders of secured debt II. Holders of subordinated debentures III. General creditors IV. Preferred stockholders

I, III, II, IV. holders of secured debt, general creditors, holders on subordinated debt, preferred stockholders The liquidation order is as follows: the IRS (and other government agencies), secured debt holders, unsecured debt holders and general creditors, holders of subordinated debt, preferred stockholders, and common stockholders.

Which of the following quotations are expressions of firm quotes? I. "It is 30-31." II. "Last I saw was 30-31." III. "We can trade it 30-31." IV. "30-31 work out."

I. "It is 30-31." AND III. "We can trade it 30-31." Examples of firm quotes include: "It is 30-31," "We can trade it 30-31," "The market is 30-31," and "We will do it 30-31."

A municipal dollar bond is quoted at 98¼ to 98¾. The municipal dealer's spread is equal to I. $5.00 II. $50.00 III. 5 basis points IV. 50 basis points

I. $5.00 and IV. 50 basis points The spread is half a point. In each point, which is worth $10, there are 100 basis points. Therefore, half a point is worth $5 and represents 50 basis points.

Which of the following orders may be used to acquire a security at a specific price or better? I. A buy stop limit II. A buy limit III. A sell stop IV. A sell limit

I. A buy stop limit AND II. A buy limit Only buy orders can acquire stock. Only buy limit orders can acquire stock at a specific price or better.

Which of the following can be rolled over into an IRA? I. Another IRA II. Balances from savings accounts III. A corporate profit-sharing plan IV. Judgments from lawsuit settlements

I. Another IRA and III. A corporate profit-sharing plan Assets from any qualified corporate plan or from another IRA may be rolled over into an IRA.

Which of the following would not be examples of overlapping debt? I. Debt to build a state office building within city limits II. Debt to maintain a county park district serving a municipality III. Debt backed by two states cooperating in the construction of a bridge IV. Debt for a high school district within city limits

I. Debt to build a state office building within city limits and III. Debt backed by two states cooperating in the construction of a bridge State debt cannot overlap with any other municipal entity.

Which of the following statements regarding joint accounts registered as tenants in common (TIC) are true? I. Each party specifies a percentage of interest in the account. II. Each party has an equal interest in the account. III. The interest of a deceased tenant passes to the estate of the decedent. IV. The interest of a deceased tenant passes to the cotenant.

I. Each party specifies a percentage of interest in the account. AND III. The interest of a deceased tenant passes to the estate of the decedent. In a TIC account, each party must specify a percentage of interest in the account. If one party dies, his percentage of ownership passes to his estate, not to any other party to the account.

Which of the following statements regarding index options are true? I. Exercise is settled in cash. II. Exercise settlement value is based on the value of the index at the time exercise instructions are received. III. Exercise settlement value is based on the closing index value on the day exercise instructions are tendered. IV. Exercise settlement is T+2.

I. Exercise is settled in cash. and III. Exercise settlement value is based on the closing index value on the day exercise instructions are tendered. All index option exercises are settled in cash. The amount a writer owes the holder is known as the intrinsic value of the option, and the settlement value is based on the closing index value on the day exercise instructions are tendered. Exercise settlement is the next business day.

A firm may assign option exercises using which of the following methods? I. First-in, first-out (FIFO) II. Last-in, first-out (LIFO) III. Random assignment IV. Based on holders of the smallest positions

I. First-in, first-out (FIFO) AND III. Random assignment A firm may assign an exercise either randomly or using the FIFO accounting method. LIFO is not permitted, nor is assigning by position size, smallest, or largest.

A customer has entered an option order with your broker-dealer. At which of the following locations could such an order be executed? I. NYSE II. CBOE III. Nasdaq PHLX IV. None of these

I. NYSE, II. CBOE, and III. Nasdaq PHLX Options orders can be executed on the NYSE, the CBOE, and the Nasdaq PHLX, which offers a hybrid of electronic and on-floor execution availability.

Which of the following activities are disallowed under FINRA rules? I. Opening an account for a 16-year-old individual II. Accepting a sale in a joint account from one of the owners and having the check payable in the name of that individual III. Accepting a sale order from the husband only in a joint account owned by both husband and wife IV. Requiring written discretionary authorization before accepting orders for a discretionary account

I. Opening an account for a 16-year-old individual AND II. Accepting a sale in a joint account from one of the owners and having the check payable in the name of that individual The question is asking for the nonallowable practices. We cannot open an account for a minor, nor may we make a check payable to only one of the parties in a joint account.

If an investor keeps $100,000 invested in U.S. Treasury bills at all times during a 10-year period, she is subject to which of the following? I. Stable principal II. Unstable principal III. Stable interest IV. Unstable interest

I. Stable principal and IV. Unstable interest Treasury bills are purchased at a discount and mature at face value. This feature provides principal stability to investors who own them. The discount on bills is determined by current market interest rates and fluctuates accordingly.

Which of the following regarding T-bills are true? I. T-bills trade at a discount to par. II. T-bills have maturities of 1 to 10 years. III. Most T-bill issues are callable. IV. T-bills are a direct obligation of the U.S. government.

I. T-bills trade at a discount to par. and IV. T-bills are a direct obligation of the U.S. government. T-bills trade at a discount to par, are six months or less to maturity, and are a direct obligation of the U.S. government. T-bills are also noncallable.

If a client has a margin account with $23,000 in securities and a debit of $12,000, and Regulation T is 50%, which of the following statements are true? I. The account is restricted. II. The client will receive a margin call for $500. III. The client may withdraw securities if she deposits 50% of the market value of the securities withdrawn. IV. The account has excess equity of $5,250.

I. The account is restricted. and III. The client may withdraw securities if she deposits 50% of the market value of the securities withdrawn. The account is restricted by $500 because the equity of $11,000 is less than the Regulation T requirement of 50% ($11,500). However, the client will not receive a margin call for the $500 because Regulation T applies only to the initial purchase. Because the account is restricted, any withdrawal of securities requires a cash deposit of 50% or a deposit of securities with a loan value of 50% of the value of the securities withdrawn. The account is $5,250 above the required minimum maintenance margin, but this amount is not considered excess equity.

Which of the following applies to an open order to buy 600 XYZ at 44? I. The order may be partially filled at the limit price or better. II. The order is entered in the order book. III. This is considered a market order. IV. It must be executed at the price specified or better.

I. The order may be partially filled at the limit price or better. II. The order is entered in the order book. and IV. It must be executed at the price specified or better. Unless the order is specified as an all-or-none (AON) or a fill-or-kill (FOK), it may be partially filled. Finally, it is a limit order, which must be executed at the specified price or better.

Which of the following statements regarding a 2-for-1 stock split are true? I. The share price is reduced by half. II. The total market value of the outstanding stock decreases. III. The total market value of the outstanding stock may increase or decrease as a result of the split. IV. The number of shares doubles.

I. The share price is reduced by half. and IV. The number of shares doubles. In a 2-for-1 stock split, the number of outstanding shares is doubled, and the price is reduced by half. The total market value (market cap) of the issuer's stock remains the same.

Which of the following statements regarding hedge funds are true? I. They may not be suitable for discretionary accounts where account holders are not familiar with the risks associated with them. II. They are conservatively managed using no advanced or aggressive investment strategies without first receiving regulatory approval. III. They may charge both an annual fee and a fee based on the funds' profits. IV. They are subject to the same rules as mutual funds.

I. They may not be suitable for discretionary accounts where account holders are not familiar with the risks associated with them. and III. They may charge both an annual fee and a fee based on the funds' profits. Hedge funds use aggressive investment strategies, and therefore, they might not be suitable for discretionary accounts where the customer may not be familiar or comfortable with the risks associated with the product. Because they are unregulated, there are no limits to the fees they may charge.

Which of the following trades settle next business day? I. U.S. government debt II. U.S. government agency debt III. Municipal bonds IV. Listed options

I. U.S. government debt AND IV. Listed options U.S. government debt and listed options settle next day (T+1); government agency debt and municipal bonds, along with corporate securities, are subject to regular way settlement of T+2.

If a customer writes 1 ABC Jan 35 call at 13.50 and 1 ABC Jan 55 put at 12.50 when ABC is trading at 45, excluding commissions, this position will be profitable if ABC is I. above $29. II. below $29. III. above $61. IV. below $61.

I. above $29. and IV. below $61. This is a short in-the-money combination. To compute the breakeven points, add the combined premiums (26) to the strike price of the call and subtract the combined premiums from the strike price of the put. The breakeven points are 61 (35 + 26) and 29 (55 − 26). With a short combination like a short straddle, the customer makes money if the stock stays inside the breakeven points.

In recent years, the regulators have increased their concern over the financial exploitation of senior adults. FINRA's Rule 2165 became effective on February 5, 2018. FINRA states that financial exploitation may take place by a person using a power of attorney, or similar legal power, to obtain control over the specified adult's money, assets, or property by using I. deception. II. intimidation. III. coercion. IV. undue influence.

I. deception, II. intimidation, III. coercion, and IV. undue influence. Any of these tools used by a person to obtain power or control of a specified adult's account is financial exploitation. If a registered representative ever has reason to suspect that any of these are taking place, prompt notification to the appropriate supervisory person is required.

With regard to position limits, what is the bullish, or buy side of the market? I. Long calls II. Short calls III. Long puts IV. Short puts

I. long calls and IV. short puts Long calls give the holder the right to purchase stock, while short puts obligate the writer to buy stock, and both are bullish strategies.

A specialist (designated market maker) must refuse I. not-held orders. II. good-for-a-month orders. III. stop limit orders. IV. market orders.

I. not-held orders. and II. good-for-a-month orders. Specialists (designated market makers) cannot accept not-held orders or good-for-a-month orders. Not-held orders are the responsibility of floor brokers (commission brokers); these orders give the floor broker discretion as to time and price. Good-for-a-month orders are not standard orders. A time-qualified order must be day or good-til-canceled order.

A sell limit order is executed when a stock is I. rising. II. falling. III. at or below the limit price. IV. at or above the limit price.

I. rising. and IV. at or above the limit price. A sell limit order is placed above the prevailing market price. Therefore, it may be executed if the market is rising. If executed, limit orders will be filled at the limit price or better, which in the case of a sell limit is the limit price or higher.

If a customer establishes a debit spread, the customer profits if I. the spread widens. II. the spread narrows. III. the option expires. IV. the options are exercised.

I. the spread widens. and IV. the options are exercised. Because debit spreads are closed as credits, the customer profits if the spread widens. In addition, to realize maximum profit, both contracts must be exercised. If they expire, the customer loses the net debit paid for a maximum loss.

Closed-end investment company shares may be I. traded in the secondary markets, including the exchanges and over-the-counter (OTC) markets. II. sold in the OTC primary market only. III. redeemed by the closed-end investment company. IV. traded by institutional investors.

I. traded in the secondary markets, including the exchanges and over-the-counter (OTC) markets. AND IV. traded by institutional investors. Once a closed-end investment company has distributed shares, those shares are traded in the secondary market by both individual and institutional investors.

Which of the following trade actively in the secondary market? I. Open-end funds II. Closed-end funds III. Unit investment trusts IV. Real estate investment trusts (REITs).

II. Closed-end funds and IV. Real estate investment trusts (REITs). Closed-end funds and REITs trade actively in the secondary market. Open-end funds and unit investment trusts do not trade in the secondary market; instead, shares are redeemed by the issuer.

A registered representative is explaining the characteristics of a Coverdell Education Savings Account (ESA) to a customer. Which of the following statements regarding this type of savings account is correct? I. Contributions are tax deductible. II. Contributions are not tax deductible. III. When used for qualified educational expenses, withdrawals are taxable. IV. When used for qualified educational expenses, withdrawals are not taxable.

II. Contributions are not tax deductible. and IV. When used for qualified educational expenses, withdrawals are not taxable. Contributions to a Coverdell ESA are made with after-tax dollars. Distributions used for qualified educational expenses are tax free.

Which of the following accurately depicts communications with the public designated as correspondence? I. Review by a principal must occur before use. II. Review by a principal can occur either before or after use, in accordance with the firm's written procedures. III. Filing with FINRA is required. IV. Filing with FINRA is not required.

II. Review by a principal can occur either before or after use, in accordance with the firm's written procedures. AND IV. Filing with FINRA is not required. Correspondence review by a principal can occur either before or after use, in accordance with the firm's written procedures. Filing of correspondence with FINRA is not required.

Which of the following statements regarding corporate zero coupon bonds are true? I. Interest is paid semiannually. II. The discount is in lieu of periodic interest payments. III. The discount must be accreted and is taxed annually. IV. The discount must be accreted annually with taxation deferred until maturity.

II. The discount is in lieu of periodic interest payments. AND III. The discount must be accreted and is taxed annually. The investor in a corporate zero coupon bond receives the return in the form of growth of the principal amount over the bond's life. The bond is purchased at a deep discount and redeemed at par at maturity. That discount from par represents the interest that will be earned at maturity date. However, the discount is accreted annually, and the investor pays taxes yearly on the imputed interest.

An investor purchased an interest in a limited partnership, paying $10,000 in cash and signing a recourse note to the partnership under a letter of credit for $40,000. Which of the following statements are true? I. The investor's tax basis will be $10,000. II. The investor's tax basis will be $50,000. III. The investor's maximum loss will be $10,000. IV. The investor's maximum loss will be $50,000.

II. The investor's tax basis will be $50,000. and IV. The investor's maximum loss will be $50,000. A recourse note means that the limited partner agrees to pay the note no matter what happens. He is legally liable for the $40,000, which makes both his tax basis and maximum loss potential $50,000.

Which of the following statements regarding variable annuities are true? I. The number of accumulation units is always fixed throughout the accumulation period. II. The number of accumulation units can rise during the accumulation period. III. The number of annuity units is fixed at the time of annuitization. IV. The number of annuity units rises once annuitization begins.

II. The number of accumulation units can rise during the accumulation period. and III. The number of annuity units is fixed at the time of annuitization. The number of variable annuity accumulation units can rise during the accumulation period when additional units are being purchased. When a variable annuity contract is annuitized, the number of annuity units is fixed.

Having held 100 shares of GHI stock for 15 months, a customer purchases 1 GHI Jan 50 put in December. If the put is exercised before the expiration date and the long stock is delivered, which of the following statements are true? I. The premium is added to the sale proceeds. II. The premium is deducted from the sale proceeds. III. Any gain is long term. IV. Any gain is short term.

II. The premium is deducted from the sale proceeds. and III. Any gain is long term. Because the customer already held the stock long term when he purchased the put, he was not trying to stretch a short-term gain into a long-term gain. There is no effect on his established holding period of 15 months. Whenever a put is exercised, the stock's sale price (exercise price) is reduced by the premium paid for buying the put.

According to FINRA rules, duplicate confirmations of transactions must be sent to an account owner's employer—if requested to do so by the employer—whenever establishing a margin account for I. a bank officer. II. an employee of another broker-dealer. III. an independent insurance agent. IV. an officer of another broker-dealer.

II. an employee of another broker-dealer. AND IV. an officer of another broker-dealer. According to FINRA rules, when an employee of a member firm opens an account with another member broker-dealer, duplicate confirmations and account statements must be sent to the employer by the broker-dealer establishing the account when the employer requests it to do so. An officer of a broker-dealer is considered an employee.

The Bond Buyer's 30-Day visible supply includes I. issues of notes sold on a competitive basis. II. issues of bonds sold on a competitive basis. III. issues of notes sold on a negotiated basis. IV. issues of bonds sold on a negotiated basis.

II. issues of bonds sold on a competitive basis. and IV. issues of bonds sold on a negotiated basis. The visible supply includes only bonds. Notes are not considered because they do not compete directly with the bonds.

A taxable gain or loss on a long call option transaction would be recognized when I. the option is purchased. II. the option expires. III. the option is sold. IV. the option is exercised.

II. the option expires. and III. the option is sold. In addition to being exercised, call options can either be sold or allowed to expire. If either of these situations occurs, the owner of the call would determine hers gain or loss (for tax purposes) at the time of expiration or sale. This would be determined by comparing what she paid for the call versus the price at which she sold the call. If it expires, the entire amount of the premium originally paid is considered a loss. Gains or losses are not determined at the time that calls are exercised. Once exercised, the underlying security must then be sold at the current market value. Then the owner of the call would calculate her profit or loss, taking into account the premium paid, what she paid for the stock, and what she subsequently sold the stock for.

Which of the following are considered joint accounts? I. UTMA account II. Guardianship account III. Tenants in common IV. Tenants with right of survivorship

III. Tenants in common AND IV. Tenants with right of survivorship Tenants in common and tenancy with right of survivorship represent ownership by more than one party. UTMA and guardian accounts are in the name of a single fiduciary. To be a joint account, all parties associated with the account must be able to enter orders. In UTMA and guardian accounts, only the custodian and guardian, respectively, possess that authority.

If a registered representative (RR) of a FINRA member firm wants to open an account with another member firm, which of the following statements are true? I. The account may only be opened if the RR is also a principal of his employing broker-dealer. II. The account may be opened, but the RR may not engage in securities transactions that he could do through his own firm. III. The member firm opening the account must send duplicate confirmations to the employing member firm if the employing member firm has requested that the member firm do so. IV. The employing broker-dealer must receive prior written notice and give prior written consent in order for the account to be opened.

III. The member firm opening the account must send duplicate confirmations to the employing member firm if the employing member firm has requested that the member firm do so. AND IV. The employing broker-dealer must receive prior written notice and give prior written consent in order for the account to be opened. The employing broker-dealer must be notified, in writing, and give prior written approval for the account to be opened. It must receive duplicate confirmations and account statements if it has requested them.

Which of the following events will cause the special memorandum account (SMA) to decrease? I. An increase in the short market value (SMV) II. A decrease in the long market value (LMV) III. The purchase of long securities on margin IV. The short sale of securities

III. The purchase of long securities on margin and IV. The short sale of securities SMA, once created, does not go away until used. Using the SMA to buy more securities or sell more securities short will decrease the amount of SMA.

Hedge funds I. are highly regulated. II. use investment techniques suitable for most investors. III. are unregulated. IV. use investment techniques most suitable for sophisticated investors.

III. are unregulated. and IV. use investment techniques most suitable for sophisticated investors. Hedge funds are unregulated and have aggressively managed portfolios that employ investment techniques such as shorting positions, utilizing derivative products, and trading on margin—all generally considered suitable for sophisticated investors.

Rank the following from first to last in order of payment at liquidation of a corporation. I. General creditors II. Preferred stock III. Subordinated debentures IV. Accrued taxes

IV, I, III, II The complete order of liquidation is as follows: secured debt, debentures and general creditors, subordinated debentures, preferred stock, common stock.

If general interest rates increase, the interest income of an open-end bond fund will do which of the following?

Increase What does a bond fund invest in? Bonds. One of the features of an open-end company (mutual fund) is the continuous issuance of new shares. The question states that interest rates are increasing. As that new money is received, the fund's manager will be able to invest in bonds offering that higher return. This will cause the income of the fund to increase. It is not part of the question, but the increase in interest rates will likely lead to the NAV of the fund decreasing (as interest rates go up, the price of the bonds in the fund's portfolio will go down).

All of the following risks are considered diversifiable except A) financial risk. B) sovereign risk. C) inflation risk D) business risk.

Inflation Risk Purchasing power risk, also known as inflation risk, is a systematic risk and, as such, is one that cannot generally be lessened through diversification. The other choices are forms of unsystematic (nonsystematic) risk and can be reduced through diversification.

Intangible drilling costs would include all of the following except A) casing. B) fuel. C) land surveys. D) wages.

Intangible drilling costs are those associated with drilling a well, but do not include the cost of capital equipment (e.g., pumps, casing). They include wages, fuel, repairs, hauling, supplies, surveys, tests, and drilling mud, and they are incidental to and necessary for the drilling activity.

Which of the following securities is the least suitable recommendation for a qualified retirement account plan account? A) Investment-grade municipal bond B) Treasury bill C) Blue-chip common stock D) A-rated corporate bond

Investment-grade municipal bond Municipal bonds provide tax-exempt interest payments and, consequently, offer lower yields. Because earnings in a qualified retirement plan account grow tax deferred, the municipal bond is not a suitable investment. In addition, they will be fully taxed upon withdrawal.

Which of the following statements regarding the visible supply in The Bond Buyer is true?

It is a daily listing of bonds to be offered in the next 30 days. The visible supply implies that the supply of bonds will be available for the visible future.

John is the annuitant in a variable plan, and Sue is the beneficiary. Upon John's death during the accumulation period, Sue takes a lump-sum payment. What is her total tax liability?

It is the proceeds minus John's cost basis taxed as ordinary income at Sue's tax rate. Annuity death benefits are generally paid in a lump sum. The beneficiary is taxed at ordinary income rates during the year the lump sum is received. The amount taxed is the amount of the lump-sum payment minus the deceased's cost basis in the investment.

Why do hedge funds tend to limit the participation to a maximum of 100 investors?

Keeping it to 100 or less avoids the need to register with the SEC. Under SEC rules, once the number of investors in a hedge fund exceeds 100, the fund will most likely have to register with the SEC. That puts many restrictions on the fund manager's flexibility.

An investor with no current holdings wants to invest $110,000 in the stock of a company she feels looks sound with good upside potential. A history of small dividend distributions is another reason she likes it. This scenario should prompt a discussion of which of the following first?

Lack of diversification

An investor with no current holdings wants to invest $110,000 in the stock of a company she feels looks sound with good upside potential. A history of small dividend distributions is another reason she likes it. This scenario should prompt a discussion of which of the following first?

Lack of diversification Currently with no other holdings, a single stock investment with a substantial sum would not be deemed suitable. While there may be a number of suitability issues to discuss, including tax consequences regarding dividends, lack of diversification would be an obvious starting point.

When a company issues additional bonds, which of the following is true?

Leverage is increased. Leverage is the use of someone else's money at a fixed cost to benefit the common shareholders. Issuing additional bonds increases the company's debt (money borrowed from someone else), and therefore, increases leverage for shareholders.

The price of which of the following will fluctuate most with fluctuating interest rates?

Long-term bonds Long-term debt prices will fluctuate more than short-term debt prices as interest rates rise and fall. When buying a debt instrument, a person is really buying the interest payments and final principal payment. Money has a time value: the longer it takes to receive the money, the less it is worth today.

Which of the following securities carries the highest degree of purchasing power risk? A) Short-term note B) Blue-chip stock C) Long-term, high-grade bond D) Convertible cumulative preferred stock

Long-term, high-grade bond The longer a fixed-income investment is held, the more vulnerable the investor is to purchasing power risk from inflation. Although preferred stock is also a fixed-income investment, convertible preferred will increase in value with the underlying common stock.

Which the following statements regarding customer accounts is not true? A) Margin trading in a fiduciary account does not require any special documentation. B) The customer who opens a numbered account must sign a statement attesting to ownership. C) Stock held under joint tenants with rights of survivorship (JTWROS) goes to the survivor in the event of the death of one of the tenants. D) Many states publish a legal list of securities approved for fiduciary accounts.

Margin trading in a fiduciary account does not require any special documentation. Trading on margin is prohibited in fiduciary accounts except with the appropriate documentation. Numbered accounts are permitted with a letter signed by the customer. Stock held under JTWROS passes to the survivor(s) in the event of death of one of the tenants.

For a client to get immediate execution on an order, it should be placed as which of the following?

Market A market order is executed immediately at the best available market price.

A customer purchases $100,000 of original issue discount municipal bonds. How will this trade be considered for tax purposes when the bonds mature?

No capital gain Original issue discount profit at maturity is treated as part of the tax-free interest on a municipal bond. However, for a municipal bond bought at a discount in the secondary market, the discount is considered ordinary income subject to tax.

An investor sells an uncovered GHI Aug 120 call for 5½ points. What is the investor's maximum gain, maximum loss, and breakeven point?

Maximum gain = $550; maximum loss is unlimited; breakeven point = $125.50. The initial action is a sale resulting in a credit of $550. That credit is the most the investor can make. Because this is an uncovered call, the maximum loss occurs when the stock rises. Because there is theoretically no upper limit on a stock's price, the maximum potential loss is unlimited. Breakeven follows the call-up rule. Add the $550 premium to the $120 strike to arrive at $125.50.

A customer opens the following positions: Buy 100 shares of CDL @$40; sell 1 CDL Apr 40 call @2. What is the customer's maximum gain, maximum loss, and breakeven point?

Maximum gain is $200; maximum loss is $3,800, breakeven point is $38. The first step is to identify the position. This long stock with a short call (i.e., a covered call position). Breakeven is the customer's net cost. The price of the stock ($40) minus the premium ($2) received equals the $38 per share breakeven point. The strategy is to generate some income with a little protection against a decline in the price of the CDL stock. The premium income is the most this client can make. If the stock should rise well above the cost of $40 per share, the short call will be exercised and the customer will deliver the stock purchased at $40 and receive $40. Regardless of how high the stock price rises, this customer can never make more than the $2 premium. If the stock's price should decline, the call will expire unexercised. That 2-point premium protects the long stock, but only for those 2 points. Once the market price falls below $38 (the breakeven point), it is all a loss for the customer, down to a maximum $3,800 if the price drops to zero. Why doesn't the breakeven follow the "call-up" rule? That rule applies when the only positions are options. Once there is a long or short stock position along with an option position, it is the stock controlling the breakeven.

An investor opens the following positions: Sell short 100 shares of FEW @80; buy one FEW Jan 80 call @5. What is the customer's maximum gain, maximum loss, and breakeven point?

Maximum gain is $7,500; maximum loss is $500; breakeven point is $75. The first step is to identify the position. This is a short sale with a protective call. That is, the customer has shorted the stock and purchased a call to protect the upside. This investor will breakeven when the stock's price is equal to the sale price ($80) minus the premium paid ($5), or $75. Short sellers lose when the price of the stock goes up. That means a short sale has potentially unlimited risk of loss. A way to ensure that the loss is limited is to purchase a call on the stock. If the stock should rise significantly above the $80 received on the short sale, instead of having to cover the short in the market at that high price, the investor will exercise the long call and be able to cover at $80. That means the maximum loss is the $500 premium paid for the protection. If the stock's price falls, and it can decline to zero, the investor's cost to cover is zero, resulting in a profit of $8,000 minus the premium of $500. Why doesn't the breakeven follow the "call-up" rule? That rule applies when the only positions are options. Once there is a long or short stock position along with an option position, it is the stock controlling the breakeven.

A high net worth couple in their 40s earning a combined $480,000 annually have $100,000 they would like to conservatively invest. Given their income bracket, they stress reducing tax exposure and also note that having access to the funds is important. Which of the following would be the most suitable investment?

Municipal bond ETF Municipal bonds and GNMAs are the two conservative choices offered here. However, distributions of interest from the GNMA fund will be taxed at the federal, state, and local level. Income distributions from the municipal bond ETF will be tax exempt at the federal level, reducing tax exposure at their income level, and is the better choice. Direct participation programs are not conservative or liquid, and special situation mutual funds offer no reduced tax exposure.

You are having a discussion with one of your clients regarding suitable investments. The client is in a high-income tax bracket and has a high net worth as well. During the conversation, your client mentions several investments that she is thinking about that might be beneficial to her now. Of those listed, which would be the best recommendation?

Municipal bonds Of the choices listed, only municipal bonds offer a tax consequence suitable for high-income, high-net-worth investors in the form of tax-free interest payments. Corporate bond interest, mutual fund dividends, and income earned from REITs are all taxable.

Both of the securities acts of 1933 and 1934 address fraud in the securities industry. Regarding the antifraud provisions outlined in these acts, which of the following statements is true?

No securities, issuers, or transactions are exempt from the antifraud provisions of these acts.

When an officer or director acquires control stock when a company goes public and then wants to sell the securities to a retail investor, what is the mandatory holding period?

None Because the securities were received in a public offering, they are registered securities (not restricted), and therefore, there is no holding period. However, the sale is subject to Rule 144 volume limits. Control stock that is received in something other than a public offering is restricted and would have a six-month holding period in addition to volume limitations.

Upon notification of the death of a client, which of the following actions would not need to be taken by the registered representative assigned to the account? A) Obtaining the names of the beneficiaries of the estate for the purpose of notifying all parties B) Canceling all day orders currently entered for the account C) Marking the account Deceased until all proper documentation has been received D) Canceling all good-til-canceled (GTC) orders currently entered for the account

Obtaining the names of the beneficiaries of the estate for the purpose of notifying all parties Upon being notified of the death of a client, the registered representative assigned to the account should cancel all open orders (GTC and day) and mark the account Deceased. The firm should not permit any trades until proper documents are received from the estate representative. There is no requirement, nor is it the responsibility of the firm to contact the decedent's attorney or beneficiaries.

Proponents of which of the following technical theories assume that small investors are usually wrong?

Odd lot. Odd lots are usually traded by small investors; some analysts believe small investors are generally wrong.

If a customer transfers his holdings from one fund to another within the same family of funds, what are the tax consequences?

On the transaction date, any gain or loss is recognized for tax purposes. An exchange is a taxable event. The cost basis of the shares in the original account must be compared to their redemption value. Any gain or loss is recognized in the year of the exchange. The exchange privilege allows the investor to avoid paying an additional sales charge. It does not allow the investor to avoid taxes.

A client has purchased a nonqualified variable annuity from a commercial insurance company. Before the contract is annuitized, your client, age 60, withdraws some funds for personal purposes. What is the taxable consequence of this withdrawal to your client?

Ordinary income taxation on the earnings withdrawn until reaching the owner's cost basis. Contributions to a nonqualified annuity are made with the owner's after-tax dollars. Distributions from such an annuity are computed on a last-in, first-out basis, with the income taxed first. Once the cost basis is reached, any further withdrawals are a nontaxable return of principal. Because the client is older than 59½ at the time of distribution, the additional 10% penalty tax is not incurred.

Which of the following is not a type of CMO tranche? A) TAC B) PSA C) Z D) PAC

PSA PSA is the Public Securities Association. What does that have to do with CMOs? Plenty. It is the PSA table that is used to project the prepayment schedule for CMO tranches. However, unlike the PAC (planned amortization class) or the TAC (targeted amortization class) and the Z (zero), it is not a tranche.

Which of the following actions could affect the holding period on a long position in IBS stock? A) Sale of 1 IBS out-of-the-money call B) Purchase of 1 IBS call C) Sale of 1 IBS out-of-the-money put D) Purchase of 1 IBS put

Purchase of 1 IBS put The purchase of a put affects a stock's holding period when the stock has been held short term. The holding period begins anew once the put position is closed.

If a client who seeks diversification through real estate is concerned about illiquidity associated with investing in real estate, which of the following investments is most suitable?

Real estate investment trust Real estate investment trusts are best suited to the client because they are market-traded securities that provide an investor with a liquid market in which to invest in real estate.

A customer wishes to diversify an investment portfolio to include real estate. However, the customer is concerned that real estate tends to be illiquid. Which of the following would be a suitable recommendation?

Real estate investment trusts REITs (real estate investment trusts) allow investors the opportunity to invest in real estate and provide a high level of liquidity because they are publicly traded on exchanges and OTC. The real estate DPP does provide an opportunity to invest in real estate, but it is considered illiquid because there is generally no available secondary market. When a mutual fund is a diversified company, it means that it complies with the 75-5-10 rule, not that its portfolio is diversified into real estate. GNMA securities are debt obligations and do not provide the equity diversification this investor is seeking. Please note: As pointed out in the LEM, there has been a growth in the number of nontraded REITs. Obviously, they do not have the liquidity of those that are publicly traded. For exam purposes, unless something in the question indicates that the REIT is nontraded, you can safely assume it is a liquid investment.

If a corporation attaches warrants to a new issue of debt securities, which of the following would be a resulting benefit to the corporation?

Reduction of the debt securities' interest rate Usually, a warrant is issued along with a debt instrument, which is an enhancement that allows the issuer to offer a slightly lower interest rate.

Once individuals have passed the Series 7 exam and are now registered as general securities registered representatives, compliance with Regulation BI would allow including which of these on their business card following their name?

Registered representative. Regulation BI limits the use of the words adviser or advisor to those who are registered as investment advisers or representatives of those who are. Passing the Series 7 exam allows one to use the term registered representative. Most of our Series 7 students go on to pass the Series 66 exam and become IARs. In that case, investment adviser representative is permitted. **This question deals with material not covered in your LEM, but it relates to recent rule changes and/or student feedback.

When used on the exam, the term "nonexempt security" refers to a security that must register with the SEC. There are cases, however, when registration may be avoided. In most cases, that would be when the transaction involving that security is exempt. Each of the following is a transaction exemption found in the Securities Act of 1933 except A) Regulation M. B) Regulation D. C) Regulation A+. D) Regulation S.

Regulation M This is a process of elimination question. There is a Regulation M under the act, but it has no relevance to your exam (which is why it is nowhere in your material). It sometimes happens that the correct answer choice is something you've never heard of. That happens most often when the question ends with the word except. Regulation A+ is the exemption for small and medium corporate offerings. Regulation D is the most significant private placement exemption. Regulation S is the exemption for offers and sales made outside the United States by both U.S. and foreign issuers.

Which of the following would be the least considered factor in determining if a particular type of options trading is suitable for a customer? A) Understanding maximum gain or loss potential B) Ability to meet margin calls C) Willingness to assume risk D) Understanding the strategy being employed

Remember that while most firms require that options trades be done in margin accounts, options purchases are not marginable. Therefore, of the choices listed, the ability to meet margin calls would be the least considered factor regarding suitability.

The sale of nonexempt securities may take place without an SEC registration if done in a manner that qualifies for a transactional exemption. An example of this would a sale complying with A) Rule 498. B) Rule 156. C) Rule 506(b). D) Rule 135A.

Rule 506(b). If the transaction is exempt, a security that would otherwise have to be registered is exempt from registration. Rule 506(b) is part of the private placement exemption under Regulation D of the Securities Act of 1933. Rule 498 deals with a summary prospectus for a mutual fund and Rules 156 and 135A deal with advertising.

On the morning of the ex-date for a cash dividend, which of the following orders on the order book will not be reduced? A) Sell stop B) Sell limit C) Sell stop limit D) Buy limit

Sell limit It is sell limit and buy stop orders that are placed above the market and are not reduced. Orders entered below the prevailing market [unless marked DNR (do not reduce)] are reduced on the morning of the ex-date by the amount of the cash dividend. Those orders are buy limits and sell stops, including sell stop limits.

Which of the following would accelerate a decline in a bear market?

Sell stop Sell stops, placed below the current market, become market orders to sell when the stock trades at or through (below) the stop price. Market sell orders can accelerate declines in the price of the stock.

An investor purchases 100 shares of a bond ETF at a price of $50 per share on September 5, 2019. On November 1, 2019, and February 1, 2020, the fund distributes a $0.50 per share dividend. On May 11, 2020, the investor sells all the shares at $57 per share. What are the 2020 tax consequences of the sale?

Short-term capital gain of $700 Taxation of an ETF is similar to that of a mutual fund. The question asks about the tax consequences of the sale, so we ignore the dividend distributions. Buying at $50 per share in September and selling at $57 per share the next May is a $700 capital gain over a period of less than one year. It is not part of the question, but the dividends would be taxed as interest because they are coming from bonds.

One of your customers called you with the good news that they are new grandparents. They are looking for a way to provide funds for the new child's college education and would like some kind of tax break if possible. What would be the most suitable suggestion?

Start a Section 529 plan for the child Among the many benefits of the Section 529 plan is that all earnings between now and withdrawal can be tax free when used for qualified expenses. In addition, if the grandparents have a substantial estate, they can contribute up to $75,000 ($150,000 if a joint gift) without any gift tax ramifications. The income from an UTMA account is taxable in the year received. The annual accretion is taxable on the zero-coupon bonds. Although it is true that the child's income from the investments may be below the taxable threshold today, in the later years that might not be the case. IRAs can only be funded from earned income, not gifts.

Which of the following governmental bodies receive the least amount of their revenues from property taxes?

State governments. State governments generally do not assess property (ad valorem) taxes. These are assessed by local governments. Generally, state governments receive most of their income from sales and income taxes.

Which of the following securities would have a Moody's MIG rating? A) GOs B) BAs C) TANs D) T-bills

TANs TANs are tax anticipation notes. These are short-term municipal securities and that is what Moody's MIG ratings represent. MIG stands for Municipal Investment Grade. GOs are rated with the normal letter ratings and BAs (bankers' acceptances) and T-bills are not municipal securities.

The name of the FINRA system that tracks both NYSE and NASDAQ transactions is A) DMM (Designated Market Maker). B) ECNs (Electronic Communications Networks). C) TRACE (Trade Reporting and Compliance Engine). D) TRF (Trade Reporting Facilities).

TRF (Trade Reporting Facilities). The TRF is the only FINRA system that tracks transactions on the NYSE and Nasdaq, including OTC (over-the-counter) transactions. ECNs are electronic trading systems that automatically match buy and sell orders at specified prices; they are not a tracking system. TRACE tracks corporate and government agency bond transaction in the OTC market. DMM's are the specialists who work at each of the trading posts on the NYSE. They are responsible for maintaining a liquid and orderly market for stocks.

A customer purchased 10 ABC 9s of 2045 convertible debentures at 99. The debentures are callable at 101. The conversion ratio is 40. Some time later, the debentures are called while the common is trading at $24 and the debenture is trading at 98. Which of the following options would be most beneficial to the customer? A) Sell the bonds B) Tender the bonds to the corporation C) Convert the bonds and sell the common stock D) Wait for a better offer from the corporation

Tender the bonds to the corporation First of all, recognize that the investor purchased 10 of the debentures. They have a coupon of 9% and mature in 2045. None of that is relevant to answering the question, but we want to be sure you understand the terminology.The option most beneficial to the investor is tendering the debentures to the corporation for $10,100 (10 times $1,010). If the debentures were sold on the market, the investor would receive $9,800 (10 times $980). If the debentures were converted into common, the investor would receive 400 common shares (40 shares per debenture times 10) that could be sold for their current price of $24, for a total of $9,600.

An investor purchases $1,000 of the XYZ Growth Fund on a Tuesday. The order is time-stamped at 2:45 pm ET. When will the investor's price per share be determined?

That day based on the market close Mutual funds use forward pricing when investors buy or sell. That means the investor will buy or sell at the price based on the next computed net asset value (NAV). The NAV must be calculated at least once each business day at market close (4:00 pm ET). This order was entered before the close. Therefore, the next time the price is calculated is that day's market close.

Your mutual fund has sent you a Form 1099, listing some long-term capital gains on which you must pay taxes. You are concerned that the 1099 is in error because you have owned your shares for only four months. Which of the following statements is true?

The 1099 is correct because in this case, the holding period to be considered is that of the investment company, not yours. The investment company designated the gains as long term because the company held the securities for more than a year before selling them. The holding period on your shares is relevant only if you redeem your shares for a gain.

An investor wishing to determine the liquidity of a corporation would find which of the following most helpful? A) The book value B) The price-to-earnings ratio C) The current ratio D) The dividend payout ratio

The Current Ratio The current ratio compares the company's current assets to its current liabilities. It is one of the two tested liquidity ratios. The other is the quick ratio (quick asset ratio or acid-test ratio). That is similar to the current ratio except the inventory is excluded.

Which of the following is not a characteristic of certificates of deposit (CDs)? A) A CD can be negotiable or nonnegotiable. B) The Federal Deposit Insurance Corporation (FDIC) provides insurance for CDs to $500,000. C) A CD is often issued by a bank. D) A CD may be payable to the bearer or registered in the name of the investor.

The Federal Deposit Insurance Corporation (FDIC) provides insurance for CDs to $500,000. The FDIC provides insurance for CDs up to $250,000. All of the other characteristics are applicable to CDs.

Which of the following occurs in a partnership account if one partner dies?

The account is frozen until a new or amended partnership agreement is received. Upon a partner's death, a partnership account is automatically frozen until a new or amended partnership agreement is received. The deceased partner's share usually goes to an estate, not to the other partners.

Three 3% bonds are listed in the newspaper. One bond will mature in one year, another bond will mature in 10 years, and the third bond will mature in 20 years. If interest rates are going up, which bond will have the greatest decrease in value?

The bond with the 20-year maturity Long-term bonds have the greatest interest rate risk. A bond with only one year to maturity will trade very close to par.

Tenants in common (TIC) ownership provides that a deceased tenant's fractional interest in an account is retained by which of the following?

The deceased tenant's estate TIC ownership of an account provides that a deceased tenant's interest in an account is retained by that tenant's estate and not passed on to the surviving tenant.

Which of the following describes additional paid-in capital?

The difference between the total dollar amount received from the issuance of common stock and the stock's aggregate par value Additional paid-in capital is the difference between the dollar amount received from the sale of stock and the stock's aggregate par value. Earned surplus is another name for retained earnings.

Electronic communications networks are most likely to be categorized as being part of which type of market?

The fourth market. Electronic communication systems are considered part of the fourth market because they are primarily used for transactions between institutional investors. Yes, the fourth market is a secondary market, (trading in outstanding issues), but always choose the most specific answer. The primary market is the trading of a new issue.

A recommendation to purchase a security traded in which of the following venues would most likely result in the greatest scrutiny by your manager?

The grey market Securities trading on the "grey market" are not quoted on any U.S. quotation system. Broker-dealers are not willing or able to publicly quote these securities because of a lack of investor interest, company information availability, or regulatory compliance. **This question deals with material not covered in your LEM, but it relates to recent rule changes and/or student feedback.

When a registered representative recommends a municipal bond purchase to a customer, which of the following would be of least consideration regarding suitability? A) The bond's rating B) The intended use of funds raised by the issue C) The customer's state of residence D) The customer's tax bracket

The intended use of funds raised by the issue The customer's state of residence and tax bracket are important because these factors help establish the tax benefits offered by the municipal bond. The bond's rating is important in evaluating the credit risk assumed by the investor. While the intended use of the funds may be of interest to the customer, it would be of little consideration when making a purchase recommendation.

What are the tax consequences an investor incurs when exercising the conversion privilege within a family of funds?

The investor treats the exchange as a sale and new purchase. When exchanging one fund for another in the same fund family, the exchange is done at NAV. This avoids any sales charges. The IRS considers this as the sale of the old fund (capital gain or loss applies) and the purchase of the new fund. That begins a new cost basis and holding period. The Section 1035 exchange allowing investors to move from one investment to another without current tax consequences is applicable only to insurance products.

Which of the following does not participate in the syndicate (joint account) for a municipal underwriting? A) A municipal broker-dealer B) A financial advisor acting as a municipal securities dealer C) A bank dealing in municipal securities D) The issuing municipality

The issuing municipality A syndicate or joint account helps spread the risk of underwriting an issue among a number of underwriters—in this case, banks and broker-dealers who deal in municipal securities. The issuing municipality would not be a member of the syndicate (joint account) formed for the purpose of selling their municipal securities to the public.

Three years ago, a customer purchased 300 shares of ACE Fund. He sold the shares on August 15 for a loss of $400. He then purchased 300 shares of the same fund on September 4 of the same year. If the investor is in a 10% tax bracket, how will the loss be treated for tax purposes in the current year?

The loss is not deductible. Because the customer repurchased the shares within 30 days of the loss transaction, the loss is disallowed under the wash sale rule, and therefore, is not deductible. A wash sale occurs when the same shares are purchased within 30 days before or after the date of sale in which the loss is incurred.

One of the computations in a margin account is that of the SMA. Which of the following actions would cause the SMA to increase? A) The withdrawal of cash from a short margin account B) The purchase of securities in a long account C) The market value of short securities decreasing D) The market value of long securities decreasing

The market value of short securities decreasing Excess equity creates SMA in the account. Excess equity is created when the stock in a margin account moves in a favorable direction. In the case of a short margin account, the equity goes up as the market value of the securities goes down. In the case of a long account, a decline in the market value causes the equity to decrease. The purchase of securities in a long account may decrease SMA if SMA is used to meet the margin call. If not, SMA remains the same; it does not increase. Withdrawing cash will cause the SMA to go down if the funds are withdrawn from the SMA. Otherwise, the SMA remains the same; it does not increase.

Which of the following may not lead to an industrial development bond being called? A) Interest rates are falling. B) The facility is destroyed by a storm. C) The municipality is approaching a statutory debt limit. D) Funds are available in the surplus account to call the bond.

The municipality is approaching a statutory debt limit. An issuer of industrial development revenue bonds is likely to call bonds to reduce interest costs when interest rates are falling, discontinue interest payments if the facility is destroyed by a natural disaster, or reduce debt if funds are available in a surplus account. Industrial development revenue bonds are not affected by the issuer's statutory debt limits, as they affect the issuance of general obligation bonds only.

Which of the following documents would describe the roles of the general and limited partners in a limited partnership offering? A) The certificate of limited partnership B) The prospectus C) The subscription agreement D) The partnership agreement

The partnership agreement The partnership agreement describes the roles of the general partners (GPs) and the limited partners (LPs). The certificate of limited partnership is a document filed with the limited partnership's home state for legal recognition. The LPs complete the subscription agreement (essentially the request to purchase) disclosing important personal information. The most important is the financial information, such as income and net worth. The GPs use that in determining suitability. The subscription agreement is not effective, and the LPs are not accepted into the partnership, until it is signed by the GPs. The prospectus is a document that discloses all material facts regarding the investment to investors. Note that if the program is a private placement, in lieu of a prospectus, the investor receives a private placement memorandum containing essentially the same information.

A sophisticated investor wants to purchase stock of a foreign company or an American depositary receipt (ADR) representing the shares of that company. The purchase would align with the investor's goal of growth and income, but he makes several statements about the potential purchase, and only one of them is accurate. You feel it is important to point out and discuss from a suitability perspective which statements were and were not accurate. Which of the following is the accurate statement?

The purchase of ADRs representing the shares exposes me to currency risk. From a suitability perspective, correcting any inaccuracies about an investment that an investor might have is important. Currency risk cannot be avoided when investing in foreign companies, either directly or using ADRs. While ADRs trade on U.S. exchanges, foreign shares do not, and ADR issuers generally do not pass on voting rights to the ADR holders.

A municipal bond dealer is making a bona fide quote. Which of the following statements regarding such a quote is true?

The quote must have a reasonable relationship to fair market value. A bona fide quote must have a reasonable relationship to fair market value and can be made in consideration of any anticipated market movement. A bona fide quote is one the dealer is prepared to buy or sell on, as opposed to a workable, nominal, or subject quote. On the offer side of a bona fide quote, a dealer may make an offer to sell bonds that it does not hold in its own inventory, but it must know where to obtain the bonds if they are needed to complete the transaction.

Which of the following is limited in the case of a limited tax municipal bond?

The type of tax that can be used to service the debt A general obligation (GO) bond may be backed by a specific tax. For example, a limited tax GO may be serviced only from sales tax revenue, not income tax revenue. As the source of debt service is limited (it is not backed by the full taxing authority of the issuer), these bonds are sold with higher yields than conventional GOs.

If a writer of an XYZ equity call option is assigned, which of the following should be delivered to the Options Clearing Corporation?

The underlying XYZ security When a call is exercised, that specific security must be delivered by the assigned writer. The option contract does not allow for exercise settlement in cash, securities of equivalent value, or securities exercisable to purchase the underlying securities such as rights or warrants.

Regulation T governs the purchase of all of the following except A) American depositary receipts. B) corporate convertible bonds. C) listed options. D) U.S. government bonds.

U.S. government bonds. Regulation T applies only to nonexempt securities. Because governments and municipals are exempt, there is no federal regulation.

A registered representative speaking to a customer is explaining registered funds that invest in nonregistered hedge funds. Which of the following statements is not correct? A) To divest of your fund of hedge fund investment, the shares will need to be redeemed by the mutual fund issuer. B) These funds, called funds of hedge funds, eliminate all of the risks associated with hedge funds. C) Hedge funds are directly available to sophisticated (accredited) investors, while funds of hedge funds allow all investors to invest in hedge funds indirectly. D) These funds generally allow purchases with an initial investment that is lower than what is required to invest directly in a hedge fund.

These funds, called funds of hedge funds, eliminate all of the risks associated with hedge funds. Because the portfolio of the registered fund consists of shares of nonregistered hedge funds, virtually all of the risks associated with hedge funds are transferred to the mutual fund. Funds of hedge funds allow all investors—not just accredited investors—to have access to hedge fund investments, and they are likely to have lower initial investment requirements, making that access even easier. To divest of fund of hedge fund shares, the issuer would have to redeem them from the investor, as is the case with all registered mutual funds.

If a 40-year-old customer earns $65,000 a year, and his 38-year-old spouse earns $40,000 a year, how much may they contribute to IRAs?

They may each contribute 100% of earned income or the maximum annual allowable dollar limit, whichever is less, to an IRA. Regardless of the amount, individuals or couples may contribute to their IRAs if they have earned income. Each is entitled to contribute 100% of earned income up to the maximum allowed. However, if either or both of them are covered under a qualified plan, limits may exist on the deductibility of the contributions.

Which of the following documents must an existing customer sign to establish a discretionary account?

Trading authorization To establish a discretionary account, the agent must receive written authorization from the customer(s) in whose name(s) the account has been established. An existing customer has already completed the new account application and signed any required customer agreements.

If a customer sells short 100 ABC at 40 and sells 1 ABC Jun 40 put at 4, what is the maximum loss?

Unlimited. A short stock position subjects an investor to unlimited risk. Writing a put against a short stock position only protects the investor to the extent of the premium received.

Which of the following best describes a debenture?

Unsecured corporate debt

Which of the following best describes a debenture?

Unsecured corporate debt A debenture is unsecured corporate debt.

XYZ Corporation has set Friday, January 23, as the record date for its next quarterly dividend distribution. To receive the dividend, a customer, long 1 XYZ Feb 40 call, must issue exercise instructions on or before

Wednesday, January 21. Dividends are paid to investors who are owners of record as of the close of business on the record date. When a call option is exercised, money and stock are exchanged (settlement) on the second business day after notice is given to the Options Clearing Corporation. Therefore, an investor who wishes to receive a dividend must exercise a call no later than the second business day before the record date (i.e., the day before the ex-date). This is no different from anyone else purchasing stock before the ex-date.

An investor believes that ICBS, a Nasdaq security, is overpriced at 40. She can sell ICBS short in the over-the-counter (OTC) market under which of the following circumstances?

With no restrictions As on exchanges, short sales on the Nasdaq Stock Market can occur at any time in the trade sequence.

An option investor might do all of the following except A) make an opening sale. B) hedge a short stock position with a long call. C) hedge a long stock position with a short put. D) make a closing purchase.

Writing a put does not reduce the risk of a long stock position. The short put creates an obligation to purchase additional shares if the put is exercised (which will happen if the stock falls). A long call is an effective hedge against a short stock position.

According to Municipal Securities Rulemaking Board (MSRB) rules, can a municipal securities representative give $50 crystal vases to 10 of his favorite clients?

Yes, he is permitted. If each gift meets the $100 annual gift limitation, then they are permitted.

The City of Podunk has an outstanding 25-year maturity issue that is callable in seven years. It has prerefunded the issue and established an escrow account containing the proper government securities with face amounts and maturities approximating the call provisions of the original issue. In quoting the original issue, which of the following must be used?

Yield to call When a bond issue is prerefunded, the issuer is going to redeem the bond on the first call date. The yield must be quoted to call.

A risk associated with investing in most bonds is reinvestment risk. What type of bond can an investor buy that does not expose the investor to reinvestment risk of interest?

Zero-coupon bonds Zero-coupon bonds do not expose investors to reinvestment risk of interest. The risk is that as periodic income is received from an investment, such as bond interest, the investor cannot find another investment to reinvest into offering the same rate of return for the same level of risk. Because zero-coupon bonds do not pay periodic interest, (the return of the face value at maturity is the income), there is nothing to reinvest. The fact that a bond is selling at par, premium, or discount does not remove the reinvestment risk.

A customer holds the following positions: Short 100 XYZ shares at 40 Short 1 XYZ Oct 40 put at 5 With XYZ trading at 35, the customer is assigned an exercise notice on the put, and he uses the stock purchased to cover the short stock position. This results in

a $500 gain. When exercised, the customer is forced to buy stock at 40 that is used to cover the short position for no gain or loss. Because the premium of $500 was received, the investor has a gain of $500 on this position.

The diversification and professional management offered by many investment companies tends to lower the investor's risk. That does not mean elimination of risk. Of the following, it is likely the greatest risk would be investing in A) an broad market index fund. B) a high-yield bond mutual fund. C) an equity UIT. D) a 2x ETF.

a 2x ETF. Leveraged funds (2x or 3x) are considered the investment companies carrying the highest risk. Although high-yield bonds, at least on an individual basis, are considered high risk, the diversification and professional management of the mutual fund reduce (do not eliminate) the risk.

A direct participation program (DPP) would not be structured as A) an LLC B) a limited partnership. C) a C corporation D) an S corporation

a C corporation Although most DPPs are structured as limited partnerships, they may also use the other business forms allowing flow-through of income and losses (s corporations and LLCs). Because the C corporation is a taxable entity, it is not used as the structure for a DPP.

An investor purchases one ABC Jul 50 put and sells one ABC Oct 50 put. This strategy is

a calendar spread. A calendar (or time or horizontal) spread is one where the options are of the same type (long and short puts or long and short calls) on the same security, but with different expiration dates. We know this is not a debit spread because the option October expiration has more time value than the July expiration. Therefore, the option sold (Oct) will have a higher premium than the one bought resulting in a credit. Although a combination involves different expiration months, it is either two longs or two shorts, not a long and a short.

An investor purchased a new issue corporate zero-coupon bond for $600. The bond has a maturity of 20 years. Six years later, the investor sells the bond for $700. For tax purposes, this would result in

a capital loss of $20. The $400 discount is accreted over the 20 years to maturity. That is an annual accretion of $20. After 6 years, that is $120, making the tax basis of the bond $720. Because the sale at $700 is $20 less than the basis, the investor has a long-term capital loss.

An option writer liquidates a position by purchasing an option. This order must be marked as

a closing purchase. Writers liquidate (close) their short positions by purchasing back an option of the same series as the one they wrote. This order is known as a closing purchase transaction.

An investor is following the new issue municipal bond market. The primary source material is found in the Daily Bond Buyer. This publication is distributed on

a daily basis. This is a "Who is buried in Grant's Tomb" type of question, and there are students who do miss it. We won't say there are many questions this easy on the exam, but there are a few—please do not miss them.

You receive a not-held order from a customer who wants you to buy 1,000 shares of ABC when the price is right. Under NYSE rules, this order is

a day order. Unless the customer instructs you otherwise, not-held orders must be executed on the day received.

A client writes 1 Dec 45 put and buys 1 Dec 60 put. This is

a debit bear spread. This is a debit bear spread, and bears buy puts. The 60 put is worth more than the 45 put because it has a higher strike price.

An arrangement in which the registered representative has the authority, or power of attorney, to make trades from funds in the account without prior approval from the investor is known as

a discretionary account. Discretionary accounts are arrangements in which the registered representative has the authority, or power of attorney, to make trades from funds in the account without prior approval from the investor.

One of your clients has appointed his daughter as the trusted contact person per FINRA Rule 2165. She contacts you to explain that her father's cognitive abilities are declining. Because of that, before it gets too late, she wants to know what can be done to give her control over the account. It is likely that the best suggestion would be to have her father sign

a durable power of attorney. A durable power of attorney is used when the account owner has diminished physical or mental capacity. It is durable because it survives the client becoming legally incompetent (but does not survive death).

For the underwriting of a municipal bond issue, competitive bids are submitted by underwriters as

a firm commitment. For new municipal bond issues, underwriters must submit bids for the entire bond offering—a firm commitment. Standby commitments are used only for corporate stock rights offerings. Best efforts commitments are used for corporate securities, and an all-or-none commitment is a type of best efforts commitment.

A sharing arrangement in which only deductible costs are apportioned to the investor, with the sponsor bearing all capitalized costs is called A) an overriding royalty arrangement. B) a functional allocation. C) a reversionary sharing arrangement. D) a carried interest.

a functional allocation. Functional allocation is a sharing arrangement in which the general partner pays for all tangible drilling costs (capitalized costs), and the limited partners pay for all intangible drilling costs (deductible costs).

An order confirmed for the entire underwriting syndicate's benefit is called

a group net order A municipal group net order is credited to syndicate members according to their percentage participation in the account. This order type is given priority over designated or member takedown orders (but not over presale orders). The normal order of priority is presale orders, group or syndicate orders, designated orders, and member orders.

The type of REIT in which the issuer purchases real estate properties that produce income as well as mortgages on income-producing real estate properties is

a hybrid REIT. A hybrid REIT receives its name from the fact that it combines the strategy of an equity REIT and a mortgage REIT. On the equity side, it takes an ownership position in real estate properties. The goal is income from rentals with possible capital appreciation when the properties are sold in the future. On the mortgage side, they provide financing for income-producing real estate by purchasing or originating mortgages and mortgage-backed securities (MBS) and earning income from the interest on these investments. Therefore, the hybrid REIT deserves the name because it is a mixture of both types of REITs. A participating trust has nothing to do with REITs and will not be the subject of a question on your exam.

A broker-dealer informs a customer that her order was not executed because of stock ahead. The order was most likely

a limit order. When there are a number of limit orders entered at the same price, they are filled in the order in which they have been received. It is possible that those orders received later could not be executed because the execution of the earlier orders has caused the market price to move away from the limit price. Market orders are always executed at the best price in the market (even though that might be far away from what the investor expected). If the investor wants immediate execution—turn in a market order. If the client wants a specific price—turn in a limit order but give the warning that it might never be executed.

An investor is long 1 ABC Mar 200 call at 4, and long 1 ABC Mar 200 put at 2. The customer has created

a long straddle. This is a long straddle. With a long straddle, the customer expects the stock's price to be volatile, but does not know the direction. A short straddle would be the short call and short put. With a short straddle, the customer expects price stability and hopes both options expire unexercised. A straddle, long or short, involves the same underlying stock, same expiration month, and same strike price. A combination is the same underlying stock, but either the expiration month and/or the strike price of the options would be different.

A municipal bond dealer gives your firm's trading desk an estimate of a municipal security's market value. This is

a nominal quote. A nominal, or subject, quotation indicates a dealer's estimate of a security's market value. Nominal quotations are provided for informational purposes only and are permitted if the quotes are clearly labeled as such. A workable indication is usually a firm bid price from a dealer and holding a quote is one that is firm for a specified time.

A customer who has, as part of her account holdings, unlisted REITS, as well as a limited partnership interest in an oil and gas program, may expect her servicing member firm to show

a per share estimated value of the securities. A general securities member must include in a customer account statement a per share estimated value of a DPP or unlisted REIT security, in a manner reasonably designed to ensure that the per share estimated value is reliable.

The ABCD Corporation's common stock is listed on the NYSE. For expansion purposes, ABCD plans to issue an additional 300,000 shares using authorized, but unissued, shares. This additional public offering (APO) is classified as

a primary offering. New and primary are the same, and because the issuer receives the sales proceeds, these are known as primary or issuer offerings. Once these shares trade later between investors, this is the secondary market. A registered secondary offering is the sale of previously issued stock to the public. An example would be a selling stockholder (typically an insider or affiliate) wishing to sell more than Rule 144's safe harbor.

A large institution dealing with several different broker-dealers would probably find it beneficial to open

a prime brokerage account. A prime brokerage account is one in which a customer—generally an institution—selects one member firm (the prime broker) to provide custody, trading, and other services, while other firms, called executing brokers, typically execute most of the trades placed by the customer.

The individual responsible for the overall supervision of all of a firm's options activities on behalf of its customers must be

a registered options principal (ROP). Any firm that transacts options trades will have a ROP to oversee trading and a ROP designated to carry the ultimate responsibility of supervision and ensure that options compliance procedures are followed.

A customer, concerned about a possible pull-back in XYZ stock, instructs her broker to "Sell my XYZ stock if it falls to 40, but I don't want less than 39.75 for my shares." The broker should enter

a sell stop limit order. A sell stop limit order would be appropriate (sell 100 XYZ 40 stop 39.75). Once the price of XYZ trades at or through (below) 40, the order is elected and becomes a limit order to sell at 39.75 or better (higher).

An investor purchased 100 shares of ABC common stock at $50 per share on June 17, 2019. On May 11, 2020, with the ABC selling at $60, the investor hedges by purchasing one ABC Oct 55 put at 2. Immediately prior to the expiration date, ABC is selling for $45 per share and the put option is exercised using the long stock for delivery. This would result in

a short-term capital gain of $300. The investor purchased a protective put against a long position that had a short-term holding period. That means the holding period of the stock is erased and does not restart until the earliest of the date the investor disposes of the stock, exercises the put, sells the put, or the put expires. That means this investor's gain will be short term. The gain is the difference between the cost and the proceeds. When exercising a put, the cost of the stock is the investor's cost basis. The exercise price minus the option premium paid is the sale price. In our question, the cost is the $50 initial purchase price ($5,000 total). The sale price is the 55 strike minus the $2 premium, or $5,300. That results in a short-term capital gain of $300.

For a regular standardized option, any gain on the sale of the contract is

a short-term capital gain. Regular standardized options have a maximum expiration of 9 months, so a gain on these types of contracts can only be short term for tax purposes. It is only the LEAPS options where it is possible to hold a long option position for more than 12 months. That is the only case where a long option can realize long-term treatment. The question is dealing solely with the options contract, not the underlying security.

All of the following may be included in an advertisement for a collateralized mortgage obligation (CMO) issue except A) a disclosure of the CMO's coupon rate and final maturity date. B) a statement that the CMO is guaranteed by the U.S. government. C) a disclosure that payment assumptions may or may not be met. D) a generic description of the CMO tranche.

a statement that the CMO is guaranteed by the U.S. government. The U.S. government does not issue or back CMOs. It is also misleading to state or imply that a CMO's anticipated yield or average life is guaranteed. CMOs must include the coupon rate and the final maturity date, a generic description of the CMO tranche, and disclosure that payment assumptions may or may not be met.

A taxpayer's most advantageous tax benefit is A) a depletion allowance. B) straight-line depreciation. C) a tax deduction. D) a tax credit.

a tax credit. A tax credit reduces a person's tax liability dollar for dollar. Deductions, depreciation, and depletion reduce taxable income.

It is not uncommon for one company to attempt to take over another by acquiring a significant percentage of its voting shares. This is called

a tender offer. The SEC defines tender offer as "an active and widespread solicitation by a company or third party (often called the bidder or offeror) to purchase a substantial percentage of the company's securities. Bidders may conduct tender offers to acquire equity (common stock) in a particular company or debt issued by the company."

FINRA Rule 2232 deals with customer confirmations. Among the requirements in that rule are all of the following except A) if the broker-dealer is acting as principal, stating whether it is a market maker in the security. B) the confirmation must conform to the requirements of SEC Rule 10b-10. C) a written confirmation of a transaction for a customer must be given or sent to such customer at or before completion of the transaction. D) a written confirmation of a transaction for a customer must be given or sent to such customer within one business day of the transaction.

a written confirmation of a transaction for a customer must be given or sent to such customer within one business day of the transaction. The delivery requirement for a confirmation of a customer transaction is at or before completion of a transaction. The completion of a transaction is the transaction's settlement date. With regular way settlement being T+2, the exception (the false statement) is within one business day of the transaction. SEC Rule 10b-10 lists all of the requirements for the contents of a customer confirmation including indicating the broker-dealer's status as a market maker if that is the case.

If a customer sells a zero coupon bond before maturity, gain or loss will be the difference between sales proceeds and

accreted value. Zero coupon bonds must be accreted for tax purposes. Each year, the annual accretion is taxable to the holder. In addition, the customer may adjust the cost basis of the zero upward by the amount of the annual accretion.

One of your customers calls for an explanation of the net amount due showing on a confirmation of a recent purchase transaction. The customer has multiplied the number of shares times the price per share, and that is lower than the amount the confirmation says is due. You would explain that the net amount is determined by

adding the commission and any firm-added fees to the principal amount. The net amount due on purchases is determined by adding expenses (commissions and fees added by the firm) to the principal amount (or subtracting them from the principal amount if a sale). FINRA has levied fines for mischaracterizing or disguising increased commissions by improperly describing them as handling or postage fees. Interest is added to the principal amount when bonds are traded with accrued interest. The question refers to shares, so the purchase must be of an equity security. A broker-dealer acting in a principal capacity buys securities from a customer with a markdown (and sells with a markup). **This question deals with material not covered in your LEM, but it relates to recent rule changes and/or student feedback.

At 3:55 pm ET, a registered representative receives a market order from an officer of XYZ to buy 75,000 shares of XYZ for the company's account. The registered representative must

advise the officer that a safe harbor under SEC Rule 10b-18 no longer exists before placing the order. Under SEC 10b-18, an issuer purchasing its own securities cannot affect the opening or closing of the security. A safe harbor is available if the issuer is not involved in the first transaction of the day or in any transaction in the last 30 minutes of trading (10 minutes if the security is actively traded). If the issuer were to purchase its own securities during the last 30 minutes of trading, it may be forced to justify that its purchase did not affect the closing price. If a registered representative receives an order from an issuer at 3:55 pm ET, he must advise the issuer that a safe harbor is not available. The representative may then place the order.

An accumulation unit in a variable annuity contract is

an accounting measure used to determine the contract owner's interest in the separate account. When money is deposited into the annuity, it is purchasing accumulation units.

A municipal finance professional (MFP) is

an associate of a broker-dealer engaged in municipal securities representative activities other than retail sales. An MFP is an associate of a broker-dealer engaged in municipal securities representative activities other than retail sales. Those activities can include the solicitation of municipal bond business. MFPs are subject to the MSRB reporting rules regarding gifts to elected officials and political parties (MSRB Rule G-37).

All of the following would be considered either retail communications or correspondence except

an email to several municipalities sent out in a single day offering your firm's services for underwriting their municipal securities. Communications with government entities, which includes municipalities, fall under the heading of institutional communications. The others are all examples of either retail communications or correspondence, depending on how many recipients there are within a 30-calendar-day period. (Retail equals more than 25 retail investors within any 30-calendar-day period, and correspondence is 25 or fewer retail investors within any 30-calendar-day period.)

Each of the following persons is accredited under Rule 501 of Regulation D except A) an officer of the issuer. B) an individual with a net worth of $200,000 or more. C) an individual with annual income of $200,000 or more in the past two years. D) an institution.

an individual with a net worth of $200,000 or more. Accredited investors include individuals with annual incomes of $200,000 or more; individuals with a net worth of $1 million or more, not including net equity in a primary residence; officers or directors of the issuer; and institutions.

A broker-dealer has set up a prime brokerage account for one of its customers. This customer is most likely

an institutional customer. A prime brokerage account is one in which a customer—generally an institutional customer—selects one member firm (the prime broker) to provide custody and other services, while other firms—called executing brokers—handle all trades placed by the customer.

Issuers of municipal securities are exempt from registration requirements under the Securities Act of 1933, but there is a full and fair disclosure document that is identified as

an official statement. The full and fair disclosure document for municipal securities is called the official statement.

You are asked to read the prospectus for a new issue of common stock for a client. You would expect the prospectus to include A) the FINRA disclaimer statement on the front cover. B) the date the registration statement was filed with the appropriate regulatory body. C) the legal opinion. D) an overview and history of the issuer's business and any risks associated with the offering.

an overview and history of the issuer's business and any risks associated with the offering. The prospectus will include an overview and history of the business, as well as any risks associated with it. It is the SEC disclaimer that is on the front cover and it is the effective date, not the date of filing that is shown. A legal opinion does not apply to common stock.

An individual who invests in an undeveloped land limited partnership would be most interested in

appreciation. Investors seek appreciation when investing in undeveloped land limited partnerships.

Municipal bonds—known as dollar bonds—are generally quoted A) net yield. B) yield to call. C) yield to maturity. D) as a percentage of par.

as a percentage of par. Although municipal bonds are usually quoted on a yield basis, actively traded bonds known as dollar bonds are often quoted as a percentage of par (price). The term dollar bond comes from the quote being made in dollars. Remember that a percentage of par value ($1,000) equals a dollar price.

Special tax bonds are

backed by sales, excise taxes, or both. A special tax bond is backed by one or more designated taxes (sales, cigarette, fuel, alcohol, etc.) other than ad valorem taxes. The designated tax need not be directly related to the project purpose. These bonds are not considered self-supporting debt.

Level 1 Nasdaq service provides subscribers with all of the following information except

bid and ask quotes for each market maker. Level 1 Nasdaq service provides subscribers with information on the inside market, last sale, and volume. The bid and ask quotes of each market maker in a particular security are shown over Level 2.

The Securities Act of 1933 covers all of the following except A) prospectus requirements. B) blue-sky laws. C) full and fair disclosure. D) liabilities for misleading filings.

blue-sky laws. The purpose of the Securities Act of 1933 is to provide investors with full disclosure about a new securities issue. The act is federal in scope, whereas blue-sky laws refer to state securities regulations.

In performing their natural job functions, all of the following may act in a principal capacity in a transaction with customers except A) designated market makers (DMM) on the NYSE. B) branch managers of FINRA member firms. C) OTC market makers. D) broker-dealers.

branch managers of FINRA member firms. When acting as a principal in a transaction, you are buying for or selling from inventory. Although branch managers of member firms are generally required to be registered as principals, that is different from acting in a principal capacity in a transaction. DMMs facilitate trading on the floor of the NYSE. Doing that sometimes requires them to buy or sell as principals. OTC market makers, by definition, act as principals. The term broker-dealer means that the firm can act either as a broker or a dealer. There are a number of words used in this exam than can have multiple meanings. Principal is one of those.

Without any position in the stock, an investor wrote an ABC Jul 60 call for 6. On the expiration date, ABC is selling for 66, and the investor closes the position at the intrinsic value. For tax purposes, the investor has

broken even. With the stock at 66, the call is 6 points in the money (call-up rule). That means the option will cost 6 points to close the position. The writer sold the option for 6 points and bought it back for 6 points. That results in breaking even.

If a U.S. corporation exports machine tools to Switzerland and will be paid in Swiss francs (SF), to protect against foreign-exchange risk, it should

buy SF puts. What is the concern of this corporation? If, when the Swiss company pays for the tools, the value of the SF has fallen against the U.S. dollar, the company will receive less money. For example, if the bill is for 100,000 Swiss francs and, at the time of the sale, that amount of SF is worth $110,000 (each SF is worth $1.10), a decline in the value of the SF to $1.05 means the exporter will receive $105,000 instead of the expected $110,000. To hedge against a decline value (regardless of the asset), we buy puts. In the case of exporters from the Unitred States, they buy puts on foreign currency. There are two important acronyms to remember, EPIC and IPEC. Referring to U.S. based companies, Exporters buy Puts and Importers buy Calls. Because there are no options on the U.S. dollar, the Swiss company should buy calls on its own currency to hedge In other words, EPIC works in reverse if the question is dealing with a foreign company. Then it is IPEC for Importers buy Puts and Exporters buy Calls.

An investor is short stock at $70. If the stock's market price is $40, and the investor anticipates the price will continue to decline, to hedge against a rise in the price, the investor should A) buy a straddle. B) buy a put. C) sell a call. D) buy a call.

buy a call. If the investor buys a call on the stock, he has the right to buy it back (cover his short) at a fixed price. The best way to hedge an unrealized gain on a short stock position is to buy a call.

An investor who buys a stock and wishes to limit the potential downside risk should

buy a put option on that stock. The purchase of a put option limits the downside risk to the difference between the stock price and the put's strike price. To that, the investor must add the cost of the "insurance" (the put). For example, consider an investor who purchases 100 shares of ABC stock at $55 per share and purchases one ABC 50 put for a premium of 2. If the stock price declines to $45 per share, the investor can put the 100 shares to the option writer and receive the $50 exercise price. Buying at $55 and selling at $50 is a loss of $5 per share ($500 on 100 shares). In addition, the investor paid $200 for the put so the maximum loss is $700. Expressed in math terms, the maximum loss is the purchase price of the stock minus the strike price of the put plus the premium ($55 minus $50 = $5 + $2 = $7 per share or $700). Because sell limit and buy stop orders are placed above the current market price, they don't hedge a decline in price. A sell stop order would work, but that is not a choice here.

The practice of dollar cost averaging requires the investor to

buy a security in a falling market and buy it in a rising market. Dollar cost averaging requires the investor to invest a fixed amount of money on a regular basis, regardless of whether the stock market is rising or falling.

You sell a municipal bond that has been advance refunded. It will be called at 102 four years from now. On the confirmation, the yield must be stated as the yield to

call. Municipal Securities Rulemaking Board rules require that, when a call date has been fixed by a prerefunding, the yield to call so fixed must be reflected on the confirmation statement. Because of the prerefunding, this bond issue will be called at the call date. There is no uncertainty surrounding this event. Therefore, it is appropriate to price the bond to the call date. The old maturity on the bond has no further significance.

Your client with a short call position in the S&P 100 index (OEX) is assigned an exercise notice. The obligation is fulfilled by delivering

cash equal to the difference between the closing value of the index and exercise price. Unlike equity options, index options are settled in cash only. Upon exercise, cash equal to the amount that the option is in the money (i.e., excess of market value over strike for a call or excess of strike over market value for a put) is delivered on the settlement date.

The IRS will generally consider a direct participation program to be an abusive tax shelter unless the program can show a profit motive. A popular method of measuring the economic viability of a DPP is A) cash flow analysis. B) passive loss analysis. C) income to debt analysis. D) the ratio of gains to losses

cash flow analysis. On the exam, there are two accepted measures of the economic viability of a DPP. Those are cash flow analysis and internal rate of return (IRR). Cash flow analysis compares the income to the expenses, not the debt.

A customer must present a signed representation letter stating that he is not a restricted purchaser before buying a new issue of

common stock. New issues of common stock may not be sold at the public offering price to any account in which a restricted person has a beneficial interest. Before buying an IPO, a customer must present a representation letter stating he is not a restricted person.

Services offered by prime brokers include all of the following except A) processing transactions. B) complying with FINRA's advertising rules. C) providing back office support. D) supplying clearing services.

complying with FINRA's advertising rules. Prime brokers provide services primarily to institutional investors. They have nothing to do with that institution's advertising. They do supply clearing services, lending services for marginable transactions, as well as back office support including cash management, account statements and transaction processing.

All of the following are advantages of investing in American depositary receipts (ADRs) except A) currency risk is virtually eliminated. B) dividends are received in U.S. currency. C) transactions are done in U.S. currency. D) ADRs fall under the oversight of the SEC.

currency risk is virtually eliminated. ADRs carry currency risk because distributions on ADRs must be converted from foreign currency to U.S. dollars on the date of distribution. In addition, the trading price of the ADR is affected by foreign currency fluctuation.

All the following information must be contained on an order ticket except A) type of account. B) customer's name. C) time of entry. D) customer account number

customer's name. The customer's name is not required on order tickets. The customer's account number, the type of account (e.g., cash or margin), and time stamps, however, are required.

A technical analyst is least concerned with A) new highs and lows. B) declaration of increased dividends. C) trading volume. D) open short positions.

declaration of increased dividends. A technical analyst is interested in statistics about market or price performance, not the fundamental factors, the market, or the company's dividend policy. Technical analysts are interested in trading volume as a market statistic, new highs and lows, and open short positions, which could indicate future buying potential in the security.

Market interest rates have been rising, which means the price of bonds traded in the secondary market has

decreased. When interest rates rise, bond prices fall.

If your client has a $21,000 net capital loss this year, and he plans to apply the maximum deduction toward his ordinary income for the year, after this year, he may

deduct a maximum of $3,000 per year and carry the remaining loss forward indefinitely. Capital losses may be used to offset capital gains. Once all capital gains have been offset, $3,000 of net capital losses may be used to offset ordinary income annually. Remaining losses may be carried forward in future years until the loss is exhausted.

Although there are general suitability rules that always apply, FINRA's Rule 2330 on variable annuity suitability specifies that, to be considered suitable, there is a reasonable basis to believe that the customer has been informed—in general terms—of various features of

deferred variable annuities. FINRA's primary suitability concern is with deferred variable annuities. That does not mean there are no requirements for being careful with the others, it is just that most of the violations have involved the deferred VA.

Common stockholders have certain voting rights. Those rights do not include voting on

determining the annual dividend rate. Common stockholders cannot determine the annual dividend rate. Only the board of directors can declare dividends. Common stockholders do have the right to vote for or against conversions, substantial corporate decisions, and the declaration of stock splits.

Shareholder approval is required for all of the following corporate events except A) the issuance of convertible bonds. B) stock splits. C) dividends. D) the acceptance of a tender offer from a nonaffiliated company.

dividends. Shareholder approval is not required for the payment of dividends but is normally required for actions that increase (or potentially increase) the number of shares outstanding, such as stock splits and the issuance of convertible bonds. A corporation's acceptance of a tender offer requires shareholder approval.

The Municipal Securities Rulemaking Board (MSRB) writes the rules in the municipal bond market. The MSRB

does not enforce its own rules. The MSRB writes the rules for the municipal bond market, but is not an enforcement authority. The rules are enforced by other regulatory bodies depending on who the bond dealer making the trade is. If it was completed by a community bank, the FDIC is the enforcement body. If it is done by a member bank of the Federal Reserve, the regulator is the Federal Reserve Board (FRB). If the trade is made through a FINRA member firm, then FINRA is the enforcer. The SEC has regulatory authority over the MSRB and all other self-regulatory organizations (SROs) not the IRS.

The Municipal Securities Rulemaking Board (MSRB) writes the rules in the municipal bond market. The MSRB

does not enforce its own rules. The MSRB writes the rules for the municipal bond market, but is not an enforcement authority. The rules are enforced by other regulatory bodies depending on who the bond dealer making the trade is. If it was completed by a community bank, the FDIC is the enforcement body. If it is done by a member bank of the Federal Reserve, the regulator is the Federal Reserve Board (FRB). If the trade is made through a FINRA member firm, then FINRA is the enforcer. The SEC has regulatory authority over the MSRB and all other self-regulatory organizations (SROs).

All of the following pieces of information must be obtained from new individual customers except

educational background. A customer's educational background is not required to open a new account. In the case of an account opened in a business's name, the business address and tax identification number are required.

Municipal securities broker's brokers

execute trades for other municipal securities broker-dealers. Broker's brokers are what the term implies. They represent other firms and use their connections to facilitate trades, usually with institutions. They never take an inventory position or get involved with underwriting new issues. One of their key roles is preserving the anonymity of their clients.

Investors in collateralized mortgage obligations (CMOs) tend to choose a tranche meeting their maturity expectations. Should the debt continue past the expected payoff date, it is an example of

extension risk. A CMO's yield and maturity are estimates based on historical data or projections of mortgage prepayments from the Public Securities Association (PSA). The particular tranche an investor owns determines the priority of their principal repayment. When the debt continues beyond the projected date, the investor has encountered extension risk. Prepayment risk is that the debt will be paid off early rather than later.

Each of the following choices are potential sources of funds associated with the backing of a revenue bond issue except A) ticket revenues. B) lease payments. C) concessions. D) fines.

fines Fines received by a municipality or a state may be used to service the debt of a general obligation bond. Often fines are levied for reasons that include parking tickets, traffic violations, and the late payments of taxes. Concessions and lease payments are sources of funds commonly associated with airport revenue bond issues. Ticket revenue is another term for user fees, as would be the case for the ridership revenue of a municipal bus service.

To be defined as a pattern day trader, the customer must execute at least

four day trades in five business days. FINRA defines a pattern day trader as one who executes 4 or more day trades in a 5-business-day period. That can stretch over two different weeks.

The SEC requires that all sell orders be identified as either long or short. A person is not considered to be long a security if he A) has purchased the security but the trade has not yet settled. B) has title to it. C) owns a security convertible into or exchangeable for the security and has tendered such security for conversion or exchange. D) has sold an in-the-money put option on that security simultaneously with entering the short sale order.

has sold an in-the-money put option on that security simultaneously with entering the short sale order. Selling a put only obligates the investor to buy the stock when the option is exercised. Merely writing the option does not result in possession of the stock. Having title to it does. Once purchased, the investor is long the stock even if the trade has not yet settled. Turning in a convertible security with conversion instruction has the same effect as entering a purchase order and waiting for settlement.

The SEC requires that all sell orders be identified as either long or short. A person is not considered to be long a security if he: A) has title to it. B) owns a security convertible into or exchangeable for the security and has tendered such security for conversion or exchange. C) has sold an in-the-money put option on that security simultaneously with entering the short sale order. D) has purchased the security but the trade has not yet settled.

has sold an in-the-money put option on that security simultaneously with entering the short sale order. Selling a put only obligates the investor to buy the stock when the option is exercised. Merely writing the option does not result in possession of the stock. Having title to it does. Once purchased, the investor is long the stock even if the trade has not yet settled. Turning in a convertible security with conversion instruction has the same effect as entering a purchase order and waiting for settlement.

One of the distinguishing differences between variable annuities and mutual funds is that variable annuities

have a separate account. The variable part of a variable annuity is the separate account. That is what makes it a security product similar to a mutual fund. The only guarantee with a variable annuity is that, once annuitized, payments will continue for life (or the designated period certain). The amount of those payments will vary based on the performance of the separate account. One of the negative features of variable annuities is that they typically have surrender charges that can last a decade or longer.

A new issue of common stock has been filed with the SEC, and a final prospectus can be found on the SEC website. This information has been given to a customer interested in the securities. In this instance, the access-equals-delivery requirements regarding that prospectus

have been met. A prospectus must precede or accompany a security for sale and will be deemed so if the final prospectus has been filed with the SEC. Because prospectuses filed with the SEC can be viewed on the SEC website, the access-equals-delivery requirement is satisfied.

If an investor who has owned FLB stock for two years buys 1 FLB Oct put, this will

have no effect on the holding period. The investor already held the stock long term when the put was acquired, so there is no effect on the holding period.

Both mutual funds and hedge funds are pooled investment vehicles managed by a professional money manager. As their registered representative, one of your customers asks you to explain how these investment vehicles are different. You inform them that

hedge funds are less liquid than mutual funds. Hedge funds typically have a lockup period during which time the investor cannot liquidate the investment. The capital is locked up. However, mutual funds are liquid. These are redeemable, so an investor can sell shares back to the issuing company at any time. While hedge fund managers can use aggressive investment strategies such as short selling, margin trading, and speculative derivative transactions, mutual fund managers cannot. The Investment Company Act of 1940 prohibits mutual fund managers from using these investment strategies

Using listed options to reduce the market risk in a stock position by taking an opposing position in the options that represent an equivalent number of shares is known as

hedging. This describes hedging, which is a technique used to reduce the market risk or adverse price movement in a stock position using options. Each standardized listed equity option contract represents 100 shares, so the number of contracts needed to hedge an existing stock position is determined by the number of shares the investor is currently long or short.

A customer buys 300 LMN at $45 per share and writes 3 LMN Aug 45 calls at 4. The customer will profit under all of the following circumstances except A) if LMN remains at $45 through expiration. B) if LMN rises, and the calls are exercised. C) if LMN is below $41 at expiration. D) if LMN is between $41 and $45 at expiration.

if LMN is below $41 at expiration. Breakeven is $41 ($45 − $4). The stock price must be above breakeven for the investor to make a profit.

Communications with the public include all of the following except A) informational material on a new mutual fund intended for sales personnel. B) television appearances by an officer of the firm. C) independently prepared reprints forwarded to your firm's customers. D) institutional sales material.

informational material on a new mutual fund intended for sales personnel. Material intended for internal use only is not considered a communication with the public.

You are at a social gathering speaking with an individual who is a tenured professor of astrophysics at the state university. She mentions that she participates in the school's TSA plan. That means she

is participating in a retirement plan likely offering tax sheltered annuities. TSA stands for tax-sheltered annuity and is the most common name for the 403(b) retirement plan. Although investments can be made into mutual funds, some 85% of the funding is through annuities.

Reasonable-basis suitability, as used in FINRA Rule 2111 means the member or associated person making the recommendation should have a reasonable basis to believe that the recommendation

is suitable for at least some investors. The reasonable-basis obligation requires a member or associated person to have a reasonable basis to believe, based on reasonable diligence, that the recommendation is suitable for at least some investors.

FINRA's 5% markup policy does not apply to

issues sold by prospectus. FINRA's 5% markup policy applies to all secondary market trades, whether customers are charged markups, markdowns, or commissions. Issues sold by prospectus and municipal securities, however, are exempt from the policy.

One of your customers has decided to commit $10,000 to fixed income. She is trying to decide if it makes more sense to invest in the bonds of a single corporate issuer or to buy an exchange-traded fund (ETF) tracking a corporate bond index. You could explain that the purchase of the ETF results in the greatest reduction of

liquidity risk. In general, the bond market has some liquidity risk because bonds, especially in quantities as low as $10,000, do not have the trading activity we see with listed securities. For ETFs, trading on an exchange significantly lessens their liquidity risk. If the ETF contains bonds with maturities similar to the individual bonds she is considering, the interest rate risk and inflation risk are essentially the same. Currency risk applies only when foreign securities are involved.

A 27-year-old client is in the lowest tax bracket and seeks an aggressive long-term growth investment. If his investment adviser representative recommends a high-rated general obligation municipal bond, the investment adviser representative (IAR) has

made an unsuitable recommendation based on the client's needs and objectives. In recommending a conservative, tax-exempt investment to this customer, the IAR has failed to make a suitable recommendation given the client's objectives. Municipal bonds are better suited for individuals in high tax brackets and offer little upside appreciation potential.

The general partner of an oil and gas drilling program might be considered to have a conflict of interest for all of the following reasons except A) commingling the funds of several different programs. B) managing the partnership. C) borrowing funds from the partnership. D) owning mineral rights on undeveloped adjacent land.

managing the partnership. Managing the partnership is what the general partner's job is, so there would not be a conflict of interest. Owning a potentially competing project could be a conflict of interest. Commingling (combining) investor funds from different programs is not legal and would certainly present a conflict, as would borrowing money from the partnership.

All of the following statements about trading index options on the Chicago Board Options Exchange are true except A) limit orders are maintained in an order official's book. B) market makers may trade for their own accounts. C) floor brokers may execute orders for others on a commission basis. D) market orders entered by a market maker have priority over public orders.

market orders entered by a market maker have priority over public orders. Public orders must be filled before member orders. This is true if it is stock, bonds, and options of any kind.

In performing the role required on the NYSE, a designated market maker (DMM)

may act as a broker or a dealer to facilitate trading on the floor. DMMs facilitate trading in specific stocks on the floor of the NYSE. Their chief function is to maintain a fair and orderly market in those stocks. In fulfilling this function, they act as both brokers and dealers. They act as dealers when they execute trades for their own accounts and as brokers when they execute orders other members leave with them. Floor brokers work for member firms. They are the people who bring to the DMM the orders their firm's clients have placed.

A customer wishes to open a new account but refuses to provide suitability information. Under FINRA rules, the member

may open the account, but any recommendations must be limited to suitability information the firm has on the customer. A recommendation may be made if the firm has a reasonable basis to believe it is suitable. This can be based on information that the firm knows about the customer. For example, you do know the customer's age and occupation. You do have the customer's home address. In the real world, firms rarely rely solely on this, but for test world purposes, limited recommendations may be made.

When compared to statutory voting, cumulative voting gives an advantage to

minority stockholders. Cumulative voting allows shareholders to aggregate their votes and cast them as they please. For example, they could cast all of their votes for a single candidate. Cumulative voting makes it easier for a minority group of shareholders to gain representation on the board.

A call is in the money when the market price of the underlying stock is

more than the strike price. Call options are in the money whenever the market price of the stock is greater than the strike price.

A separate account will invest in a number of different securities. The separate account is not likely to invest in

municipal bonds. The earnings on dollars invested into a variable annuity accumulate tax deferred, which is why variable annuities are popular products for retirement accumulation. As with all tax-deferred accounts, municipal bonds are not appropriate investments because interest earned on municipals is already tax exempt at the federal level.

Guarantees on insurance products

must be specific to the insurance contract and not associated investments. Guarantees are only permissible for the specific guarantees within an insurance contract. Companies cannot guarantee returns or even principal values in investments, including separate accounts. While high ratings may indicate stability and quality in a company, they cannot be used to make a guarantee.

Regulation SHO prohibits

naked short selling. Regulation SHO mandates a locate requirement with regard to short sales. Before entering a short sale order, members are required to locate the security to be ensured that delivery can be made on settlement date. The locate requirement applies to short sales in all equity securities. Failure to positively locate the stock before making the short sale is the prohibited practice of naked short selling. Although one can sell short only in a margin account, the prohibition against doing so in a cash account is part of Regulation T, not SHO. Short selling, just as with any activity in customer's accounts, must be suitable. That is part of the suitability requirements, not Regulation SHO. Short sales can be made on any tick: up, down, or the same.

Stock prices in the over-the-counter (OTC) market are determined by

negotiation between buyers and sellers. The OTC market is a negotiated market (not an auction market as is the case with an exchange) in which dealers negotiate stock trades with each other.

A client has 100 shares of GHI when the stock undergoes a split. After the split, the client has

no effective change in the value of the position. When a stock splits, the number of shares each stockholder has either increases or decreases (in the case of a reverse split). The customer experiences no effective change in position because the proportionate interest in the company remains the same.

The Profligate Perpetual Growth Fund's prospectus states that the fund meets the Investment Company Act of 1940's qualifications of a diversified management company. If the fund has net assets of $1 billion,

no more than $300 million can be invested in the voting shares of any single issuer. no more than $300 million can be invested in the voting shares of any single issuer.

The locate requirement of Regulation SHO for short sales does not apply to A) over-the-counter equity securities. B) preferred stock traded on the NYSE. C) American depositary receipts traded on the Nasdaq Stock Market. D) nonconvertible bonds traded on the NYSE.

nonconvertible bonds traded on the NYSE. The locate requirement is applicable to all short sales of equity securities. It is unlikely to be tested, but, just in case, for purposes of this rule, convertible bonds are considered equity securities.

If a registered options principal is asked to approve a discretionary order to buy 1 XYZ Oct 60 put and sell 1 XYZ Oct 55 put for a net debit of $5, he should

not approve the order. Because this is a debit spread, the maximum gain occurs if both sides are exercised. If this occurs, the investor earns $5 (buy stock at 55 when the short put is exercised and sell stock at 60 by exercising the long put). Because the net premium paid for the spread is $5, there can never be any gain. This spread is not economical.

A municipality is allocating the revenues from an industrial revenue bond under a net revenue pledge. The first priority is

operation and maintenance. Under a net revenue pledge, operations and maintenance are paid first, with debt service following. In a gross revenue pledge, debt service is paid before operations and maintenance.

Advertisements for the Abstemious Balanced Fund (ABF) describe the investment as a no-load fund. In order to make this claim, the fund must

not have a conditional deferred sales charge When a fund promotes itself as a no-load fund, not only must there be no front-end load, there cannot be a back-end load (CDSC) either. The 12b-1 charge maximum is 0.25%. The concept of breakpoints applies solely to Class A shares (front-end load).

All of the following deal with the secondary market except A) dealer quotes. B) Thomson's Muni Market Monitor (formerly Munifacts). C) broker's broker. D) notice of sale.

notice of sale. A notice of sale is published to provide syndicates with information on proposed new (primary market) issues. Dealers are selling out of inventory (secondary trading). A broker's broker executes trades in municipal securities for or on behalf of another Municipal Securities Rulemaking Board member firm. Transactions by a broker's broker could be in both the primary and secondary markets.

One of your clients owns 300 shares of common stock in a publicly traded corporation. The acquisition cost of those shares was $60,000 and the last trade of the stock was $220 per share. There was a news report that the company was going to pay shareholders a 100% stock dividend. The client wants to know how this dividend will affect the holding. You would respond that the customer will

now own 600 shares and the market price will be approximately $110 per share. The effect of a 100% stock dividend is the same as a 2:1 stock split. The customer will have twice as many shares worth half as much each. That would be 600 shares worth $110 per share for a total value of $66,000. Note that the total value is unchanged from the pre-split value of 300 shares at $220 per share.

If a 50% stock dividend is declared, the owner of 1 XYZ Jul 30 call owns

one contract for 150 shares with an exercise price of 20. When a company pays a stock dividend or effects a fractional stock split, the number of contracts remains the same, but each contract is adjusted by increasing the number of shares the contract covers (100 × 150% = 150 shares). The effective strike price is adjusted so that the position value remains the same before and after the adjustment—in this case, 20 ($3,000 / 150 shares = $20).

Last-sale information is always available for all of the following securities except A) NYSE listed. B) over the counter (OTC), non-Nasdaq. C) CBOE listed option contracts. D) Nasdaq.

over the counter (OTC), non-Nasdaq. Last-sale information is available for listed (exchange-traded) securities and for all Nasdaq securities. While there are a number of sources for last-sale information in general, it may not always be available for a security that is OTC non-Nasdaq.

To qualify for favorable tax treatment, real estate investment trusts (REITs) must do all of the following except A) be organized as trusts. B) invest at least 75% of their assets in real estate-related activities. C) pass through losses to shareholders. D) distribute at least 90% of their investment income to shareholders.

pass through losses to shareholders. REITs engage in real estate activities and can qualify for favorable tax treatment if they invest at least 75% of their assets in real estate-related activities and pass through at least 90% of their net investment income to their shareholders. Although they can pass through income, they cannot pass through any losses.

Inverse exchange-traded funds, also known as reverse or short funds, are managed to

perform contrary to a benchmark market index such as the S&P 500. Inverse funds, which are also commonly referred to as short funds, try to deliver returns that are the opposite of the benchmark index they are tracking. When they are exchange traded, they can be bought on margin and are priced throughout the trading day like other exchange-traded products. They are available to individual public investors and firm proprietary traders as well.

All of the following would be found in the money market except A) preferred stock. B) commercial paper. C) Treasury bills. D) repurchase agreements.

preferred stock. The money market consists of short-term, high-quality debt instruments. This would not include preferred stock, which is an equity instrument.

All of the following are redeemable securities except

real estate investment trusts (REITs). A redeemable security has no secondary market. To sell (redeem) a redeemable security, the investor must go back to the issuer or its agent. REITs trade in the secondary markets either on exchanges or over the counter.

Programs allowing for the direct pass-through of losses and income to investors include all of the following except A) real estate investment trusts (REITs). B) oil and gas drilling direct participation programs. C) new-construction real estate direct participation programs. D) S corporations.

real estate investment trusts (REITs). REITs allow for the direct pass-through of income, but not losses. The other choices are forms of business that allow for pass-through of income and losses.

All of the following may be used to service special tax bond issues except A) business license taxes. B) gasoline taxes. C) real estate taxes. D) excise taxes

real estate taxes. Special tax bonds are sometimes included in the larger and more general category of revenue bonds. Bonds supported from the proceeds of specified income generators, such as gasoline, cigarettes, liquor, and business licenses, are special tax bonds. Ad valorem (real estate) taxes never service special tax bonds.

The provisions of the Securities Act of 1933 include all of the following except A) prohibition of fraud in the sale of new securities. B) regulation of the secondary market. C) regulation of offerings of new securities. D) the requirement that an issuer provide full and fair disclosure about an offering.

regulation of the secondary market. The Securities Act of 1933 regulates new issues of corporate securities sold to the public and is designed to prevent fraud in the sale of newly issued securities. Trading and the secondary markets are regulated under the Securities Exchange Act of 1934.

If interest rates are dropping, an investor with a maturing bond will be most concerned with

reinvestment risk. When interest rates decrease, investors with maturing bonds will have to accept lesser yields to replace their investments. This is often referred to as a form of reinvestment risk.

All the following retail communications must be prefiled with FINRA except A) retail communications concerning investment companies with custom ratings. B) retail communications concerning options without previously providing an ODD. C) retail communications concerning public DPPs. D) retail communications concerning the member firm's opening for business last month.

retail communications concerning public DPPs. Retail communications concerning public DPPs do not need to be prefiled. Retail communications for all new member firms, concerning investment companies with custom ratings or options without previously providing an ODD must be prefiled with FINRA. New member firms, defined as being in their first year of business, must file retail communications with FINRA 10 business days in advance of use.

All the following retail communications must be prefiled with FINRA except A) retail communications concerning investment companies with custom ratings. B) retail communications concerning options without previously providing an ODD. C) retail communications concerning the member firm's opening for business last month. D) retail communications concerning public DPPs.

retail communications concerning public DPPs. Retail communications concerning public DPPs do not need to be prefiled. Retail communications for all new member firms, concerning investment companies with custom ratings or options without previously providing an ODD must be prefiled with FINRA. New member firms, defined as being in their first year of business, must file retail communications with FINRA 10 business days in advance of use.

When a FINRA member firm has been designated as a "taping" firm, it is required to

retain copies of all mandated recordings for a period of three years from the date the tape was created. FINRA Rule 3170 (Tape Recording of Registered Persons by Certain Firms)—commonly referred to as the taping rule—requires certain firms to install taping systems to record all telephone conversations between their registered persons and existing and potential customers, review those recordings, and file reports with FINRA. All tape recordings made pursuant to the requirements of the rule are to be retained for a period of not less than three years from the date the tape was created, the first two years in an easily accessible place. ** This question deals with material not covered in your LEM, but it relates to recent rule changes and/or student feedback.

A registered representative is preparing a PowerPoint slide presentation, to be delivered in a live seminar, for a group of invited institutional clients. To use the slides, they may have to be

reviewed by a principal of the broker-dealer. Communications material that is intended for use with institutional customers only need be supervised and reviewed by a principal of the member firm. Alternatively, if the member's procedures do not require review of institutional communications, they must include a provision for the education and training of associated persons so that they will understand the firm's requirements. Though FINRA can request spot checks of any material used to communicate with the public, submission of institutional communications to FINRA or the SEC for review or approval is not required.

Reasons to suggest that your customers name a beneficiary for their accounts would include all of the following except A) ensuring the property goes to the proper person. B) avoiding probate. C) saving income taxes. D) the assets generally get into the hand of the heir(s) without delay.

saving income taxes. While naming a beneficiary may save on estate taxes by using the marital deduction (not tested), passing property through death does not incur income taxes. With a named beneficiary, probate can be avoided and, of course, it limits the disputes on how the property is to be divided when there are multiple heirs. Another benefit is the relatively short time it takes for the assets to be transferred to the heir or heirs.

Information found in The Bond Buyer would include all of the following except A) secondary market volume. B) Revdex. C) the placement ratio. D) the 30-day visible supply

secondary market volume. The Bond Buyer is a source of information for new (primary market) municipal bond issues. It contains Revdex, an index for revenue bonds, as well as general obligation bond indexes. Additionally, it includes the 30-day visible supply and the placement ratio.

A customer sells an FLB Mar 35 call. To establish a straddle, she would

sell an FLB Mar 35 put. Straddles involve options of different types, but both options must be of the same series. An option series has the same strike price, expiration date, and underlying security.

Once the IRS determines that a tax shelter is abusive, it may do all of the following except

sentence the abuser to a prison sentence. The IRS does not have the authority to hand out prison sentences. It can bring you to court, and the court can sentence you to jail.

Once the IRS determines that a tax shelter is abusive, it may do all of the following except A) charge interest on back taxes. B) sentence the abuser to a prison sentence. C) disallow all deductions. D) charge the taxpayer with intent to defraud.

sentence the abuser to a prison sentence. The IRS does not have the authority to hand out prison sentences. It can bring you to court, and the court can sentence you to jail.

For the purpose of reporting sales to the IRS, the method available to investors by the IRS that offers the most flexibility in anticipation of the investor's year-end tax needs is

share identification. Share identification is the most flexible of the three methods. The investor keeps track of the cost of each share purchased and specifies which shares to sell based on his anticipated year-end tax needs. For investors, the idea is to minimize tax liability, if possible, by limiting gains or maximizing loses in anticipation of what one's year-end tax liability might be.

Depletion allowances in oil and gas programs are based on the amount of oil

sold. Depletion allowances are allowed to compensate for a mineral resource, which is considered accomplished when it is sold.

When a bond is issued by a national government, it is referred to as

sovereign debt. The term sovereign debt applies to securities issued by national governments. U.S. Treasuries are an example of sovereign debt issued here. Other countries have their versions, such as the Gilts of the United Kingdom. These are not considered alternative investments. Alternative investments are structured products that are complex and not easy to understand. Two of the most popular structured products are the ELNs and the ETNs.

A technical analyst would be most interested in

support levels. Technical analysts care about price and volume trends in the marketplace, such as support levels. A corporation's earnings, P/E ratio, and current ratio would be of most interest to a fundamental analyst who reviews a company's financial statements in more detail.

A producer of fine French wines has just signed a contract to export $10 million of wine to a distributor in the United States. Using listed foreign currency options, this producer would have the best protection against currency risk by

taking a long position in euro calls. There are no listed options in the U.S. dollar. That reduces your choice to a long euro put or call. Because the contract will be paid for in dollars, the producer is concerned that the dollar will fall against the euro. Or, stated another way, the concern is that the euro will rise against the dollar so that the $10 million will not buy as many euros as on the day the contract was signed. When one is afraid the price of asset will rise, such as those who take a short position in a stock, the best protection is buying a call. The easiest way to remember this is through the acronym, IPEC - Importers buy Puts and Exporters buy Calls. This is used when the party involved in the question is in a foreign country. Because we are dealing with a French exporter, buying calls on the local currency offers the best protection.

A collateralized mortgage obligation (CMO) makes an interest-only payment to an investor. This payment will be

taxed as ordinary income. Interest-only payments made by CMOs are taxed as ordinary income.

All of the following are true of stockholders' equity except A) that it consists of stock issued, capital surplus, and retained earnings. B) that it is reflected in the book value of the stock. C) that it is carried as an asset on the balance sheet. D) that it is also called net worth.

that it is carried as an asset on the balance sheet. Stockholders' equity or net worth (total assets less liabilities) is what a stockholder is entitled to should a company liquidate.

J.B. Collingsworth is the CEO and largest single shareholder in Collingsworth Industries, Inc. (CII). Three months ago, J.B. purchased 2,000 shares of CII in the secondary market at $6 per share. Yesterday, J.B. sold those shares for $8 per share. Under federal law,

the $4,000 profit on the sale must be returned to CII. J.B. has violated the "short swing" profit rule of the SEC. That rule states that any affiliate of a public company realizing a profit on a sale of that company's stock within a six-month period of the purchase, must return that profit to the company. The term used is "disgorged." There is nothing illegal about this, but J.B. doesn't get to keep the profit. The sale is well under the Form 144 de minimis limit of 5,000 shares or $50,000 in market value. Because the shares were purchased in the secondary market, they are already registered. Nothing in the question implies that J.B. was acting on inside information.

New Housing Authority (NHA) bonds are a relatively safe investment because

the U.S. government guarantees a contribution to secure the bonds. NHAs are considered safe because, in addition to the backing of rental income, they are secured by a subsidy from an agency of the U.S. government. Thus, they are rated AAA.

A walk-in customer completes the new account form and includes all of the information required by the customer identification program. However, the customer supplies none of the requested financial data and is unwilling to discuss objectives. Under Regulation BI of the SEC,

the account may be opened, but no recommendations may be made. Regulation BI deals with recommendations from a broker-dealers to its customers. As long as there are no recommendations, the rule does not apply. When the customer refuses to supply the necessary suitability information, the account may be opened, but trading must be limited to unsolicited orders. ** This question deals with material not covered in your LEM, but it relates to recent rule changes and/or student feedback.

An analyst interested in measuring the breadth of market movement as an indicator of future market direction would monitor

the advance/decline line. The advance/decline line, which measures the number of stocks that have advanced versus the number of stocks that have declined, is an indicator of the breadth of the market's advance or decline.

A municipal revenue issue's flow of funds statement is contained in

the bond contract. The bond contract describes the nature of the contract and the issuers' duties to bondholders. The bond contract is a more expansive document than a bond resolution. The contract is comprised of the bond resolution (or trust indenture) and other security agreements and laws in force at the time of bond issuance.

The call provisions of a municipal issue would be detailed most completely in

the bond resolution. The bond resolution is the document that authorizes the issuance of a municipal bond. The resolution also describes the proposed issue's features and the issuer's responsibilities to its bondholders.

All of the following would be included in a penny stock risk disclosure statement except A) the risks of investing in penny stock. B) the definition of penny stock. C) the broker-dealer's statement of guarantee. D) investors' legal rights.

the broker-dealer's statement of guarantee. Penny stock disclosure statements must be furnished to all buyers of unlisted, non-Nasdaq stocks of less than $5 per share. The disclosure must include the risks of penny stock investing, the rights of the investors, and the responsibilities of the broker-dealer to the investor. There would be no statements—either implied or expressly written—regarding guarantees.

The document attesting to the formation of a limited partnership, filed with designated authorities, is called

the certificate of limited partnership. The Uniform Limited Partnership Act requires that two or more persons sign and swear to a certificate of limited partnership. It is filed with the state and is a public document available for review.

PDQ Corporation has a 6.25% $100 par value convertible preferred stock (conversion ratio of 4) outstanding. The stock has an antidilution covenant. If PDQ declares a 10% stock dividend, the antidilution covenant will adjust

the conversion price to approximately $22.73. When a $100 par value preferred stock is convertible into four shares of common stock, the conversion price is $25 per share, ($100 ÷ 4 = $25). The antidilution covenant means the investors will have the same conversion rights after a stock split or stock dividend as they had before. After a 10% stock dividend, each share of preferred stock will be convertible into 4.4 shares (4 shares x 110% = 4.4). The par value of the preferred stock does not change. Divide that $100 par value by the new number of shares to get the new conversion price. It looks like this: $100 ÷ 4.4 = 22.73. Alternatively, you can divide the original conversion price of $25 by 110% arrive at the same answer.

All of the following have an impact on the marketability of a block of municipal bonds except A) the price and date of call provisions. B) the length of time until the bonds mature. C) the quantity and quality of the bonds in the block available. D) the dated date of the bonds in the block.

the dated date of the bonds in the block. The dated date has no effect on marketability. A close call date or low call premium can make an issue less marketable because the chance of a call is greater. Maturity, quality, and the size of the block affect marketability.

The date on which interest will begin accruing on a new municipal issue is

the dated date. New issues of municipal bonds begin accruing interest as of the dated date.

A city has issued bonds to construct a new sewage treatment facility. If the bonds are not backed by the full taxing authority of the city, all of the following statements about the bond issue are true except A) there is no debt limitation on the issue. B) the interest on these bonds is not considered a preference item for the alternative minimum tax. C) the disbursement of principal and interest payments must be approved semiannually by the state public service commission. D) if earnings fall short of the amount needed to make principal and interest payments, the debt service reserve can be used.

the disbursement of principal and interest payments must be approved semiannually by the state public service commission. As an exclusion question, we are looking for the false statement. The public service commission would have no approval power over revenue bond interest and principal payments. Because the bond is not backed by the taxing authority of the city, it is a revenue bond rather than a general obligation bond. The funds for payment of interest and repayment of principal are generated through the fees paid by those using the city's water and sewage facilities. Being a public, rather than private, facility, these would not be alternative minimum tax bonds.

A corporation's income statement reports net income of $10 million for the year. The company has one million shares of 4% $50 par value preferred stock and two million shares of common stock. If the corporation paid a quarterly dividend of $0.60 per share of common stock,

the dividend payout ratio was 60%. The dividend payout ratio is the percentage of the net income (after preferred stock dividends) paid out to the common shareholders. The net income is $10 million. The preferred dividend is $2 per share ($50 par times 4% = $2). With one million shares, the total preferred dividend is $2 million (1 million shares at $2 per share). Because the preferred dividend must be paid before any earnings are available to common stockholders, we subtract that $2 million from the net income. That leaves $8 million in earnings available to common. There are 2 million shares receiving an annual dividend of $2.40 ($0.60 quarterly). That means $4.8 million of the $8 million available is paid, or a ratio of 60% ($4.8 million ÷ $8 million = 60%). Or, the earnings per share is $4.00 ($8 million divided by 2 million shares) and $2.40 in dividends paid out of $4.00 earnings made is 60%. The preferred stock is paying a dividend of 4% of the par value, but that does not tell us the current yield. To know the current yield, we must know the current market price of the stock and the question does not supply that value.

In a customer's margin account, a broker-dealer must segregate

the excess securities above 140% of the accounts debit balance. A broker-dealer may hypothecate (pledge) 140% of a customer's debit balance. Any customer securities in excess of 140% of the debit balance must be physically segregated.

It would be correct to describe a warrant in all of the following ways except A) the exercise price is generally slightly below the current market price. B) warrants do not have voting rights. C) it may be used as a sweetener for a bond issue. D) it has a longer life than a stock right.

the exercise price is generally slightly below the current market price. Warrants always carry an exercise price that is above the current market price of the underlying security. The upside potential is viewed as a sweetener for other issues, particularly bonds. This results in the issuer borrowing at a lower interest cost. Unlike preemptive (stock) rights that have a short life, warrants do not expire for a long time, sometimes five years or longer.

A 3× leveraged inverse exchange-traded fund means

the goal of the fund is to have the shares go up by three times the amount that the targeted index falls. A leveraged inverse fund attempts to produce returns opposite of the targeted index in the amount designated by the leverage multiplier—in this case, three times. If the targeted index rose, the inverse fund would fall by the leverage multiplier, and in that regard, substantial risk must be recognized due to the leverage. Leveraged funds will use derivative products like options and futures to help meet the desired goal, and they can use investment strategies such as margin to purchase securities for the fund portfolio as well. Lastly, because the fund shares are exchange traded, they can be purchased on margin by investors.

A moral obligation bond is one where

the governing legislature is empowered, but not obligated to appropriate funds to prevent the bond from going into default. The MSRB defines a moral obligation bond as: "A bond that, in addition to its primary source of security, is also secured by non-binding covenant that any amount necessary to make up any deficiency in debt service will be included in the budget recommendation made to the governing body, which may appropriate funds to make up the shortfall. The governing body, however, is not legally obligated to make such an appropriation."

In a margin account, the broker-dealer lends money to the customer to assist in the purchase of a marginable security. Instead of delivering the security to the purchaser, the broker-dealer holds it as collateral for the loan. The form signed by the customer agreeing to this is

the hypothecation agreement. There are three special margin account agreement forms. The hypothecation agreement is the one in which the customer agrees to allow the broker-dealer to keep the securities purchased as collateral for the margin loan. The credit agreement contains the terms of the loan, such as interest to be charged, and the loan consent agreement is an optional form agreeing to let the broker-dealer lend out those securities.

A municipal firm acts in an agency capacity to execute a purchase order for a customer. All of the following are required to be on the customer's confirmation of purchase except A) the identity of the contra broker. B) the settlement date. C) a description of the securities. D) the capacity of the broker-dealer.

the identity of the contra broker. The identity of the contra party (the other side of the trade) is not necessary, as long as it is stated that the name is available upon request.

The bond resolution includes all covenants between

the issuer and the trustee acting for the bondholders. The bond resolution describes not only the characteristics of the proposed offering, but also the obligations the issuer has to its bondholders.

The unqualified legal opinion on a municipal bond states that

the issuer has the authority to issue bonds that are legal, valid, and enforceable obligations of the issuer. Bond counsel attests that, to the best of its knowledge, the issuer has the legal right to issue the securities in question. In the case of tax-exempt bonds, the interest the issuer will pay on the bonds is exempt from federal taxation, and the bonds are exempt from federal registration requirements. The legal opinion does not go to the issue's marketability—or safety—debt service requirements.

An investor in an equipment-leasing direct participation program (DPP) using straight-line depreciation would probably not be concerned about A) legislative risk. B) the likelihood of recapture. C) the quality of the management. D) liquidity risk.

the likelihood of recapture. Recapture of deductions is a concern when accelerated, but not when straight-line depreciation is used. In any business, there is always concern about the quality of the management. By and large, DPPs are not liquid investments, so an investor needing a quick sale may have problems. The nature of DPPs tends to make them more sensitive to legislative risk than most other securities.

All of the following are risks of investing in publicly traded mortgage-backed securities except A) falling interest rates might accelerate early repayment of principal. B) borrowers might default on their mortgage payments. C) the market for mortgage-backed securities is illiquid. D) rising interest rates might extend the date of repayment of principal.

the market for mortgage-backed securities is illiquid. As publicly traded securities, liquidity risk is not a major concern to investors in mortgage-backed securities. When interest rates decline, there is generally an increase in mortgage refinancing and that results in investors receiving repayment of principal ahead of schedule. That is prepayment risk. Although it is nice to get the money back, there is reinvestment risk because these lower interest rates mean that reinvesting the principal into new securities will now be at a lower return than previously earned. The opposite happens when interest rates increase. Homeowners are unlikely to refinance, causing principal repayments to slow. Without the repayment, investors holding these securities are receiving below market returns. This is an example of extension risk. As is the case with any loan, there is always default risk. Recent history has shown that it is possible for a large number of foreclosures where the lender does not recoup the full principal.

All of the following statements regarding municipal revenue bonds are true except: A) the interest and principal are paid from revenue received from the facility. B) revenue bonds can be issued by inter- or intrastate authorities. C) the maturity of the revenue bond will usually exceed the useful life of the facility being built. D) no debt limitation is set by the issuing municipality.

the maturity of the revenue bond will usually exceed the useful life of the facility being built. Revenue bonds are usually structured so their maturity is shorter than that of the facility they were issued to build.

All of the following must be considered by an investment adviser representative before recommending a municipal general obligation (GO) bond to a customer except

the municipality's coverage ratio. The coverage ratio is specific to revenue bonds only and tells how many times annual revenue from that issue will cover the debt service of the issue. It is not a factor of suitability to be considered when recommending a GO bond, but more of a factor to consider when comparing two municipal revenue bonds. The customer's state of residence and tax status are essential when determining suitability for a municipal security. The security's rating is also important because it measures the overall safety and quality of the bond.

When opening a new account for an individual investor, FINRA asks its member to make a reasonable effort to obtain certain information about the account. Included information would be all of the following except

the name(s) of the customer's dependents. Nowhere in the FINRA rules on account opening does it require or suggest obtaining personal information about family members. That information becomes important when we look at the suitability rules. Please note that the tax ID or Social Security number is required under the customer identification program (CIP), but not FINRA rules.

When opening a new account for an individual investor, FINRA asks its member to make a reasonable effort to obtain certain information about the account. Included information would be all of the following except A) the customer's tax identification or Social Security number. B) the name(s) of the customer's dependents. C) whether the customer is an associated person of another member. D) the occupation of the customer and name and address of the employer.

the name(s) of the customer's dependents. Nowhere in the FINRA rules on account opening does it require or suggest obtaining personal information about family members. That information becomes important when we look at the suitability rules. Please note that the tax ID or Social Security number is required under the customer identification program (CIP), but not FINRA rules.

Balance sheets contain

the net worth of the firm as of the date of the balance sheet. The balance sheet provides a snapshot of the financial condition of the firm on that date. It does not provide information on the flow of expenses, revenues, and cash during the reporting period, as they would appear on the income statement. Among other references will be if inventory was calculated using first-in-first-out (FIFO) or last-in-first out (LIFO).

The predictions of the VIX Index reflect expected change over

the next 30 days. The current VIX Index value quotes the expected annualized change in the S&P 500 Index over the upcoming 30 days, as computed from options-based theory and current options-market data.

When a bond is selling at a discount

the nominal yield will always be lower than the current yield. When a bond is selling at a discount, all of the yields are higher than the nominal (coupon) yield. The sequence in ascending order of yield is NY, CY, YTM, YTC.

An option confirmation must include all of the following except A) the strike price. B) the type of option and commission. C) the positions market attitude (bullish or bearish). D) the number of contracts and premium.

the positions market attitude (bullish or bearish). An option transaction's confirmation must completely recap the essential elements of the trade, including the number of contracts traded, the description of the security traded (type of option and strike price), the price of the security traded (premium), and any commission charges. The positions market attitude (bullish or bearish) is not included on the confirmation.

The Securities Act of 1933 applies to all of the following except A) full and fair disclosure. B) prospectus preparation. C) the regulation of insider trading. D) the registration of new issues.

the regulation of insider trading. The regulation of insider trading is covered under the Securities Exchange Act of 1934. The 1933 act deals with new issues and related disclosures.

When a corporation completes a stock split, it will not have any impact on

the retained earnings. When a stock splits, the par value splits accordingly. Obviously, the number of shares outstanding increases. Because there are now more shares outstanding, there is a reduction to the earnings per share. This split will have no effect on retained earnings because there have been no monetary changes to assets or liabilities.

Under the Uniform Practice Code, regular way transactions for common stock settle on

the second business day following the trade date. Under the Uniform Practice Code, regular way trades settle two business days after the trade date (T+2). Cash settlement for a trade occurs on the same day as the trade date, and Regulation T payment is due S+2, which is T+4 in a regular way equity trade.

One of your customers owns a variable annuity. When asking about how the performance of the separate account is measured, you would respond that

the separate account performance depends on the performance of the selected subaccounts. Some insurance company separate accounts have dozens of different subaccounts. These subaccounts range from highly aggressive to highly conservative. It is the performance of the specific subaccounts selected by the investor that determines the value of their accumulation unit. The actuaries get involved when determining the payout because that depends on life expectancy. Similarly, the AIR comes into play only during the payout phase.

Members of a syndicate receive notice of their share of the offering through

the syndicate letter. The syndicate letter is sent by a municipal dealer to prospective members inviting them to join the syndicate and setting forth the conditions of the syndicate. Such conditions include who the manager will be, the percentage participation (each member's share), and the amount of good faith deposit required.

All of the following might affect the credit rating of a municipal revenue issue except A) the debt service coverage ratio. B) the rate covenants set forth in the indenture. C) the tax rates of nearby municipalities. D) the quality of the facilities management.

the tax rates of nearby municipalities. The credit rating of a revenue issue would not be affected by tax rates in surrounding municipalities.

If an investor is long 5 Dec puts on the Canadian dollar, these options will expire in December on

the third Friday of the expiry month. Currency options, like equity options, expire on the third Friday of the expiry month.

In a municipal underwriting, the interest cost calculation discounts future interest payments to arrive at present value. This interest cost calculation method is A) the true interest cost. B) the relative interest cost. C) the net interest cost. D) the simple interest cost.

the true interest cost. When an issuer discounts future interest payments to arrive at present value, the interest cost method being used is the true interest cost. This method takes into consideration the time value of money.

The amount paid into a defined contribution plan is set by

the trust agreement. A defined contribution plan's trust agreement contains a section explaining the formula(s) used to determine the contributions to the retirement plan.

If a customer buys bonds that have already been called, the confirmation must disclose all of the following except A) the yield to original maturity. B) the redemption date. C) the redemption price. D) the yield to redemption.

the yield to original maturity. A customer who has purchased a called bond has probably purchased one that has been prerefunded. The yield to original maturity is no longer a factor.

An investor purchases a newly issued convertible bond at par. The bond is convertible at $25. Three years later, the underlying common stock is trading at $33 per share. If the investor sells the bond at the parity price,

there is a long-term capital gain of $320. This question involves several steps. The first is to determine the conversion ratio in shares. A bond convertible at $25 per share has a share conversion rate of 40 shares ($1,000 ÷ $25). The second step is to compute the parity price. That is, what are those 40 shares worth? Multiply 40 shares by $33 per share and that equals $1,320. When the bondholder sells the bonds at parity, that $1,320 is received. The $320 profit over the $1,000 initial cost is a long-term capital gain. An alternative that might be easier for some is to look at the appreciation of the stock. It is $8 per share higher than the conversion price of $25. That represents an increase of 32% (8 ÷ 25). If the bond is at parity with the stock, its price must be 32% higher and that brings us again to the $1,320 price.

Each of the following is true about stop orders except A) they can limit a loss in a declining stock. B) they become market orders when there is a trade at, or the market passes through, a specific price. C) they are the same as limit orders. D) they can accelerate the advance or decline of a stock's price if executed.

they are the same as limit orders. A stop order becomes a market order once the market price reaches or passes through the stop price. An investor in a long position can use the sell stop order for protection against a market decline. When a large number of stop orders are triggered at a particular price, the advance or decline of the market at that point can be magnified.

Most mutual funds operate as regulated investment companies. This means that A) they register with the SEC under the Investment Company Act of 1940. B) they register with the SEC under the Investment Advisers Act of 1940. C) they qualify for special tax treatment under Subchapter M of the Internal Revenue Code. D) their principal underwriter (sponsor) is a FINRA member.

they qualify for special tax treatment under Subchapter M of the Internal Revenue Code. The term regulated investment company refers to the Internal Revenue Code (IRC) section offering favorable tax treatment to qualifying investment companies. Triple taxation of investment income can be avoided if the mutual fund qualifies under Subchapter M of the IRC. To avoid taxation under Subchapter M, a fund must distribute at least 90% of its net investment income to shareholders. The fund then pays taxes only on the undistributed amount. This rule applies to management companies (open-end and closed-end) and UITs. That means ETFs are also included. Although not investment companies registered under the Investment Company Act of 1940, REITs can also take advantage of Subchapter M's tax benefits. It is true that mutual funds register with the SEC under the Invesment Company Act of 1940 (their portfolio managers register as investment advisers under the Advisers Act) and their principal underwriter is a FINRA member, but those statement have nothing to do with the question.

If a moral obligation bond goes into default, the only way bondholders can be repaid is

through legislative apportionment. A moral obligation bondis a state- or local-issued, or state- or local-agency issued, bond. If revenues or tax collections backing the bond are not sufficient to pay the debt service, the state legislature has the authority to appropriate funds to make payments. That is the only way the bondholders can be repaid. The legislature rarely reneges on its moral obligation.

A customer buys a municipal bond in the secondary market at 96 that has four years to maturity. Two years later, the customer sells the bond at 99. The tax consequences of this investment are

two points of ordinary income and one point of capital gain. When a municipal bond is purchased in the secondary market at a discount, the annual accretion is taxed as ordinary income. The annual accretion is one point per year (four points divided by four years to maturity). Therefore, when the bond is sold two years later, its cost basis is 98. If the bond is sold at 99, there is a long-term capital gain of one point per bond. Also, there is ordinary income taxation on the accretion of two points.

A customer requests information on a variable life insurance policy and asks her registered representative to circle the important information in the prospectus and information he thinks will be of special interest to her. This is permitted

under no circumstances. The prospectus is a legal document and may not be altered.

One of your clients purchased shares of the Ajax Mutual Fund several months ago. At that time, the net asset value (NAV) of the fund was $17.20. Today, the NAV is $17.56, and your client wants to know what accounts for the difference. You should advise her that the difference likely represents

unrealized appreciation. The NAV of mutual funds is marked to the market daily; the increase reflects higher market prices for the securities in the fund's portfolio.

It would be correct to state that hedge funds typically have

unrestricted share concentration limits. Hedge funds are unregistered and have no restrictions on concentration of their assets. Mutual funds, but not hedge funds, register with the SEC. Registration involves making detailed disclosures as required in both the Securities Act of 1933 and the Investment Company Act of 1940. Furthermore, investment strategies that are generally forbidden to mutual funds, such as selling short and trading on margin, are found in most hedge funds. Because of the complexity of hedge funds, the initial investment minimums are typically well over $100,000.

Last week, your customer's margin account showed SMA of $6,000. As of the close of business yesterday, the margin account client had a long market value of $50,000 and a debit balance of $40,000. This client

will receive a maintenance margin call for $2,500. When the equity in a long margin account falls below 25% of the market value, the customer receives a maintenance margin call for the amount necessary to bring the account back to 25%. A market value of $50,000 requires at least $12,500 in equity. The account currently has only $10,000 in equity ($50,000 minus $40,000). Therefore, a call will go out for a prompt deposit of an additional $2,500. SMA cannot be used to meet a maintenance call, only an initial margin call. Depositing fully paid marginable securities is another option. In our question, it would be enough securities to bring the LMV in the account up to 4/3 times the debit balance. $40,000 × 4/3 = $160,000 ÷ 3 = LMV of $53,333.33. With LMV of $50,000, the additional needed is $3,333.33

Three brothers open a joint account instructing you that if one of them dies, they want the cash and securities in the account to go to the remaining parties to the account. The account should be opened

with right of survivorship. Using tenants with right of survivorship, each brother's interest in the account would go to the surviving brothers.

In a competitive offering of municipal bonds, the issue is usually awarded to the syndicate that offers to sell the bonds

with the lowest net interest cost to the issuer. In a competitive underwriting for municipal bonds, competing syndicates submit bids to the issuer. The issuer (or representative) examines the bids to determine which bid provides the issuer with the lowest net interest cost.

A bond analyst plots the yields of AAA corporate bonds and compares them to the yields of U.S. Treasury bonds with similar maturities. This is known as

yield curve analysis. The plotting of bond yields results in a curve, usually one where the longer the time to maturity, the higher the yield. The term yield curve analysis is the proper way to describe comparing the yields of highly-rated corporate bonds to those of Treasury bonds. When the spread between the yields is narrow, economic conditions in the United States are generally favorable. If the spread (sometimes called the credit spread) widens, it is generally a sign of a worsening economy. ** This question deals with material not covered in your LEM, but it relates to recent rule changes and/or student feedback.

A customer purchases 10 8% Treasury notes at 101-16. What is the dollar amount of this purchase?

$10,150 Though the denomination of the T-notes purchased is not given, always assume par ($1,000) unless told differently in the question. Remember that government notes and bonds are quoted in 32nds. Therefore, a quote of 101-16 means 101 plus 16/32. 101 + 1/2 = $1,015; $1,015 × 10 bonds = $10,150.

A married couple owns Class A shares of the KAPCO Balanced Fund as follows: The older of the pair has an individual account with a current value of $10,000. The other individual's account is valued at $20,000, and they have a JTWROS account valued at $12,000. KAPCO offers rights of accumulation and has breakpoints at $25,000 (4%), $50,000 (3%), and $100,000 (2%). How much will the sales charge be if they invest an additional $15,000 into the JTWROS account?

$450. We combine all of the accounts under rights of accumulation. Let's chart them: Older:$10,000 Younger:$20,000 Joint: $12,000 Total value: $42,000 Add new purchase:$15,000 New account total:$57,000 Rights of accumulation means that we add a new purchase to the value of the existing accounts. If that total reaches or exceeds the breakpoint, the entire purchase is made with the lower sales charge. The question tells us that the next breakpoint is at $50,000. Once that dollar amount is reached (or exceeded), the sales charge on all new purchases is reduced from 4% to 3%. In this question, adding the $15,000 purchase to the existing $42,000 takes the customer's account well past the $50,000 breakpoint. That means the entire new purchase is charged 3%. Therefore, $15,000 times 3% equals $450 in sales charge.

A dealer that quotes a concession of half to another dealer means

$5 per $1,000 of par. A concession between broker-dealers on secondary market transactions is a discount from the yield that the broker-dealer is quoting. It is common for a broker-dealer to offer bonds to other broker-dealers at a price, less the concession. The net price becomes the purchase price for the buying broker-dealer. If simultaneously sold to a retail account, the markup is from the net price paid. If not simultaneously retailed but held in the broker-dealer's inventory, it is fair for the broker-dealer to market her inventory and mark up from there for retail sale.

A municipal bond dealer purchased $100,000 of municipal revenue bonds at a 3% yield less 3/8ths. The dealer marks the bond up by ½ point and reoffers them to his customers. The compensation to the firm on each bond is

$5.00. The price paid for the bonds is irrelevant. The important fact is that the dealer put on a markup of ½ point. That is $5 per bond.

If an investor purchases 2 Dec 81.50 Swiss franc calls at 2.5 (each contract is 10,000 francs), how much does the investor pay for the position?

$500. One call offered at 2.5 is equal to $250 multiplied by two contracts, for a total premium of $500.

A customer establishes the following positions: Buy 100 JMB at 28 Buy 1 JMB Dec 25 put at 2 What is the maximum potential loss?

$500. The investor loses money on the long stock position when the market value falls. With the purchase of the put, the investor can sell the stock for no less than the strike price, but also loses the premium. In this example, the investor loses a maximum of $3 on the stock (28 − 25) plus the premium of $2, for a total loss of $500 on 100 shares.

In a new margin account, a customer sells short $60,000 worth of ABC stock and deposits $30,000 to meet the Regulation T requirement. If the value of ABC falls to $55,000, the special memorandum account (SMA) balance in the account would be

$7,500. For every $1 decrease in market value in a short account, $1.50 of SMA is created. Therefore, if the market value falls by $5,000, the SMA balance would be $7,500.

Cement Mixer Corporation has 1 million shares of convertible preferred stock and 2 million shares of common outstanding. Each share of preferred can be converted into half a share of common. The preferred stock is selling at $17.50, and the common stock is selling at $35.75. If all preferred shares were converted, how many shares of common stock would be outstanding after conversion?

2,500,000 One million shares of preferred, each converted to half a share of common, is 500,000 common shares, and 500,000 shares after conversion, added to 2 million shares of common previously outstanding, equals 2.5 million common shares.

For how long must a FINRA member firm keep a record of written customer complaints?

4 years. According to FINRA Rule 4513, which went into effect in December 2011, written customer complaints and any correspondence related to the complaints must be kept for 4 years.

United States Treasury notes are intermediate length securities. Treasury notes are not issued with maturities of

4 years. U.S. Treasury notes are issued with maturities of 2, 3, 5, 7, and 10 years.

A fundamental analyst is reviewing GEMCO's financial statements. The company has a current ratio of 4:1, a price-to-earnings (P/E) ratio of 12:1, $10 million in 5% debentures, and net income after preferred dividends of $4 million. If the current market price of GEMCO stock is $60 and the company pays dividends at a rate of $0.75 quarterly, the dividend payout ratio is

60%. As with many computational problems, there is some unnecessary information given. The current ratio is irrelevant, and so is the information on the debentures. What is needed is the amount available to pay the common so that we can compare that to the amount actually paid. We see that $0.75 in quarterly dividends are paid. That is equal to $3 per year. The next key is determining the earnings. With a market price of $60 per share and a price-to-earnings ratio of 12:1, the earnings per share must be $5. The dividend payout ratio should be thought of as "dividends paid out of earnings made." The dividends paid are $3; the earnings made are $5. That is a 3 to 5 ratio, or, as usually expressed in percentage form, 60%.

In an effort to raise additional capital, which type of registered investment company may issue debt securities?

A closed-end investment company. The capital structure of closed-end investment companies differs from other investment companies. Closed-end investment companies may issue debt securities, as well as preferred stock. Open-end companies and UITs can purchase debt securities for their portfolios but can only issue one class of equity.

Which of the following positions will be profitable if the market price of the underlying asset is equal to the exercise price at expiration date?

A short put. At expiration, if the strike price and the underlying asset price are the same, the option has no value and will expire unexercised. Option writers (short positions) benefit when this happens because they earn the entire premium. Those with long option positions need price movement to profit. While a short option position profits without movement, a short stock position only profits when the market price goes down.

If a customer wishes to buy 1 XYZ option and sell another XYZ option, but he is not willing to spend more than $300, which of the following orders should be entered?

A spread order. A spread involves the simultaneous purchase and sale of different option contracts of the same type. A spread incurs a gain or loss depending on what happens to the difference in the premiums between the two contracts. Because this investor wants to limit his risk to $300, he would buy the spread at a net debit of $300 or less. (This is one order, not two.)

Which of the following would be most likely to require a mandatory sinking or surplus fund? A) A general obligation B) A tax anticipation note C) A public housing authority. D) A water and sewer revenue bond

A water and sewer revenue bond. Sinking or surplus funds force revenue bond issuers to set aside a portion of their revenue for debt retirement.

Dividends may be paid to holders of

American depositary receipts (ADRs). ADR owners have most of the rights common stockholders normally hold. One of these includes the right to receive dividends when declared. Rights and warrants allow holders to purchase stock from a corporation, and Treasury stock is stock that has been issued by the corporation and then bought back. Rights, warrants, or Treasury stock holders do not have the right to receive dividends.

A technical analyst would find which of the following to be a bullish indicator?

An increase in the short interest. As the short interest increases, it is a bullish signal to technicians. Those short sales are going to have to be covered one day and that buying pressure is bullish on the stock. A head and shoulders top is bearish - it signifies that the market has reached a top. Odd-lot purchasers (the little guy) are always buying and selling at the wrong time. When they are buying more than selling, it is a sign that the market price of the stock is soon going to fall. When the advances outnumber the decliners, it is bullish. When it is reversed, as in the answer choice, it is bearish.

Which of the following is considered a double-barreled bond?

Bridge authority revenue bonds guaranteed by the full faith and credit of a city. Double-barreled bonds are backed not only by a specified source of revenues, but also by the full faith and credit of a municipal issuer with authority to levy taxes. Even though they are rated and traded as if they were general obligation bonds, it is proper to include them in the revenue category because the initial backing is from revenue. The additional backing of PHAs is the full faith and credit of the U.S. government—not the issuer. The additional backing of moral obligation bonds are legislative appropriations, which are not mandatory.

An investor purchases 200 shares of ABC common stock, but is concerned about market risk. While the investor cannot use diversification to reduce market risk by investing in the same asset class, they can hedge against the risk. Which of the following options positions would be used as such a hedge?

Buy 2 ABC puts. The investor should buy two ABC put options (the investor is long 200 shares). Options can be used as a hedge (protection) against risk or can be used to provide income. To use as a hedge, the investor should buy (go long) the option. To use as income, the investor should sell (go short) the option. Here the investor is using the option to hedge, so they will be buying the option, not selling. Investors use long puts to hedge a long stock position and long call options to hedge a short stock position.

A direct participation program (DPP), organized as a limited partnership, must avoid at least two characteristics of a corporation. Which two characteristics are easiest to avoid?

Continuity of life and freely transferable interests. Continuity of life and freely transferable interests are the easiest to avoid. The limited partnership is formed to exist for a limited time, and general partner (GP) must approve any transfer of interests. Centralized management is the hardest characteristics to avoid because management of the program is the responsibility of the general partner (GP), so management is centralized.

A registered representative must obtain written verification of an investor's net worth for which of the following investments? A) Variable contract B) Mutual fund C) Direct participation program D) Real estate investment trust

Direct participation program Before an investor can become a limited partner, the investor must provide a written verification of net worth. The investor is accepted as a limited partner only when the general partner signs the subscription agreement.

An investor sold 100 shares of RAN common stock short at $50 per share. The RAN is now at $38. The investor is still bearish on the stock but would like to protect that gain. What would you recommend if this were your client?

Enter a buy stop order at $40 Buy stop orders are used as a protective tool for short sellers. In this case, if the stock should begin to rise from its current price of $38, once it reaches or exceeds $40, a buy order at the market is entered. The stock purchased is used to cover the short position, and the investor's profit is the $50 sale price minus the cost of the purchase. Buy stops are always placed above the current market. Because the investor is short, the only protective order would be a buy, not a sell (you buy protection)..

Municipal Securities Rulemaking Board (MSRB) rules for NYSE member firms are enforced by

FINRA. The board's rules are enforced by FINRA for securities firms. The MSRB has rule-making authority but no enforcement or examination authority.

All of the following records must be kept for six years except A) FOCUS reports. B) the general ledger. C) customer account records. D) trade blotters.

FOCUS reports. FOCUS reports (it will not be tested, but the acronym stands for Financial and Operational Combined Uniform Single) are part of the large group of records with a three-year retention requirement. Blotters, ledgers, and most records containing detailed information about customers have a six-year requirement.

The purchaser of a general obligation (GO) municipal bond should be concerned with I. property tax assessments. II. the maintenance covenant. III. market risk. IV. feasibility studies.

I. property tax assessments. and III. market risk. GO bonds are issued by municipalities and, like all debt instruments, are subject to interest rate changes (market risk). Ad valorem (property taxes) are the primary source of debt funding for municipal GO bonds and are based on property assessments. Feasibility studies and maintenance covenants are associated with municipal revenue bonds where user fees from municipal projects and facilities are used to fund the debt.

A client of your member firm dies. In correct order, you should I. freeze the account. II. accept orders from the executor. III. obtain the death certificate and other legal documents. IV. cancel all open orders.

IV, I, III, II cancel all open orders, freeze the account, obtain the death certificate and other legal documents, accept orders from the executor Upon the death of a client, all open orders must be canceled. The account is then frozen until proper legal documentation is received. Once that has occurred, the executor may begin conducting activity in the account.

If a mutual fund's objective is income, it would not hold which of the following securities in its portfolio?

Income bonds. A fund designed to generate current income for its shareholders would not hold an income bond, also known as an adjustment bond. Income bonds pay interest only if the issuer has enough earnings to do so. They are often issued by companies coming out of bankruptcy. As a result, these bonds tend to trade like zeroes.

Which of the following are excluded from the Trade Reporting and Compliance Engine (TRACE) reporting requirements?

Money market securities. TRACE is the FINRA-approved trade reporting system for corporate, government agency, and Treasury securities trading in the over-the-counter (OTC) secondary market. While most corporate debt securities, ABS, and CMOs are TRACE-eligible, there are exclusions, such as debt of foreign governments, money market instruments, and debt securities that are not Depository Trust Company eligible.

Toby Jensen originally purchased 400 shares of CSC stock on margin at a price of $60 per share. The initial margin requirement is 50%, and the maintenance margin is 25%. CSC stock price has fallen dramatically in recent months, and it closed today with a sharp decline, bringing the closing price to $40 per share. Based on FINRA requirements, will Jensen receive a maintenance margin call?

No, the account meets the minimum maintenance margin requirement. Minimum maintenance requires equity equal to 25% of the current market value. If the price per share is $40, then the value of the 400 shares is $16,000. That would make the minimum equity requirement $4,000 ($16,000 times 25%). The initial purchase was $24,000 resulting in a debit balance of 50%, or $12,000. If the LMV is $16,000 and the debit balance is $12,000, the account is right at the minimum maintenance level of $4,000. Toby might be below the house maintenance, but because that is not given in the question, there is no way to tell.

All of the following statements regarding planned amortization class (PAC) collateralized mortgage obligations are true except A) PACs have higher yields than comparable TACs. B) PACs have companion tranches. C) PACs have a more certain maturity date than comparable TACs. D) PACs have a lower-than-average prepayment risk.

PACs have higher yields than comparable TACs. PACs have two companion tranches: one to absorb prepayments and one to buffer against extension risk. Because there is less risk and a more certain maturity date, PACs tend to have lower yields than comparable TACs.

Which of the following mortgage-backed securities would provide investors with the most predictable maturity date?

Planned amortization classes (PACs) PACs are planned amortization class collateralized mortgage obligations and have established maturity dates. Prepayment risk is transferred to the PAC companion—or support—class bonds.

An investor, with a well-diversified portfolio oriented toward growth, has 60% invested in the stocks of 28 different companies. She would like to hedge the downside risk for the equities and is comfortable using options to do so. Which of the following is most suitable?

Purchase index option puts Selling options to hedge adds income to the account (premiums received), but the protection is limited. The best hedging protection is to purchase options, and to hedge long stocks, purchasing puts is the most suitable. With so many stocks to hedge, doing so individually would not be cost effective due to commissions. On the other hand, hedging the entire portfolio with index options allows the hedge to be done in fewer—if not a single—transaction.

An experienced investor wants to allocate 10% of an existing portfolio to owning real estate but does not want to maintain properties, be a landlord, or wait if cash is needed. Which of the following choices would be suitable, given the investor's objectives?

Purchase shares of an equity REIT Equity real estate investment trusts (REITs) are a way to have an ownership interest in real estate without having to manage properties or worry about collecting rents. REITs trade on exchanges and over the counter; therefore, they are liquid investments. Both of these characteristics meet the investor's objective and make equity REITs the most suitable recommendation of those offered here. A mortgage REIT does not own real estate, so it will not meet the investor's objective.

A customer wishes to diversify an investment portfolio to include real estate. However, the customer is concerned that real estate tends to be illiquid. Which of the following would be a suitable recommendation?

Real estate investment trusts. REITs (real estate investment trusts) allow investors the opportunity to invest in real estate and provide a high level of liquidity because they are publicly traded on exchanges and OTC. The real estate DPP does provide an opportunity to invest in real estate, but it is considered illiquid because there is generally no available secondary market. When a mutual fund is a diversified company, it means that it complies with the 75-5-10 rule, not that its portfolio is diversified into real estate. GNMA securities are debt obligations and do not provide the equity diversification this investor is seeking. Please note: As pointed out in the LEM, there has been a growth in the number of nontraded REITs. Obviously, they do not have the liquidity of those that are publicly traded. For exam purposes, unless something in the question indicates that the REIT is nontraded, you can safely assume it is a liquid investment.

All of the following statements about the special memorandum account (SMA) are true except: A) nonrequired cash deposits generate SMA. B) it is a line of credit. C) SMA in a long margin account decreases when the market value decreases. D) the SMA balance may be used to meet the Regulation T requirement for purchases.

SMA in a long margin account decreases when the market value decreases. The amount of SMA in a long account decreases only when it is used, and is unaffected by market value decreases.

All of the following issue mortgage-backed securities except A) Ginnie Mae. B) Freddie Mac. C) Fannie Mae. D) Sallie Mae.

Sallie Mae. Sallie Mae is the name for the Student Loan Marketing Association, which does not issue mortgage-backed securities. The backing for those is, as the name states, student loans. Ginnie Mae (Government National Mortgage Association), Fannie Mae (Federal National Mortgage Association), and Freddie Mac (Federal Home Loan Mortgage Corporation) do issue mortgage-backed securities as is clearly indicated in their names.

A new client, age 26, has begun to search for her first home. She's been told that finding the right home within her budget might take four to six months, depending on the availability of homes in the area she is targeting. With $45,000 currently in a checking account to use for the down payment, which of the following would represent the most suitable recommendation for those funds until the right home is found?

Savings account (cash), money market account, short-term T-bills (1-3 months). For short-term liquidity, savings or checking accounts are always options; money market funds would generally allow for a slightly better return, as would short-term T-bills. In light of the time horizon and liquidity needs, nothing long term (T-notes or bonds), illiquid (VAs, DPPs), or instruments not intended for short-term trading (mutual funds, particularly front load A shares) would be appropriate.

Which of the following types of mutual funds has capital appreciation as its investment objective? A) Municipal bond B) Income C) Balanced D) Specialized

Specialized. An objective of high-capital appreciation is most likely realized by a stock fund. A specialized fund is one that invests in stocks of one particular industry or region, and its main objective is capital or price appreciation.

Index options are frequently used to protect a portfolio against which of the following risk types?

Systematic Index options protect investor portfolios from the risk of overall market movement, also known as systematic risk.

An investor has losses on the sale of municipal bonds. Which of the following, for tax purposes, is true?

The losses can be applied against the gains on the sale of any other security. Losses on the sale of one investment can generally be deducted against gains on the sale of any other investment.

Tenants in common (TIC) ownership provides that a deceased tenant's fractional interest in an account is retained by which of the following?

The deceased tenant's estate. TIC ownership of an account provides that a deceased tenant's interest in an account is retained by that tenant's estate and not passed on to the surviving tenant.

A customer has contributed $1,000 a year for 10 years to his tax-deferred nonqualified variable annuity. The value of the separate account is now $30,000. If the customer takes a withdrawal of $10,000, what are the tax consequences?

The entire $10,000 is taxable as ordinary income. The $30,000 contract value represents $10,000 of contributions and $20,000 of earnings. When a partial withdrawal is made from an annuity, the earnings are considered to be taken out first for tax purposes (or last-in, first-out). Therefore, ordinary income taxes will apply to the entire $10,000. In addition, if the customer is not at least 59½, there will be an additional tax penalty of 10%.

A customer has a $75,000 bank CD that is about to mature, and she wants some recommendations from the registered representative handling her brokerage account. Which of the following items would be the least important when considering a suitable recommendation?

The name of the bank issuing the CD

Which of the following would not be a concern for an investor writing a naked option?

The premium the investor must pay for the contract. The writer of any option contract receives the premium; she does not pay it. A naked option writer would consider the risk/reward ratio, the loss potential, and the possibility that the contract would be exercised by the party who purchased it.

A customer goes long 1 ABC June 40 call at 4 when buying the option in their account on Tuesday, April 4. When is the settlement date and what is the amount due?

Wednesday, April 5; $400 The settlement date for an option trade is T+1 business day, meaning this trade will settle on Wednesday, April 5. On that day, the buyer of the option must have the funds to pay the premium in the options account. The premium per share is $4. Premium times contract size (100 shares) is the amount the buyer will pay, and the amount the seller will receive ($4 times 100 = $400). If the buyer of the call exercised the option, they would buy 100 shares of ABC stock for the exercise price (strike price) of $40 and so would buy the stock for $4,000. Regular way settlement on the stock trade when exercised would be T+2 business days.

Which of the following is not a factor when a communication to be distributed to the public is either being reviewed or approved within the broker-dealer?

Whether the piece will be distributed in written form or via electronic media. FINRA holds broker-dealers to certain general standards regarding all member firm communications. Consideration must be given to whether all statements in a communication are clear and not misleading, are balanced regarding the representation of risk and reward, do not omit material facts or make exaggerated claims, and do not imply that past performance can be projected to future outcomes. These standards would apply and be the same, whether the communication was distributed in written or electronic form.

You are reviewing the information on a new customer's options account agreement. Which of the following strategies would require the severest scrutiny for suitability purposes?

Writing uncovered call options. The riskiest of all options strategies is the uncovered (naked) call option. This is because the investor loses when the stock price rises and there is no limit (theoretically) as to how high a stock's price can go. In the case of a short put, the underlying security's price cannot drop below $0.00. Long positions, even with straddles, are limited loss strategies—the maximum loss is the premiums paid. Covered call options are a low-risk strategy.

One of your customers notices that the short interest on the NYSE is high. When she asks you for an interpretation, you should tell her that this signals

a bullish market. Even though short interest represents the number of shares sold short, many investors consider it a bullish indicator when this number is high. Each share that has been sold short must be replaced (covered) at some point. To replace the stock shorted, an investor must go into the market to buy that stock. When all of those short sellers have to buy back stock they shorted, it puts upward pressure on the prices of those stocks.

Nonmembers of a syndicate who are assisting in its sale of bonds buy the bonds at a discount called

a concession. Members of the syndicate buy the bonds at the offering price minus the takedown, and nonmembers buy at offering price minus a concession. The basis price is the yield to maturity.

A client writes 1 Jan 60 put and buys 1 Jan 50 put. This is

a credit bull spread; the investor wants the price to stay above 60. This is a put credit spread, and bulls sell puts. The 60 put is worth more because it has a higher strike price. Long the lower put is bullish; short the lower put is bearish.

A municipal bond, issued with a covenant that states, "If revenue collections are not sufficient to meet debt service requirements, the issue will be backed by the full faith and credit of the municipality," is known as

a double-barreled bond. When a municipal bond is backed by both a source of revenue and the taxing ability of the issuer, this is referred to as a double-barreled bond.

Purchasers of municipal revenue bonds are interested in knowing the priority of their claim on the revenues generated by the project. In general, the most senior position is held by

a gross revenue pledge. The simplest way to think about this question is to look at our paycheck. What is the higher number - your gross pay or your net (take-home) pay? You could also say, "Who has the first claim on my earnings?" The government and that is why the taxes come out of your gross paycheck rather than your net check. The concept is the same here. In a gross revenue pledge, interest to the bondholders is paid out of the gross revenues before any other deductions. In a net revenue pledge, certain expenses are paid first, and then, from what remains, the bondholders receive their due.

Broker-dealers are required to maintain customer identification programs and check the names of new clients against

a list compiled by the Office of Foreign Assets Control (OFAC). All financial institutions are required by federal law to maintain a customer identification program and check the identifying information against a list maintained by the OFAC for suspected terrorists or terrorist organizations.

A customer creates a long straddle by buying 5 ABC Nov 50 calls and 5 ABC Nov 50 puts, paying a total premium of $3,750. If ABC is at 56.50 at expiration, the customer has

a loss of $500. When an investor goes long a straddle, the expectation is that the stock's price will be volatile, but the direction (up or down) is uncertain. In this question, the stock's price increased putting the calls in the money. At expiration, the call positions would be closed by selling them for their intrinsic value ($56.50 market value minus the $50 strike = $6.50). With each contract representing 100 shares, the $6.50 equals $$650 and because there are five contracts, the total sale proceeds are $3,250. Compare that to the $3,750 initial cost and the result is a loss of $500. What about the put options? When the market price is higher than the strike price, the puts are worthless and expire unexercised.

A customer wants to open a new cash account and give her sibling trading authorization. The required documents to accommodate her request would be

a new account form and a limited power of attorney. When a customer wants to give trading authorization or discretionary privileges to a third party in a cash account, a member firm requires a new account form (as with all new accounts) and a limited power of attorney. A limited power of attorney gives the third party trading authority but prohibits that party from withdrawing assets (cash or securities) from the account.

If an investment company invests in a fixed portfolio of municipal or corporate bonds, it is classified as

a unit investment trust. A unit investment trust issues shares that represent units of a particular portfolio; management has no authority—or only limited authority—to change the portfolio. The portfolio is fixed, not traded.

Synapse Communication Corporation (SCC) is growing. To finance the expansion, the company has a $100 million debenture offering. Attached to the offering are five-year warrants to purchase SCC common shares. Each warrant allows for the purchase of two SCC shares at a price of $53 per share. Three years after the issue date, SCC stock is trading at $63 per share. Each warrant has

an intrinsic value of $20. A warrant has intrinsic value when the exercise price is lower than the stock's current market price. In this question, each warrant allows the holder to purchase two shares of a $63 stock for $53 per share. That is a $10 per share value times two. Therefore, intrinsically, the warrant is worth $20. With two years to go, it also has time value, but the question is not dealing with that. ** This question deals with material not covered in your LEM, but it relates to recent rule changes and/or student feedback.

A transaction that will create or increase a long position in an option contract is

an opening purchase transaction. One takes a long position by buying an option. Creating or increasing the position is opening not closing. A closing purchase transaction eliminates the position. An open writing transaction creates or increases a short, not long, position.

An investor buys a municipal bond at a discount. The bond carries a 3% coupon. At maturity, the bond will pay the face amount to the holder of the bond on the maturity date. In the meantime, the IRS requires accretion of the discount. That accretion is tax-exempt income when the bond is

an original issue discount bond. If the discount is an original issue discount (OID), the annual accretion is not taxable. An OID means that the bond's issuance price in the primary market was below par (a discount). As an OID, the annual accretion is considered part of the interest being paid by the issuer and that is why it is tax exempt. However, if the discounted transaction were in the secondary market, then the annual accretion would be taxable as interest income. That does not affect the tax-exempt status of the coupon interest paid. Please note: It is highly unlikely that your exam will ask about the tax treatment of the accretion when a secondary market purchase of an OID municipal bond occurs at a price above or below its accreted value. As we show you in the LEM, the test could ask about capital gain or loss, but we do not expect a question about figuring the tax on the accretion in this unusual case.

An investor, long 100 shares of XYZ at $23.50, writes 1 XYZ May 25 call at 2. At expiration, if XYZ is trading at $20, the investor has

an unrealized loss of $150. Breakeven is 21.50 (23.50 − 2). If the stock is trading at 20, the customer has an unrealized loss of $150.

The writer of an in-the-money put will receive the upcoming dividend from the underlying issuer if the contract is exercised

before the ex-date. When assigned, the put writer is obligated to purchase the stock. Provided the stock is purchased before the ex-date, its buyer becomes the owner of record on or before the record date and is therefore entitled to the dividend. This is no different from anyone else purchasing before the ex-date.

An analyst comparing revenues with expenses is most likely analyzing

cash flow. The analyst is most likely measuring the income statement for cash flow (money coming in against money going out). Working capital analysis—not the income statement—would involve examining the balance sheet's current assets and current liability entries. Capitalization analysis involves examination of long-term debt and stock issues. Liquidity analysis involves examining current assets and liabilities from the balance sheet.

All of the following must meet the nondiscrimination provisions of the Employee Retirement Income Security Act (ERISA) except: A) deferred compensation plans. B) defined benefit plans. C) profit-sharing plans. D) 401(k) plans.

deferred compensation plans. Deferred compensation plans are non-qualified, and therefore, do not have to meet the nondiscrimination provisions of ERISA. Meaning they can discriminate.

A company's dividend on its common stock is

determined by its board of directors. A common stock's dividend payment and amount are determined by the company's board of directors.

Many fixed-income investors diversify their portfolios by maturity. If one were investing in CMOs, that would be done by buying

different tranches. Although CMOs technically have maturity dates, it is rare for one to ever last that long. The standard way to vary the expected return of principal is by using different tranches. Serial bonds are generally issued by municipalities and are not mortgage-backed. Different issuers has nothing to do with maturities and yield-based options might be used as a hedge, but not to diversify maturities.

One of your clients has a profit in STV common stock. The purchase price was $40 per share and is now $60 per share. The client is concerned that the stock might backtrack and some of the profit might be lost. One way to protect that profit would be to

enter a sell stop order at $55. Stop orders, frequently called stop-loss orders, are used to protect an existing profit or prevent a loss. In the case of a client owning stock, the fear is that the stock will decline in price. Sell stop orders are always placed below the current market, never above. In this question, should the stock's price fall to 55 (or lower), the stop order will be triggered and a market order entered. This will preserve at least some of the profit. Sell limit orders are placed above the market, but that will never be executed unless the stock rises to at least $62. A sell limit order placed below the current market is, in effect, a market order and is immediately executed.

The price of DFEC common stock is $32 per share. Your customer owns one DFEC Sep 35 put purchased for a premium of 4. The option

is 3 points in-the-money. If an option has intrinsic value, it is in-the-money. Puts are in-the-money when the market price of the underlying asset is below the exercise price. The difference between the 35 strike and the 32 current market value represents 3 points of intrinsic value. Intrinsic value (the in-the-money amount) ignores the premium. However, the fact that the premium exceeds the intrinsic value by one point represents one point of time value.

Earlier in the day, you entered a customer order to buy 300 XYZ at 26.45 good til canceled (GTC). By late afternoon, you notice that XYZ is trading at your customer's limit price. At the close of trading, you contact the order desk and get a Nothing Done report because

of stock ahead. All limit orders stand in time priority.

The capitalization structure of an open-end investment company can include

one issue of common stock. Open-end investment companies may only issue shares of common stock. Preferred stock, bonds, and other forms of senior securities are not allowed. It is the closed-end investment company that can have a preferred stock and bond offering. Don't confuse what a mutual fund uses to raise money (the common stock it issues) with what it does with that money. The capital raised from the sale of shares may be used to buy whatever securities meet the fund's objectives.

A city and school district are coterminous. When evaluating the debt issues of the city, the school district debt would be considered

overlapping debt. The term overlapping debt refers to the issuer's proportionate share of the debt of other local governmental units that either overlap it (the issuer is located either wholly or partly with the geographical limits of the other units) or underlie it (the other units are located within the geographical limits of the issuer). In this case, the school district is probably within the geographical limits of the city. That's what coterminous means.

The City of Concord has floated a new bond issue to expand the public library. Concord has arranged for these bonds to be insured. If Concord defaults on these bonds for any reason, the insurance company will

pay the bondholders the principal and interest as scheduled. Bond insurance is a feature offered by many municipal bonds. The effect of the bond insurance is that both interest and principal will be paid as scheduled, over time, through the life of the bond. Payments are made to the bondholders by the insurance company.

The LLAW Manufacturing Company issued a 6.25% debenture 5 years ago. The bond is callable in seven years at 102 and matures in 15 years. The bond's current yield is 4.23%. If one of your customers decided to purchase this bond, they would have to understand they would be

paying a premium for the bond. The first thing to notice is that the current yield is below the nominal (coupon) rate. That automatically tell us the bond is selling at a premium. Whenever a bond is selling at a premium, in increasing order, the order of the yields is yield to call, yield to maturity, current yield, and nominal yield. Therefore, if the current yield is 4.23%, the yield to mature cannot be higher than that; it must be less. As we say in the LEM, "if you pay more, you get less" and "if you pay less, you get more." The issuer pays the call price when, and if, the bond is called.

Your broker-dealer has prepared an advertising piece for general distribution to all of its retail customers regarding numerous option strategies. Filing the piece with FINRA is

required at least 10 business days before first use or publication. Filing with FINRA is required at least 10 business days before first use or publication for retail communications having to do with options.

A corporation pays a 10% stock dividend to common stockholders. All the following are true regarding this dividend except: A) the total value of the position is unchanged when the dividend is paid. B) the beauty of stock dividends is that they are nontaxable. C) the cost basis per share is adjusted based on the stock dividend. D) the dividend is taxable in the year the sale of the shares takes place.

the beauty of stock dividends is that they are nontaxable. The stock dividend is taxable, but unlike cash dividends, which are taxed when received, stock dividends are taxable in the year the shares are sold. When the stockholder receives the additional shares, the cost basis is adjusted on a per-share basis with the total value of the position remaining unchanged. For example, if an investor owned 100 shares purchased at a price of $22 per share and the company paid a 10% stock dividend, the numbers would look like this. The number of shares owned is now 110 [100 + (10% of 100)] = 100 + 10. The adjusted cost basis per share (used when any of the shares are sold) is now $20 per share. The original cost is $2,200 (100 shares times $22 per share). After the stock dividend, the customer owns 110 shares, but there was no additional cost. Divide that original $2,200 by the new number of shares ($2,200 divided by 11) to arrive at an adjusted cost basis of $20 per share. The account value is still $2,200 (110 shares times $20 per share = $2,200).

A retiree contacts an agent to discuss investing his retirement savings of approximately $2.1 million; his investment objective is long-term growth. The representative and customer discuss the advantages and disadvantages of diversifying among five different mutual funds within two fund families, as opposed to purchasing just one fund. Consequently, the agent made the following purchase recommendations: XYZ Emerging Growth Class B: $495,000 XYZ Research Class B: $310,000 XYZ Investors Growth Stock Class B: $495,000 ABC Capital Enterprise Class B: $495,000 ABC Capital Opportunity Class B: $310,000 Total: $2,105,000 These recommendations are

unsuitable because Class A shares in either (or both) fund family could be purchased for a sales charge breakpoint discount at or near 0%. All mutual funds with Class B shares also have Class A shares. All Class A shares provide sales charge discounts at stated breakpoints. At a purchase of this quantity, the sales charge is likely 0%. That means that, just like with the Class B shares, all of the money is invested into the funds. The advantages of the Class A shares is that their operating expenses are lower than Class B shares and there is no back-end load to this investor if the funds were needed unexpectedly within a few years. As a practical—but probably not specifically tested—point, Class B shares are not sold in quantities higher than $100,000; Class A shares are invariably the better option.


संबंधित स्टडी सेट्स

Pneumonoultramicroscopicsilicovolcanoconiosis

View Set

Ch 18 Performance and Breach of a Sales and Lease Contract

View Set

Chapter 27: Disorders of Cardiac Function, and Heart Failure and Circulatory Shock

View Set

Chapter 1: Introduction to nursing

View Set

CH 13: Shaping Processes for Plastics

View Set